You are on page 1of 378

2

Jun 2021 2
0
2
1 Feb 2021 30

2 Oct 2020 63
0
2 84
Feb 2020
0

2
Oct 2019 121
0
Jun 2019 133
1
9 Feb 2019 145

2
Oct 2018 169
0
Jun 2018 182
1
8 Feb 2018 198

2
Oct 2017 227
0
Jun 2017 247
1
7 Feb 2017 251

2016
2015
Others 267
2013
2011

All FOP Recall till June 2021 with note 1 of 378


FOP JUNE 2021
EMQ
EMQ 1: Respiratory (100 Respiratory & ER)
OPTIONS: Reassurance, Nebulizer Salbutamol, IV Salbutamol,
IV saline, IV hydrocortisone, ENT referral, Oral
dexamethasone, Ipratropium bromide
A.18 months baby came with 3 days respiratory
symptoms, cough and mild decrease in feeding with
normal oxygen saturation spo2 98%. Mother was
anxious.
Reassurance
B.9 months baby came with Fever! coryzal symptoms
noisy breathing! harsh cough and intermittent stridor.
Oral dexamethadone
C. 3 yrs baby came with breathlessness and wheeze, his
spo2 88%, received frequent nebulization in ER with
no improvement.
IV Hydrocortisone vs ENT Refer

EMQ 2: GIT (100 GIT)


OPTIONS: Duodenal atresia, Pyloric stenosis, Cow MPA,
Malrotation.
A. 6 hrs baby delivered at home, presented with
hypotonia after 6 times greenish vomiting.
Duodenal Atresia

All FOP Recall till June 2021 with note 2 of 378


B.4 Weeks baby presented with vomiting which pass
to other wall of caravan where they live & keen to
feed after it.
Pyloric Stenosis
C.3-8 weeks baby with a history of diarrhea and
vomiting and wheezy chest and loss of weight. There is
also Hx of eczema and skin rash.
CAMP

EMQ 3: Genetic (100% Genetics)


OPTIONS: ¼, 1/2, 1/3 , 2/3, 1/1000, 1/100 ,
Population risk
A. What is the chance of a healthy mother & father
who have a sickle cell child to have another a ected
sibling? 1: 4
B. What is the chance of a father with hemophilia
disease to have a a ected boy?
Poupulation risk
C. What is the chance of a healthy appearing sibling of
a cystic fibrosis child to be a carrier?
2:3

EMQ 4: CNS (100 % Neuro )


OPTIONS: Complex partial seizure, vasovagal syncope,pseudo
seizure, Day dreaming.
A. Baby boy looks frightened, goes pale with
repetitive
Swallowing for 15-30 seconds after which he is tired.
Complex partial seizure
All FOP Recall till June 2021 with note 3 of 378
B. Girl in assembly loss her consciousness, suddenly
goes pale with twitches in hands and jerky
movements after that she regain consciousness and
become confused for seconds.
Syncope
C.Child noticed at school by the teacher he become
staring with learning difficulty. Mother didn’t notice
problem at home.
Day dreaming

EMQ 5: Limbing (100 Neuro & Neurodevolpment)


OPTIONS: Perthes, SUFE, Transient synovitis, Reactive
arthritis, Septic arthritis, Discitis, Spinal tumor
A. 4 year after URTI 2 days, C/O limitation of abduction
at hip join and can’t weight bearing.
Tranient Synovitis
B. 4 year boy with history of limping for 5 months O/E
there is limitation to external and internal rotation
and limited Flexion and pain referred to knee.
Perthe's Disease
D. 12 years old came with local tenderness on lumbar
spine and pain in the leg . Normal examinassions in all
joints.
Discites

EMQ 6: Lymphadenopathy (100% Infection)


OPTIONS: All, EBV, Reactive lymphadenitis, Atypical
mycobacterium, Brucellosis, CML, HIV, mycobacterium TB,
NHL, Rubella.
All FOP Recall till June 2021 with note 4 of 378
A. 5 yrs Bangladesh boy recently arrived to UK with
Hx of tiredness and fever on & o for 3 weeks with
paler. O/E: liver 3 cm and spleen 2 cm, bilateral cerv. LN
0.5 X 1 cm.
ALL
B. 3 years Asian presented with lump for one month on
the RT side of the neck, baby was a symptomatic, A
Febrile vaccinated BCG. O/E: there is lymph node
2 X 1 cm in anterior triangle of the neck for last
few weeks & discoloration over swelling, non tender
otherwise the pt. is vitally stable with normal X- ray,
ttted with Amoxicillin 10 days without response.
Atypical Mycobactrium TB
C. 6 yrs given ttt of Tonsillitis 1 month ago with small
mobile lymph nodes & non-tender 0.2-0.5cm.
Reactive Lymphadenopathy.

BOF
BOF 1: Concerning the Migraine in children. which of the
following is the most accurate statement ?
1. Visual aura is common.
2. Family history is needed in diagnosis.
3. It is rarely diagnosed before 5 years of age.
4. Must be excluded if unilateral.

BOF 2: Boy presented polyuria & polydipsia. Weight and


height at 5th centile, urine osmolality 200. What is the
diagnosis?
All FOP Recall till June 2021 with note 5 of 378
1. DM
2. RTA
3. Nephrogenic diabetic insipidus
4. Central diabetic insipidus
5. Habitual drinking

BOF 3: 15-year girl with regular period at first yrs then


become
Irregular, there is brown pigmentation in the neck & axilla,
also there is excessive facial hair & striae. She is over
weight. What is the diagnosis?
1. PCOS
2. Cushing $
3. Neurofibromatosis type 1
4. CAH
5. BW$

BOF 4: 3 year old boy started to walk at 18 month then


after URTI, he was reluctant to walk, he doesn’t know the
color and can’t do 3 steps commands then he stop walking.
O/E : he has severe lordosis, waddling gait and symmetrical
muscle weakness. What is investigation you will do?
1. ESR
2. Dystrophin gene
3. CK
4. TFT ?!
5. Microarray

All FOP Recall till June 2021 with note 6 of 378


BOF 5: Known case type 1 DM presented by DKA , RBS 32,
ketone 3 (normal less 0.6), father has type 2 DM, and
grandfather was receiving oral hypoglycemic. What advise
you will tell the family?
1. He will need lifelong insulin
2. Need further investigation
3. Low carbohydrate diet

BOF 6: 14 month old boy admitted due to URTI, you are


called urgently to attend because the child is having tonic
clonic seizure. When you arrived it was 2 minutes of
seizure, blood glucose 7.6 and senior nurse was attending
and applied oxygen mask. What you will do next?
1. Obtain IV access
2. Observation
3. Buccal midazolam
4. IV lorazepam

BOF 7: Baby received methadone with noisy breathing,


SPO2 96% and HR 100 b/m, RR 18. What to do now?
1. Secure airway
2. Naloxone
3. Intubation
BOF 8: Baby presented with 7 days Hx of fever, cracked
lips, red eyes, and cervical lymphadenopathy. He received
2 days antibiotics then rash appear. Lab Investigations:
high ESR, platelet 250, normal WBC and HB. what to do to
guide for further management?
1. Chest X-ray
All FOP Recall till June 2021 with note 7 of 378
2. Echo
3. ASOT
4. ECG

BOF 9: 5 weeks old baby on breast feeding came with


coryzal symptoms , the midwife noticed he is jaundiced,
total bilirubin 120, direct 90 with liver 1 cm below costal
margin. What is the diagnosis?
1. Liver dis. Biliary atresia
2. Galactosemia
3. Hepatitis A.
4. Breast milk Jaundice
5. Spherocytosis.

BOF 10: 3 weeks infant of Chinase hepatitis b surface Ag


positive mother, she refused to give the vaccine at birth
and the the baby is well and thriving and breast feeding
and found to be jaundiced and mustard stool color and
baby is well and his weight on 50th centile. What is the
diagnosis?
1. Neonatal hepatitis
2. Breast milk jaundice
3. Biliary atresia
4. Hepatitis B virus

BOF 11: 4 year baby came with intermittent abdominal pain


in last 4 weeks, vomiting 4-6 times in last 48 hours, and last
24 hours was greenish. What is your diagnosis?
1. Malrotation
All FOP Recall till June 2021 with note 8 of 378
2. Intussusception
2. Duodenal atresia
3. Viral gastroenteritis.
4. Lymphadinitis

BOF 12: 14 years girl known case of Down syndrome came


with abdominal distention and low centile weight and hight
on Down chart, he live with her grandparent and her
mother abandoned her and her TFT normal last year. What
is the diagnosis?
1. Celiac dis.
2. Psychological deprivation
3. Hypothyroidism
4. ALL

BOF 13: 18 month child came with vomiting, tachycardia,


pale, bloody diarrhea and lethargic. Investigation blood urea
39, upper normal, Hb12, WBC 14, Plt. 140, CRP 20, PH 7.2,
HCO3 11. What is the diagnosis?
1. Bacillary dysentery
2. Diverticulitis
3. Intussusception
4. E- coli
5. Mesenteric lymphadenitis

BOF 14: 14 year old came with DKA, O/E tachycardia 120,
CRT 2 sec. and sever dehydration. What is the maintenance
fluid of DKA will be?

All FOP Recall till June 2021 with note 9 of 378


1. Normal saline
2. NS + D5%
3. NS + D 10%
4. 0.45 NS with D 5%
5. 500 ml 0.9 NACL and 20 meq KCL.

BOF 15: 1 year child came with recent onset of right


Convergent squint, O/E: his HC 91% centile, weight and
height
on 5TH centile, he can cruise around furniture and say 2
words. What is the most common pediatric ophthalmological
Cause?
1. Right rectus muscle injury
2. Congenital cataract
3. Optic glioma
4.Errors of refraction.

BOF 16: Child known diabetic on basal and bolus insulin,


and after a football class he fall down and become drowsy,
chatty , nurse measure the glucose 2.5. what is the best
action to do?
1. Call the ambulance
2. Give IM glucagon
3. Give 200 ml (1/2 cup ) sugary fluid.
4. Iv glucose.

BOF 17: Full term neonate was admitted NICU 3.7 kg after 4
hours he developed hypoglycemia and received
All FOP Recall till June 2021 with note 10 of 378
maintenance 150 ml / kg / day 10 % w and withdrawn
gradually and become full feeding with close follow up of
blood sugar then before discharge he developed
hypoglycemia. what to do next?
1. Stop feeding and start IVF
2. Investigate for hypoglycemia
3. Continue formula feeding.
4. Do CT.
5. Give IV hydrocortisone

BOF 18: 10 year old child known case of DM presented


with persistent vomiting and ketones was +++ in blood RBS
2.3. what to do next?
1. Dextrose 10% 2 ml / kg
2. Dextrose 10% 10 ml/ kg
3. Dextrose 5% 10 ml/ kg

BOF 19: 8 weeks baby with breast milk jaundice and


meningitis what to give
1. Ceftriaxone
2. Cefotaxim
3. Gentamicin
4. Clindamycin
BOF 20: patient diagnosed with measles after 10 days
developed seizure tonic clonic and received lorazepam and
seizure stopped and drowsiness and unresponsive and
fundosopy is normal and no focal abnormality
1. CT and MRI
2. give measles immunoglobulin’s
All FOP Recall till June 2021 with note 11 of 378
3. immediate lumbar puncture
4. EEG

BOF 21: Teenager about to do GSCE examination came in the


afternoon before the exam and developed dysuria fever 38
and headache and dipstick was positive nitrite and negatives
others. What will be your action?
1. Give broad spectrum antibiotic and go to exam next day
2. Give trimethoprim and send culture
3. Admit and defer the exam
4. The manifestation due anxiousness of exam

BOF 22: 10 years girl with fever and no focus and non
specific symptoms and urinalysis wbc 50. All cultures is
negative leukocyte esterase negative and nitrite negative
1. Non specific viral infection
2. Bacterial vaginitis
3. UTI

BOF 23: 3 years old girl with 1-day fever 38 went to gp and
diagnosed her as URTI what is the best advice to the mother
1. Wrap and put in bed
2. The fever usually subside in 3 days
3. Come tomorrow to assess to give antibiotics
4. Return back if fever not subsided in 3 days & give
antibiotic
5. Return back if the girl start to pull her ear to give
antibiotics

All FOP Recall till June 2021 with note 12 of 378


BOF 24: 15 years old initially was active recently became
irritable and less feeding and sleepy and keep herself in her
room and not keen to go out with her friends. she claimed
that she is fussy and overactive. Don’t like to eat at home
and she said she don’t like the home meals on examination
she was quiet and no eye contact. What is the diagnosis?
1. Eating disorders
2. Depression
3. hypothyrodism

BOF 25: What is the fine motor milestone of 3 years?


1. Draw circle
2. Bridge of 12 cups
3. Draw square
4. Used the scissors
5. Cut the paper with 2 equal pieces

BOF 26: What is the most important at 6 months refer for


surgery
1. Inguinal hernia
2. Phimosis
3. Epigastric hernia
4. Umbilical hernia
5. Hydrocele

BOF 27: baby 6 missed hearing screening at birth. What to


do now?
1. Evoked brain stem response
2. Tympanometry
All FOP Recall till June 2021 with note 13 of 378
3. Distraction test
4. Visual reinforcement audiometry
5. Pure tone audiometry

BOF 28: 2 years child not saying any word he goes to the
nursery and the teacher at the nursery reported that he is
not interacting with other children he passed his neonatal
hearing screening test and his neurological examination is
free. What is the diagnosis?
1. Audiological assessment (100 Neurology)
2. Refer to speech and language therapy
3. Refer to developmental; assessment for autism
spectrum disorder
4. Refer to education psychatrist

BOF 29: girl 3 years old on salbutamol inhaler prn and


flexotide inhaler 100 micro bid and she got daily night
symptoms daily in last 3 weeks. What next step to add
1. LTRA
2. LABA
3. increase dose of ICS
4. oral prednisolon

BOF 30: child with periorbital oedema then become painful


with ecchymosis and painful eye movements. What to give?
1. iv cholramphenicol
2. iv clindamycin
3. Iv flucloxacillin
4. IV Ceftriaxone
All FOP Recall till June 2021 with note 14 of 378
5. cefuroxime

BOF 31: 3 days old in NICU the nurse forget to give vitamin k
and parents was outside. What to do?
1. Tell the parents and apologize for them
2. Tell the parents to go administrative office
3. Inform the parents and call the consultant
4. Don’t tell the parents and document on file

BOF 32: Vegetarian girl with history of URTI for 3 weeks ago
presented with tiredness and lethargy and Hb 6 wbc 5 plt 60
and low MCV. What is the diagnosis?
1. Vitamin b 12 deficiency
2. Aplastic anemia (100 Hematology)
3. Iron deficiency anemia
4. Foliate deficiency anemia

BOF 33: you are a junior doctor and parents came with their
baby 4 months old boy as the baby cant move the upper limb
on the right side as they said the baby fall from sofa. X-ray
showed spiral fracture of humorous. What to do?
1. Skeletal survey
2. Inform the registrar
3. Discuss with parents suspicion of NAI
4. Call orthopedic registrar

BOF 34: Asthmatic child 4 years on salbutamol. What is best


to be given?
1. Metered dose inhaler
All FOP Recall till June 2021 with note 15 of 378
2. Dried powder
3. Metered dose inhaler with wide volume spacer (100
Respspiratory)
4. Ultrasonic nebulizer

BOF 35: which of the following is present alone will be


indication of CT brain?
1. Bilateral bruises around the eye
2. Amnesia more than 5 minutes
3. Fall from more than 3 meter
4. Vomiting more 3 times
5. LOC more than 5 minutes

BOF 36: 1 year and had viral infection and mother


complained of difficulty of feeding and breathlessness HR
190, RR 45, liver 3 cm below costal margin and bilateral
crepitation and normal air entry. What is the diagnosis?
1. Viral myocarditis
2. Pneumonia
3. CF
4. Bronchiolitis

BOF 37: 8 years with hx of father initially healthy then


developed stroke at 35 years and her uncle died
prematurely at 37 year. Mother was worried about the girl
and ask your advice?
1. Lipid profil now and through pruberty
2. Lipid profile and discharged if normal
3. Wait until girl became competent
All FOP Recall till June 2021 with note 16 of 378
4. Reassure and discharge

BOF 38: 12 years old girl was referred from school for
obesity, her weight and height at 98 centile and BMI at 91
centile and tanner B3 P3. What further management?
1. Diet and life style management
2. Weight management
3. Lipid profile
4. Reassurance
5. Refer to dietitian

BOF 39: Child during playing rugby collapsed and regain his
consciousness immediately and after that referred to the
hospital and at the hospital was full conscious and
examination was normal. Which investigation will you
perform?
1. ECG
2. EEG
3. Echo

BOF 40: child with sudden pain and was with testicular
swelling , tenderness, nausea and absent cremastric reflex.
What is the diagnosis
1. Testicular tortion
2. Hydrocele
3. Tortion of testicular appendix
4. Testicular tumor
1. Hematocele

All FOP Recall till June 2021 with note 17 of 378


BOF 41: Baby with hx of ventous extraction and obstructed
labor with abnormal shape of skull and posing of right frontal
and left posterior occipital. What is the diagnosis?
1. Plagiocephaly
2. Cephalhematoma
3. Craniosynostosis
4. Richets

BOF 42: patient with CF and developed staphylococcal


infection with sudden breathlessness and decrease entry on
right side and shifting of trachea to left side. What is the
diagnosis?
1. Pneumothorax (100 Respiratory)
2. Empyema
3. Lung collapse

BOF 43: Baby with eczema on the face and parents was
applying hydrocortisone cream due to severe eczema and
they increased the potency from emollients and
hydrocortisone to clobetazone during acute exacerbation.
Mother was worried of using potent corticosteroid on face
and causing skin atrophy. What is will be your management?
1. Tacrollimus 0.3 % (100 Derma)
2. Increase frequency of steroids
3. Using of momentasone

BOF 44: 10 years old boy with plantar warts engaging in


competition for swimming and nurse call you for advice.
1. Apply waterproof plaster
2. Don’t allow swimming
All FOP Recall till June 2021 with note 18 of 378
3. Refer to dermatologist

BOF 45: Asian girl with facial rash and hematuria and joint
pain urinalysis showed protein 3+ RBC 2+. What the
investigation lead to diagnosis?
1. ANA
2. Anti double stranded DNA
3. ANCA
4. ESR + FBC

BOF 46: Boy 8 years with gross hematuria after 2 days of


URTI and he said that he has previous attack of previous
illness and he was ashamed to tell the parent and there is
flank pain
U/A hematuria with no RBCS cast. What is the diagnosis?
1. IGA nephropathy
2. Nephrolithiasis
3. Pscgn
4. E-coli

BOF 47: patient presented with previous history of


anaphylaxis to penicillin now presented with tonsillitis. What
to give?
1. Clarithromycin
2. Amoxicillin
3. Co amociclav
4. Clindamycin

All FOP Recall till June 2021 with note 19 of 378


BOF 48: 10 year boy came with cough and fever 38.8
diagnosed with respiratory tract infection not responded to
antibiotics and developed painful nodular lesion on chin.
What diagnosis?
1. Atypical mycobacterium
2. Mycoplasma pneumonia
3. Rheumatic fever

BOF 49: baby 9 months old presented with drowsiness gcs 6


mother said that he rolled over the sofa while his older 3
years brother was watching him. What you will do next?
1. CT scan brain
2. Skeletal survey
3. Cranial u/s
4. Clotting profile
5. Blood culture

BOF 50: 2 years old child 12 kg what will be the dose of


adrenalin if presented with anaphylaxis in hospital
1. 150 mcg IM 1:1000
2. 150 mcg IM 1:10000
3. 300 mcg IM 1:1000
4. 120 mcg IV 1 : 1000

BOF 51: 8 weeks baby , formula feeding, mother reported


that he vomits half of his feeding. O/E; he was well
hydrated, alert. Soft abdomen, wt on 50th centile. What is
your advice?
1. Advice about positioning before and after feeding.

All FOP Recall till June 2021 with note 20 of 378


2. Change to Soya formula.
3. Use of sodium alginate
4. Add thickener
5. Change to follow on formula

BOF 52: 11year JIA on S/C methotrexate weekly developed


small eruption of classic chicken pox eruption. What is best
step in management?

1. Hospital admission of administration iv acyclovir


2. Oral acyclovir for 5 days and then come for follow up
3. Reassure
4. VZIG
5. Oseltamiv oral for 5 days.

BOF 53: Baby with fever and vomiting , when to refer the
baby to the hospital?
1. Poor feeding
2. Drowsy
3. Tachycardia
4. Vomiting

BOF 54: When significant harm occur to pt. , human factor


commonly identified as contributing to this harm, in relation
to human factor in health care. Which one of the following
best summarize our understanding? FOP Case
1. Rudeness of the relatives shouldn’t be tolerated
2. Automation decrease the risk of humanerrors
3. Civilid attitude between colleagues increasethe e
All FOP Recall till June 2021 with note 21 of 378
ectiveness todeal withpregnancy ?!
4. Human errors can't be avoided by anyway

BOF 55: 12 weeks baby not following his mother. What is


the best action to do?
1. Refer to ophthalmologist
2. Reassure
3. Refer to orthoptis
4. Specialized pediatrician

BOF 56: Baby on breast feeding developed manifestation of


eczema and blood streaks in stool. What is the management?
Same like sample paper
1. Advice the mother to exclude dairy products from her
meals
2. Extensive hydrolyzed formula
3. Lactose formula
4. Soya formula

BOF 57: Child diagnosed with life limiting condition. Feeding


by NGT, he is under the care of multidisciplinary team. The
parents in the meeting the parent asked if it is suitable to
transfer him to hospice what would be the appropriate
action?
1. Refer to holistic care
2. Advice to make advanced care plan
3. It is not appropriate to do as this time as he under
multidisciplinary team.
4. Only after assessment of his condition.
All FOP Recall till June 2021 with note 22 of 378
BOF 58: 6 year asymptomatic, C/O pain in lower leg,
vitamin D level is 17. What is the management?
1. Reassurance
2. Calcium
3. High dose of vitamin D 6000 IU for 4 weeks
4. Advice of daily vit D supplm. & increase dietary calcium

BOF 59: 8 year girl came with greenish discharge on her


pants. Her mother refused to bb examined. Mother work as
sex worker. Recently the child has changed to be aggressive
behavior at school. She had several times wetting at school ,
become withdrawal from her family and like to stay alone in
her room. What is the diagnosis?
1. Non consensual contact with adult. (100 safeguard)
2. Non consensual contact with child.
3. Consensual sex

BOF 60: A girl admitted to the hospital because of sever


weight loss, she known to have anorexia nervosa. For
dealing her management. What is he least concerning to her
case?
1. Bradycardia
2. Frequent cutting of her forearm
3. Like image to be low BMI
4. Hypophosphatemia
5. Hypothermia (100 Adolescent & psychiatry)

All FOP Recall till June 2021 with note 23 of 378


BOF 61: A girl was delivered at 27 weeks , she had prolong
ventilation during neonatal period , now she cruise around
furniture, also she has tip toe walking. What is the diagnosis?
1. Spastic diplegia
2. Duchene M.D
3. DDH

BOF 62: Toddler asked to buy something from supermarket


but his mother refused, then the child started to be with
excessive crying and become cyanosis then seizing. What is
the diagnosis?
1. Pseudo seizure
2. Vasovagal syncope.
3. Anoxic reflex (100 Neuro & Neurodevelop)
4. Complex partial

BOF 63: A boy came from Bangladesh to UK one year back


presented with history of HSM & jaundice , no symptoms in
his
country. What investigation you will do?
1. Hb. Electrophoresis
2. CBC with blood film (100 Haemt , infection )
3. Abd U.S

BOF 64: Neonate birth weight was 3.4kg currently3.7 kg,


has frequent vomiting, his feeding is 120 ml every 6hours.
What is your diagnosis?
1. Overfeeding (100 Neonate , Nutrition)
All FOP Recall till June 2021 with note 24 of 378
2. GOR

BOF 65: pregnant women delivered deaf baby. Which is the


most common cause of neonatal deafness?
1. Toxoplasmosis
2. CMV (100 Neonate & Infection)
3. HSV

BOF 66: A girl has a genetic disease, her elder brother has
same disease but younger sister don’t have, her father don’t
have the disease but her mother is affected with the same
disease. All her aunts have the disease but the uncle do not.
Her grandfather has the disease but grandmother do not.
What is the mode of inheritance?
1. XLD (100)?!
2. XLR
3. AD (100)?!
4. AR
5. Multi factorial

BOF 67: Child after diagnosis of brain stem death,


confirmed by 2 seniors, many trials to convince the
parents that ventilation must be stop. Family refused.
Further discussion failed. Who will take decisions of
separation from ventilation?
1. Judge
2. hospital ethics committee
3. Local authority
All FOP Recall till June 2021 with note 25 of 378
4. Biological parents

BOF 68: Newborn has heavy meconium stain with Hx of


PROM at first stage of labour. Mother delivered with prolong
labour for 36 hours, baby cried immediately then beome
vigorous. What to do next?
1. Observation
2. Admission and start antibiotic
3. ETT suction and ventilation.
4. ETT suction and observation
5. Oropharyngeal suction and AB.

BOF 69: 8 year CP with learning difficulty, increase night


time awake & crying with bilateral hip dislocation, with Hx
of epilepsy, constipation, gastrostomy feeding. Parents gave
him regular CPT with no benefit. No home nurse. What to
give?
1. Fentanyl patch
2. Ibuprofen (100 Palliative )
3. Codeine
4. Morphine sulphate
5. Buprenorphine patch

BOF 70: 5 month baby full term sleep with parents in there
room. Mother and father are heavy smoker, takes 25
cigarette per day. Which of the following will increase SIDS
in this baby?
1. Bottle feeding

All FOP Recall till June 2021 with note 26 of 378


2. Preterm ?!
3. Smoking
4. Being 5 months

BOF 71: 14 year girl had sex with her boyfriend 15 year old
last time was unprotected sex want OCP, she doesn’t want to
inform her mother. What ot do?
1. Prescribe emergency OCP
2. Refer to social worker.
3. Advice her to Inform parents before giving OCP .
4. Talk to parent before prescribing OCP

BOF 72: 3 weeks female delivered premature, what the


diagnostic tests for DDH?
1. X- ray of hip
2. US of hip
3. Barlow test & Ortolani test

BOF 73: When to transfer a cystic fibrosis patient to adult


service?
1. Aware to take decision
2. Understand his role of shared decision making.
3. He know his medications and illness.
4. Know to Communicate with adult

BOF 74: A child came for assessment of his growth, birth


weight was normal , now decrease from 50th to 2nd centile.
What will you do?

All FOP Recall till June 2021 with note 27 of 378


1. Ask for the previous chart
2. Do Lab investigation
3. Refer to nutrition unit

BOF 75: A child has hit car while riding bicycle, the hit was
strong enough to let the boy lean forward and hit by bicycle
bar when he arrived to the hospital, he was fully conscious
and lucid & talked with doctors and nurses, examination was
free except for bruises on the forehead and upper abdomen
after 10 hours he started vomiting and his heart rate
increases, , CRT 2sec he still conscious and no focal
neurological signs. What is the cause of his deterioration?
1. Extra Dural hematoma
2. Pancreatic injury
3. Liver injury.
4. Kidney injury

BOF 76: In UK according to NIPE criteria. What is indication


of DDH screening at 4-6 weeks?
1. The drug is unlicensed
2. Explain to the family the expected side e ects
3. Write all information in the baby's notes.
4. There is no licensed drug used instead of this drug?!
5. Need to monitor K &electrolytes

BOF 77: Baby with scenario of HF with large VSD on


furosemide and spironolactone, durine round a 5th year
medical student asking why used unlicensed drugs. How do
you answering to the medical student?

All FOP Recall till June 2021 with note 28 of 378


1. The drug is unlicensed
2. Explain to the family the expected side e etc
3. Write all information in the baby's notes.
4. There is no licensed drug used instead of this drug?!
5. Need to monitor K &electrolytes

BOF 78: 15 days baby on breastfeeding, now become


dehydrated. Weight was 3.2 kg at delivery, now become
2.7 kg. Mother said he missed some feeds serum
sodium150. What will you give?
1. 100 ml/ kg/ day NGT or cup
2. 150 ml/ kg / day NGT or cup
3. 10 ml/kg/ day iv 10 % dextrose
4. 20 ml / kg / day IV 10 % dextrose

BOF 79: 2 year old child with bow leg, irritable during
crawling. Lab; low Ca=1.9 (N=2.1-2.6), low PH=1
(N=0.8-1.5), low vitamin D=17 (N=23-50), low PTH=6
(N=11-24), high ALP=800. What you will do next?
B.Give vitaminD and diatery supplement
C.Give 25 hydroxy vitaminD
D.Give calcium supplements.
E.Give phosphorus supplements

‫وبالتوفيق للجميع‬
‫و;تنسونا من صالح دعائكم‬
All FOP Recall till June 2021 with note 29 of 378
EMQ1: PALLIATIVE: no one got 100 in palliative, please revise it
again

Options: S/C morphine with syringe pump –


Movicol – Senna- Hyoscine patch – increase dose
of slow release morphine - decrease dose of
morphine.
If regional pain fentanyl after paracetamol

A. Pt. with brain tumor, headache, controlled on slow


release morphine BID and PRN morphine, but
developed difficulty of swallowing (dysphagia)
Fentanyl puch —>
what to add? Hyoscine patch ‫ﻣﺎﺣدش ﺟﺎب ﻣﯾﺔ‬ hyoscine

B. Pt. with neuroblastoma , his pain Controlled with


slow release morphine BID and PRN morphine , he
didn't open his bowel for 3 days with soiling. what
to give next? Movicol
C. Patient with metastatic osteosarcoma on slow
release morphine BID and PRN morphine, on
reviewing him by palliative care team they found
he received more than 8 doses per day PRN
morphine, but pain is still not controlled. what to
give next? subcutaneous Morphin With continue release
Or syringe pump

All FOP Recall till June 2021 with note 30 of 378


EMQ2: HEMATOLOGY: 100%

Options: Spherocytosis , SCD, IDA, Transient


erythrocytopenia of childhood, Thalassemia minor,
Thalassemia major , acquired Aplastic Anemia ,
hemolysis due to G6PD. Isolated breast feed
* first vitamin d
* spherocytosis; splenomegaly
* G6PD ; no splenomegaly
* second ida
A. Caucasian 11 mns baby, fully breast feeding,
mother want to introduced solid food at 12
months, he is pale, no other neonatal problem. IDA
B. Caucasian 10 yrs boy , on examination he is pale,
jaundiced, spleen palpable 2 cm below costal
margin , HB 97 , PLT 140, WBC within normal,
Retics 11%. There is a Hx of travelling to Grace, he
is ttted in neonatal period with phototherapy for 2
days. Heriditary Spherocytosis
C. 2 yrs Indian girl, presented with Hx of URTI. On
examination there is solitary small cervical lymph
node, pallor "microcytic hypochromic", Retics &
Ferittin
DDx
Normal Thalassemia minor 100
‫ــــــــــــــــــــــــــــــــــــــــــــــــــــــ‬
* IDA ; low ferritin ‫ــــــــــــــــــــــــــــــــــــــــــــــــــــــــــــــــــــــــــــــــــــــ****** ــــــــــــــــــــــــــــــــ‬
* Thalassemia trait ; normal ferritin
* Thalassemia major ; has severe anemia +obvious symptoms

EMQ 3. HEADACHE: 100% (Neurology)

Options: brain stem glioma , Migraine, tension


Headache, labyrinthitis

A. Female entering secondary school presented with


increasing Morning headache & vomiting and
ataxia. Brain stem glioma

All FOP Recall till June 2021 with note 31 of 378


B. Female entering secondary school, presented with
Headache associated with abdominal pain and
vomiting. Migrain
C. Female entering secondary school presented with
diffuse banding headache with no vomiting. Tension Headach
‫ــــــــــــــــــــــــــــــــــــــــــــــــــــــــــــــــــــــــــــــــــــــ****** ــــــــــــــــــــــــــــــــ ــــــــــــــــــــــــــــــــــــــــــــــــــــــ‬

EMQ4: PUBERITY: 100% Endocrine

Options: Normal puberty, Premature adrenarche,


idiopathic central precious Puberty ,CAH,
Klinefelter $, PW$, adrenal tumor, McCune
Albright $, Craniopharyngioma

A. 11 yrs Girl presented with menarche, she started


with breast and pubic hair before 2 yrs ago. HT on
If in boy;
91 centile. Normal Puberty * precocious puberty —> 80 % tumor
* premature adrenarch

B. 6 yrs girl came with scanty, slightly pigmented


Sweet odor pubic hair & acne. premature adrenarch
C. 8 yrs Girl, came with menarche, breast & pubic
hair, tuner stages B3 P3 Idiopathic central pp
‫ـــــــــــــــــــــــــــــــــــــــــــــــــــــ ـــــــــــــــــــــــــــــــــ****** ــــــــــــــــــــــــــــــــــــــــــــــــــــــــــــــــــــــــــــــــــــــ‬

EMQ5 : CARDIOPULMONERY: Respiratory

Options: acute bronchiolitis , pneumonia , Cystic


Fibrosis, hyperventilation, VSD, PDA, B. Asthma,
TB, NHL

A. 14 year old girl came with difficult breathing, she


said she have pins and needles sensations in her

All FOP Recall till June 2021 with note 32 of 378


Hyperventilation —> washed CO2
NHL ; abnormal CXR —> alkalosis —> hypocalcemia

hands . Chest examination is clear, Spo2 97%,


RR 45 , PR 120 , no Hx of note. Hyperventilation
B. 18 mns baby Boy from Syria, malnourished ,came
with cough for 2 months & Night sweating, he
Sometimes
came with his grandfather and CXR was normal. TB normal CXR
C. 2 yrs FTT, WT on 0.4 centile, came with respiratory
distress, on examination there was bilateral basal
crepitations & crackles. CF ???
VSD come in early age With this severe symptoms + late age if VSD
—> liable to develop PHN and reverse shunt
‫ــــــــــــــــــــــــــــــــــــــــــــــــــــــــــــــــــــــــــــــــــــــ****** ــــــــــــــــــــــــــــــــ ــــــــــــــــــــــــــــــــــــــــــــــــــــــ‬

EMQ 6: LIST of investigation:100% heamatology, GIT

Option: Peripheral blood film, IgA Anti TTG,


lead level, abd. X- ray, serum iron & TIBC.

A. 4 yrs presented with abdominal pain & distention ,


IgA anti ttg
found to be anemic (microcytic hypochromic).
B. baby 2.5 yrs old, presented with Hx of PICA, also
her sister with PICA, they live in old house,
renovation , HB 97, MCV & MCH in normal ranges ,
ferritin Normal. Lead level Ttt by EDTA & d penicillaine Diamonds
DMSA
capron

C. Baby came with bloody diarrhea after GE,


HUS
investigations show HB low, WBC high, Urea
* serum creatinine
* blood film
Normal upper limit & High creatinine. peripheral blood film
* blood culture
HUS

All FOP Recall till June 2021 with note 33 of 378


MSK Q1. 12 yrs old boy had clumsiness and difficult of
jumping in gym class, Now there is difficulty of walking
& climbing stairs. What is the diagnosis?
1.Becker dystrophy > 10 year

2.Duchene muscular dystrophy.


3. Polymyositis. Risk factor associated with lupus ,
Biopsy or MRI Steroid
Rheumatoid arthritis
‫ـــــــــــــــــــــــــــــــــــــــــــــــــــــــــــــــــــــــــــــــــــــــــــــــــــــــــــــــــــ ـــــــــــــــــــــــــــــــــــــــــــــــــــــــــــــــــــــــــــــــــــــ‬

Q2. Case of 2 yrs female child , she is quite and


withdrawn and reluctant to peer weight, she had bruises
Localize on
soft tissue
& petechiae on calf muscle . On examination she is thin,
crying when moving her left leg. There is no
organomegaly (same like S. paper)
Should precede by URTI , fever , petechiae all over , mainly
1.ITP prominent on bone

2.Septic arthritis One joint hip or knee

3.HSP Extensor and buttocks

4.Leukemia
Search in scenario —> support
5. NAI mechanism of NAI
‫ــــــــــــــــــــــــــــــــــــــــــــــــــــــــــــــــــــــــــــــــــــــــــــــــــــــــــــــــــــــــــــ ــــــــــــــــــــــــــــــــــــــــــــــــــــــــــــــــــــــــــــ‬

Q3. 11y Muslim Girl from Pakistan, she is fasting in


Ramadan , she wants to be included with her family
activity. Developed fainting at school during fasting and
banding her head. On hospital her RBS 3.5 mmol/dl,

All FOP Recall till June 2021 with note 34 of 378


WT on 2nd centile. Parents are 1st degree cousin,
she have 2 well other siblings. What you will advise the
family? ‫ﻣﻣﻛن اﺟﺎﺑﺗﯾن ﺻﺢ‬
1. She must stop fasting.
2. Her low blood sugar need more investigations.
Not much low Hb
3. Speak to social and safe guarding, as this is against
best interest to the child.
4. Fasting can cause fainting and if she developed
further episodes, parents should to reconsider the
decision of her fasting ? contraversial
5. Refer to Dietitian to investigate her low body weight.
‫ـــــــــــــــــــــــــــــــــــــــــــــــــــــــــــــــــــــــــــــــــــــــــــــــــــــــــــــــــــــــــــــ ـــــــــــــــــــــــــــــــــــــــــــــــــــــــــــــــــــــــــــ‬

Q4. 24month girl took first step independently at 23


month, she Was born at 30 wk. what is the cause? Female and preterm
1.DDH. 100 neuro, MSK, neurodisability
2.DMD.
3.NAI.
4. Spastic diplegia.
‫ـــــــــــــــــــــــــــــــــــــــــــــــــــــــــــــــــــــــــــــــــــــــــــــــــــــــــــــــــــــــــــــ ـــــــــــــــــــــــــــــــــــــــــــــــــــــــــــــــــــــــــــ‬

Q5. 2 yrs Child, chocking at restaurant while eating


biscuit , mother shout for help, you found the baby
drowsy with weak cough. what you do 1st?
1.Give 5 blow to back then 5 consecutive abdominal
thrusts. 100 emergency
2.Give 5 blow to back then 5 consecutive chest thrust.
* all age 5 blow back
See illustrated new edition * below 1 year —> chest
* above 1 year —> abdomin
All FOP Recall till June 2021 with note 35 of 378
3. 5 rescue breaths.
4. Encourage cough.
5. Make manual removing.
‫ـــــــــــــــــــــــــــــــــــــــــــــــــــــــــــــــــــــــــــــــــــــــــــــــــــــــــــــــــــــــــــــ ـــــــــــــــــــــــــــــــــــــــــــــــــــــــــــــــــــــــــــ‬

Q6. 3 yrs old girl, her mother separated, she noticed there
is blood in her underwire in 2 occasions after she visited
her father in weekend, Mather said the baby had
constipation and scratching. she playing with her bicycle
for long time. what to do as trainee doctor?
1.Discuss with experienced pediatrician help. 100
safeguarding,safety
2.Informe social service * if Working day —> senior
* if weekends —> admission
3.Interview her personally.
4.Detailed genital examination. Examination in present of
consultant and social worker

5. Arrange solotape for thread warms


‫ـــــــــــــــــــــــــــــــــــــــــــــــــــــــــــــــــــــــــــــــــــــــــــــــــــ ـــــــــــــــــــــــــــــــــــــــــــــــــــــــــــــــــــــــــــــــــــــــــــــــــــــ‬
Conjunctivitis
Q7. Baby presented with Hx of GE & URTI, after that he
developed tachypnea, HR 190 with gallop rhythm, CRT
4 sec and weak peripheral pulses & hepatomegaly. Symptoms of HF
what is the1st the causative organism? Triad tachycardia,
tachypnea ,
hepatomegaly
1. Adenovirus 100 cardiology

2. Influenza virus Viral myocarditis;


adenovirus & coxaki b
3. RSV
4. Salmonella
5. Strep. coccus Group A

All FOP Recall till June 2021 with note 36 of 378


(COXSACKIE VIROUS not in the options)
‫ـــــــــــــــــــــــــــــــــــــــــــــــــــــــــــــــــــــــــــــــــــــــــــــــــــــــــــــــــــــــــــــ ـــــــــــــــــــــــــــــــــــــــــــــــــــــــــــــــــــــــــــ‬

Q8. 4yrs old girl presented with severe abdominal pain&


fever, mother noticed blood in her stool.
O/E :Child looks well ,Her family works in dairy
products. what is causative organism?
1.E-COLI O157. ? no one got 100 immunity and allergy,infection

2.Rota virus.
3.Campylobacter jejune. ? Appendicitis like symptoms, same in old
sample paper, most common in UK, with
bloody diarrhea
4.Norovirus Outbreak

5.Giardia. Campaign, rare in UK

Q9. A case of girl 3 yrs old, C/O soreness down below,


her grandmother thinks that her external genitalia doesn’t Soreness,
strogen,
looks like her other cousins. Her mother didn’t notice illustrated
any abnormalities before .O/E: there is labial fusion near
urethral opening . What will you do? No complications
1.Topical emollient
SOP

2. Surgical referral The natural history of labial


adhesions is that they resolve
spontaneously and they can be
3.Topical antifungal managed conservatively. Surgical
separation or therapy with
oestrogen creams have been
tried, but in prepubertal girls the
4.Topical estrogen. 100 Endocrine Reassuring lack of circulating oestrogens
means that they often reform on
stopping the topical oestrogen.
Reassurance and patient watchful
5.Kerotyping waiting is usually successful

‫ـــــــــــــــــــــــــــــــــــــــــــــــــــــــــــــــــــــــــــــــــــــــــــــــــــــــــــــــــــــــــــــ ـــــــــــــــــــــــــــــــــــــــــــــــــــــــــــــــــــــــــــ‬

Q10. 15 yrs old boy, not ongoing to school, excessive


sleeping since 8 pm, not coming out of room for

All FOP Recall till June 2021 with note 37 of 378


breakfast until mid day and not playing video games
either. He was diagnosednwith ASD (autistic spec.
disorder) at age of 9 yrs as he is keen with math. What is
the diagnosis? ASD may associate either with ADHD or
depression

1.Depression. 100 psychatric


2.Onset of puberty.
3. Hypothyroidism.
4. Drug ???
Q11. 15 yrs girl came with fever, fatigability & joint
pain. O/E: she was pale , heart murmur 1/6. BP 145/95,
maculopapular rash.
Investigations :CBC shows anemia and thrombocytosis,
WBCs 19000, [Neutr: 2.5% Lymph: 3.7%,], urine doesn't
shows macroscopic hematuria but show RBCs++ ,
protein +++ and cast. What is your diagnosis? Skin rash + HTN => SLE
1.SLE 100 nephrology, MSK

2.Hemolytic Anemia.
3. Systemic JIA.
4. IgA nephropathy.
5. Reactive arthritis.
‫ـــــــــــــــــــــــــــــــــــــــــــــــــــــــــــــــــــــــــــــــــــــــــــــــــــــــــــــــــــــــــــــ ـــــــــــــــــــــــــــــــــــــــــــــــــــــــــــــــــــــــــــ‬

Q12. 14 year old female with tiredness. CBC show:


MCV low MCHC low HB low. What is the diagnosis?
1.Thlaesmia minor
2.Iron deficiency anemia. 100 heamatology

All FOP Recall till June 2021 with note 38 of 378


3. Folate deficiency.
4. B12 deficiency.
‫ـــــــــــــــــــــــــــــــــــــــــــــــــــــــــــــــــــــــــــــــــــــــــــــــــــــــــــــــــــــــــــــ ـــــــــــــــــــــــــــــــــــــــــــــــــــــــــــــــــــــــــــ‬

Q13. [Picture] of newborn showed bilateral cataract ask


what is the cause? ‫ﺳؤال ﺧﻼﻓﻲ‬
1.Infent of Diabetic mother. ‫اﺧﺗﻼف ﻋﺎﻻﺟﺎﺑﺔ‬

2.CMV. No cataract

3.Retinoblastoma. Not present at birth

4.Galactosemia.
‫ـــــــــــــــــــــــــــــــــــــــــــــــــــــــــــــــــــــــــــــــــــــــــــــــــــــــــــــــــــــــــــــ ـــــــــــــــــــــــــــــــــــــــــــــــــــــــــــــــــــــــــــ‬

Q14. Neonate 3 wks, boy, came with fever, non bilious


E coli
vomiting . Urine culture shows coliform bacteria. He
received N.S bolus , full maintenance and IV antibiotic,
he improved and well perfused but still febrile. Next day
still decrease UOP .
NA 128 , K 5.3 , Urea 7, Creatin 150. what will you do?
1. Add/ change to Gentamycin.
Atypical

2. Strict fluid intake. After lab works —> do imagine

3. Urgent US for kidney and bladder 100 neonate, nephro


4. Give furosemide
5. Give Albumin 4.5
‫ـــــــــــــــــــــــــــــــــــــــــــــــــــــــــــــــــــــــــــــــــــــــــــــــــــــــــــــــــــــــــــــ ـــــــــــــــــــــــــــــــــــــــــــــــــــــــــــــــــــــــــــ‬

Q15. What is the Cause of Macrocephaly?


- hydrocephalus
- high ICP - familial AD , AR with mental retardation
1.Cogenital Rubella - chronic subdural hematoma
- tumor
- genetic Down ......
- TORCH
- neurofibromatosis - fetal alcohol
- polysaccharidosis - radiation, metabolic hypoglycemia
- Soto
- fragile x
All FOP Recall till June 2021 with note - rickets 39 of 378
‫‪2.Maternal Phenylketonuria‬‬
‫‪3.Fetal alcohol syndrome‬‬ ‫‪Microchephaly‬‬

‫‪4.Neurofibromatosis 1 100 neuro, neurodevelopment‬‬


‫‪5. Trisomy 21‬‬
‫ـــــــــــــــــــــــــــــــــــــــــــــــــــــــــــــــــــــــــــــــــــــــــــــــــــــــــــــــــــــــــــــ ـــــــــــــــــــــــــــــــــــــــــــــــــــــــــــــــــــــــــــ‬

‫‪Q16.Infent had paronychia in toes , took topical‬‬


‫‪Antibiotic not improved , then developed fever, macular‬‬
‫‪rash with peeling and crusting around nose & eye no‬‬
‫‪mucosa membrane affection , sterile culture. What is the‬‬
‫?‪diagnosis‬‬
‫‪1.SSSS‬‬ ‫‪100 dermatology‬‬

‫‪2.Steven Johnson syndrome‬‬


‫‪3. TEN‬‬
‫‪4. Epidermolysis bullosa.‬‬
‫‪5. HSV.‬‬ ‫‪High fever‬‬

‫ـــــــــــــــــــــــــــــــــــــــــــــــــــــــــــــــــــــــــــــــــــــــــــــــــــــــــــــــــــــــــــــ ـــــــــــــــــــــــــــــــــــــــــــــــــــــــــــــــــــــــــــ‬

‫‪Q17. Which is of the following ionizing radiation‬‬


‫?‪increase the risk of Cancer‬‬
‫‪1. Abdomen x ray‬‬
‫‪2. CXR‬‬
‫‪3. 1ST trimester of pregnancy x ray 100 oncology, heamatology‬‬
‫‪4. Skull & spine x ray‬‬
‫?? ‪5.barrium contrast‬‬
‫ـــــــــــــــــــــــــــــــــــــــــــــــــــــــــــــــــــــــــــــــــــــــــــــــــــــــــــــــــــــــــــــ ـــــــــــــــــــــــــــــــــــــــــــــــــــــــــــــــــــــــــــ‬

‫‪All FOP Recall till June 2021 with note‬‬ ‫‪40 of 378‬‬
Q18. What is side effects of IV Salbutamol?
1.Lactic acidosis. 100 pharma
- Hyperglycemia
2.Metabolic alkalosis. - hypercalcemia
* -- hypokalemia
* hypophodphatemia
3. Hyper K. * - hypotension
- lactic acidosis
- arrhythmia
4. Hypoglycemia.
‫ـــــــــــــــــــــــــــــــــــــــــــــــــــــــــــــــــــــــــــــــــــــــــــــــــــــــــــــــــــــــــــــ ـــــــــــــــــــــــــــــــــــــــــــــــــــــــــــــــــــــــــــ‬

Q19. A case of 14 yrs boy his WT 68 kg, go for glucose


tolerance test as per insurance, found to have fasting
blood sugar 6.5 and post prandial was 8.9 . What’s your
diagnosis ? Normal person Impaired glucose tolerance Diabetes
* fasting blood sugar = 5.6 * fasting blood sugar = 6.5 * fasting blood sugar > 7
* post prandial = 7.8 * post prandial = 8.9 * post prandial > 11
1.Type 2 DM
2.Impaired glucose tolerance. 100 endocrine, diabetes
3.MODY
4.NORMAL glucose homeostasis.
5. type 1 DM
‫ـــــــــــــــــــــــــــــــــــــــــــــــــــــــــــــــــــــــــــــــــــــــــــــــــــــــــــــــــــــــــــــ ـــــــــــــــــــــــــــــــــــــــــــــــــــــــــــــــــــــــــــ‬

Q20. 7 mns baby brought for developmental assessment,


his father was LF handed. What's need further
investigation? Hand preference at one year

1.LF Hand prefer. 100 neurodevelopment


2.can't recognize strangers.

All FOP Recall till June 2021 with note 41 of 378


Q21 Neonate 6 hrs boy in post natal ward, during NIPE
midwife check oxygen saturation and it was 73 %.
what is the diagnosis?
1.VSD
2.TGA 100 cardio, neonate
3. PDA
4.ASD
5. Co-A of Aorta
‫ـــــــــــــــــــــــــــــــــــــــــــــــــــــــــــــــــــــــــــــــــــــــــــــــــــــــــــــــــــــــــــــ ـــــــــــــــــــــــــــــــــــــــــــــــــــــــــــــــــــــــــــ‬

Q22. Mother asking you what is the benefits of breast


feeding to baby what is the most strongest researched?
1. Decrease incidence of obesity.
2.Decrease risk of breast cancer of the mother.
3.Increase intelligence.
4.Reduce risk of infection. 100 nutrition Especially GE & Otitis media

5. Decrease the risk of Eczema.


‫ـــــــــــــــــــــ ـــــــــــــــــــــــــــــــــــــــــــــــــــــــــــــــــــــــــــــــــــــــــــــــــــــــــــــــــــــــــــــ ــــــــــــــــــــــــــــــــــــــــــــــــــــــ‬

Q23. Child with TSH 180, And there’s No uptake by


isotope scan. What is the diagnosis?
1.Thyroid Aplasia . 100 endocrine
2.Dyshormonogenisis
3. Sublingual thyroid.
Q24. Girl with extensive progressive Oligo JIA, not
controlled by NSAID. The rheumatologist prescribed

All FOP Recall till June 2021 with note 42 of 378


methotrexate, they found on internet that it’s used for
malignancy .What will you tell them?
100 ethics,MSK
1. Ask Rheumatology group support information advice.
2. It is acceptable to take unlicensed drug and as it
prescribed by consultant.
3. Methotrexate should be stopped.
4.Defer Methotrexate & arrange for urgent
rheumatology appointment.
5. start Methotrexate incrementally and monitor FBC
‫ـــــــــــــــــــــــــــــــــــــــــــــــــــــــــــــــــــــــــــــــــــــــــــــــــــــــــــــــــــــــــــــ ـــــــــــــــــــــــــــــــــــــــــــــــــــــــــــــــــــــــــــ‬

Q25. In post natal ward you are doing NIPE examination


for a baby boy. Mother want to do circumcision for her
baby for religious causes , she want referral as soon as
possible. She is married but separated from father who is
refusing . What is the best accurate to tell her?
A- Father must consent in joined with her. 100 ethics
B- GP must refer the child for surgery.
C- Must postpone till the baby is competent.
D- Surgery should be delayed after nappy off for the fear
of infection.
E- discuss with social services
‫ـــــ ـــــــــــــــــــــــــــــــــــــــــــــــــــــــــــــــــــــــــــــــــــــــــــــــــــــــــــــــــــــــــــــ ــــــــــــــــــــــــــــــــــــــــــــــــــــــــــــــــــــــ‬

Q26. 4 or 5 mns presented with continues crying &


diarrhea after feeding, there is eczema, he is on regular
full term formula what to give? (Same like S. paper)

All FOP Recall till June 2021 with note 43 of 378


1.Trial of extensive hydrolyzed formula 100 nutrition
2.Stop the full term formula
‫ـــــــــــــــــــــــــــــــــــــــــــــــــــــــــــــــــــــــــــــــــــ ـــــــــــــــــــــــــــــــــــــــــــــــــــــــــــــــــــــــــــــــــــــــــــــــــــــــــــــــــــــ‬

Q27. Baby on child protection plan, come with bruises


and epistaxis, purpuric rash on trunk and back. He had
viral illnesses before 3 wk . What is the diagnosis? (same
like S. paper)
1.ITP.
2.NAI.
‫ـــــــــــــــــــــــــــــــــــــــــــــــــــــــــــــــــــــــــــــــــــــــــــــــــــــــــــــــــــــــــــــ ـــــــــــــــــــــــــــــــــــــــــــــــــــــــــــــــــــــــــــ‬

Q28. Where is the Site of BCG VACCINATION?


1.INTRADERMAL Left arm at site of deltoid insertion.
2. INTRADERMAL left arm between deltoid insertion &
shoulder.
3.S/C Left arm at deltoid insertion.
4. INTRADERMAL Right arm at site of deltoid insertion.
5. S/C Right arm at deltoid insertion
Q29. 9 yrs Child with Down syndrome, WT on 9th
centile, HT in 0.4 centile on down chart, no regular
f/up before. He is well and playing football.
Examination was normal apart from features of down $.
what is the best investigation will you do for sh. Stature?
(same like sample paper)
1.TFT 100 genetic, endocrine.
2.IGf 1

All FOP Recall till June 2021 with note 44 of 378


3.Cortisol
4. Celiac disease.
5. RBS
‫ـــــــــــــــــــــــــــــــــــــــــــــــــــــــــــــــــــــــــــــــــــــــــــــــــــــــــــــــــــــــــــــ ـــــــــــــــــــــــــــــــــــــــــــــــــــــــــــــــــــــــــــ‬

Q30. 2 yrs girl has 6 perianal warts ,the mother ttted with
wart in pregnancy. What to tell the mother?
1.HPV is not needed. The only exception not to
screen for chlamydia if
mother or member of
2.Chylmidia must be screened .100 Dermatology family is has the wart,
But here in scenario ,
mother treated , so we
3.Spontaneously resolved. have exclude chlamydia ,
may be she took infection
from others
4. most likely it is due to sexual abuse.
‫ـــــــــــــــــــــــــــــــــــــــــــــــــــــــــــــــــــــــــــــــــــــــ ـــــــــــــــــــــــــــــــــــــــــــــــــــــــــــــــــــــــــــــــــــــــــــــــــــــــــــــــــ‬
Micropenis
Q31. Baby with small phallus & bilateral impalpable
gonads , no dysmorphic features , feeding after 12 hrs,
parents are not relatives, no FH of chronic disease what
is the diagnosis?
1.CAH 100 endocrine
2.Klienfelter $
3.Prader willi $
4.hypospadius
5. Bechwith-wiedemann $
‫ـــــــــــــــــــــــــــــــــــــــــــــــــــــــــــــــــــــــــــــــــــــــــــــــــــــــــــــــــــــــــــــــــــــــ ـــــــــــــــــــــــــــــــــــــــــــــــــــــــــــــــــ‬

Q32. Which is the most Highest risk Factor of neonatal


infection?
1.Preterm before 37 wks with spontaneous delivery.

All FOP Recall till June 2021 with note 45 of 378


2.Intrapartum fever more than 38.5.
3. Suspected infection in multiple pregnancy. 100 neonate
4. Maternal Colonization of GBS and bacteria in current
pregnancy.
ABC + 3 S

5. GBS Infection in previous pregnancy


‫ـــــــــــــــــــــــــــــــــــــــــــــــــــــــــــــــــــــــــــــــــــــــــــــــــــــــــــــــــــــــــــــ ـــــــــــــــــــــــــــــــــــــــــــــــــــــــــــــــــــــــــــ‬

Q33. Teenager girl was champion, she was previously


well good achiever in gymnastic before participated
school activity, now C/O: myalgia, arthralgia &
fatigability . There is Hx of viral infection before 4
months ago, she missed some school days, the family are
worried about her condition and they need her to win
before next compition. What is her Mx?
1. Graded exercise, CBT (cognitive behavioral therapy) ,
home tuition. If it has learning disability
2. Graded exercise, CBT, adjustable school time table.
100
3. Graded exercise, CBT, stimulant medication. adolescence,b
ehaviour,psych
4. Family therapy, CBT , home tuition. atric
5. CBT, M.Vit, adjustable school time table
‫ـــــــــــــــــــــــــــــــــــــــــــــــــــــــــــــــــــــــــــــــــــــــــــــــــــــــــــــــــــــــــــــ ـــــــــــــــــــــــــــــــــــــــــــــــــــــــــــــــــــــــــــ‬

Q34. 15 yrs old female presented with vomiting, severe


abdominal pain 8/10 with tenderness in right and lower
abdominal pain, no guarding, she had irregular cycle ,
last period was 6 weeks ago and today she bleed,
BP 80/50, no fever. what you will do?
1.Pregnancy test. Need consent

All FOP Recall till June 2021 with note 46 of 378


First initial thing to do
2.Urine analysis. 100 adolescent, nephro
3.Abd & pelvic US.
4.FBC .
‫ـــــــــــــــــــــــــــــــــــــــــــــــــــــــــــــــــــــــــــــــــــــــــــــــــــــــــــــــــــــــــــــ ـــــــــــــــــــــــــــــــــــــــــــــــــــــــــــــــــــــــــــ‬

Q35. Female diagnosed as seasonal sinusitis and she has


exam after 3 days. What is your advice?
1.Oral steroids for 48 hrs.
2.Oral Cetirizine starting this evening for 3 days. 100
respiratory,ENT,
3. Beclomethasone Topical / nasal spray. pharma

4. Xylometazoline nasal spray as needed during the


exam. Induce sleep
5. Oral Chlorpheniramine for 3 days
‫ـــــــــــــــــــــــــــــــــــــــــــــــــــــــــــــــــــــــــــــــــــــــــــــــــــــــــــــــــــــــــــــ ـــــــــــــــــــــــــــــــــــــــــــــــــــــــــــــــــــــــــــ‬

Q36. 4months came to pediatric clinic with Squint, the


squint was apparent during examination, mother also had
squint when she was child. What to do?
1.Refer to ophthalmology. 100 ophthalma, neuro
2.Review at 6 month.
3. Do MRI / CT.
‫ـــــــــــــــــــــــــــــــــــــــــــــــــــــــــــــــــــــــــــــــــــــــــــــــــــــــــــــــــــــــــــــ ـــــــــــــــــــــــــــــــــــــــــــــــــــــــــــــــــــــــــــ‬

Q37.18mns baby, came with Fever for 4 days, given


Amoxicillin yesterday, the fever subsided and the baby
become well, but developed maculopapular rash on the
trunk and back today. What the diagnosis? (same like S.
paper)

All FOP Recall till June 2021 with note 47 of 378


‫‪1.Roseola infantum.‬‬
‫‪2. Measles.‬‬
‫‪3. Rubella.‬‬
‫‪4. Rx reaction.‬‬
‫ـــــــــــــــــــــــــــــــــــــــــــــــــــــــــــــــــــــــــــــــــــــــــــــــــــــــــــــــــــــــــــــ ـــــــــــــــــــــــــــــــــــــــــــــــــــــــــــــــــــــــــــ‬

‫‪Q38. 12 year old boy with polyuria , polydipsia, RBS 34‬‬


‫‪and blood ketone 4 (Normal :0.6 – 1 mmol/l) .‬‬
‫? ‪What to do at time of Dx‬‬
‫‪1.Blood gas. 100 endocrine,diabetes‬‬
‫‪2.ISLET CELL ANTIBODIES‬‬
‫‪3.ANI GAD ANTIBODIES‬‬
‫‪4.C PEPTIDE‬‬
‫‪5. Urine Ketone‬‬
‫ـــــــــــــــــــــــــــــــــــــــــــــــــــــــــــــــــــــــــــــــــــــــــــــــــــــــــــــــــــــــــــــ ـــــــــــــــــــــــــــــــــــــــــــــــــــــــــــــــــــــــــــ‬

‫‪Q39. 6 yrs old girl with short stature, WT on 9th centile,‬‬


‫‪HT on 2nd centile with mild learning difficulties, mid‬‬
‫? ‪parental HT on 50th centile. which investigation to do‬‬
‫‪1.Karyotyping 100 genetic‬‬
‫‪2. TFT‬‬
‫‪3. IgA TTG‬‬
‫‪4. IGF – 1‬‬
‫ـــــــــــــــــــــــــــــــــــــــــــــــــــــــــــــــــــــــــــــــــــــــــــــــــــــــــــــــــــــــــــــ ـــــــــــــــــــــــــــــــــــــــــــــــــــــــــــــــــــــــــــ‬

‫‪Q40. 6mns Somalia Infant crying at night O/E: the baby‬‬


‫‪is well apart from Umbilical hernia which is easily‬‬

‫‪All FOP Recall till June 2021 with note‬‬ ‫‪48 of 378‬‬
reducible ( 1 cm length and 0.5 cm defect). what will you
advice regarding the umbilical hernia?
1. Wait for natural closure 100 GIT
2.Come back to emergency if vomit.
3.Reffer for surgery.
4. reduce by coin.
‫ـــــــــــــــــــــــــــــــــــــــــــــــــــــــــــــــــــــــــــــــــــــــــــــــــــــــــــــــــــــــــــــ ـــــــــــــــــــــــــــــــــــــــــــــــــــــــــــــــــــــــــــ‬

Q41. 15 yrs Girl came to you after she took 16 tablets of


paracetamol at (24hr , 12hr & 2 hrs) before she came, she
had skin rash when treated for previous paracetamol
toxicity. What will you do?
100 emergency
1. Infuse N-acetylcysteine immediately over 60 mins.
2 .Measure level of paracetamol and given N-acetyl-
cysteine over 4 hrs.
3. Take level of paracetamol after 4 hrs.
4.Take investigation and wait for result .
5.Do not give NAC due to previous sensitivity.
‫ـــــــــــــــــــــــــــــــــــــــــــــــــــــــــــــــــــــــــــــــــــــــــــــــــــــــــــــــــــــــــــــ ـــــــــــــــــــــــــــــــــــــــــــــــــــــــــــــــــــــــــــ‬

Q42. Patient with ALL on chemotherapy maintenance


and has no history of chicken pox before with sero-
negative immunity. His brother developed vesicular rash
today. The family asking over the phone for advice?
contraversial please revise the new
1.Prescribe acyclovir now.? guidlines
PEP after 7 days of exposure
2.VZIG and Acyclovir when rash appears
3. Give Vaccine & VZIG together.

All FOP Recall till June 2021 with note 49 of 378


4. Reassure the family that the rash will be self limiting
& nothing to be done.
Q43. 3 days NB, discharged at 6 hrs after birth, came
with bilious vomiting twice at home. O/E: baby was
normal, What to do now?
1.admit and investigate 100 neonate
2.No need for investigation Either duodenal atresia or
sepsis
3.Tell midwife to follow up .
4.Urgent pediatric outpatient appointment
5. Ask the mother to take sample of vomits and bring it
with her.
‫ـــــــــــــــــــــــــــــــــــــــــــــــــــــــــــــــــــــــــــــــــــــــــــــــــــــــــــــــــــــــــــــ ـــــــــــــــــــــــــــــــــــــــــــــــــــــــــــــــــــــــــــ‬

Q44. 7 years old boy on 200 micro gm fluticasone and


salbutamol but not controlled. What you will add next?
1.LTRA 100 respiratory

2.LABA
3. Increases steroid dose.
4. Continue on the same dose.
‫ـــــــــــــــــــــــــــــــــــــــــــــــــــــــــــــــــــــــــــــــــــــــــــــــــــــــــــــــــــــــــــــ ـــــــــــــــــــــــــــــــــــــــــــــــــــــــــــــــــــــــــــ‬

Q45. 7years with glassy trunk rash with depressed centre,


his friends don't want to play with him. What will do?
[Molluscum Contagiosum ]
1.Reffer for Cryotherapy 100 derma If on the body

2.Topical silver nitrate If on the face

All FOP Recall till June 2021 with note 50 of 378


3.Benzyl peroxide.
4. No need ttt.
‫ـــــــــــــــــــــــــــــــــــــــــــــــــــــــــــــــــــــــــــــــــ ـــــــــــــــــــــــــــــــــــــــــــــــــــــــــــــــــــــــــــــــــــــــــــــــــــــــــــــــــــــــ‬

Q46. 2 yr old girl get pale and appear lifeless and floppy
after minor injury, the child is subdued for sometime
after the attack. What is the Dx?
1. Reflux anoxic seizure. 100 neuro
2. Long QT
3. Epilepsy.
4. Anxiety attack
‫ـــــــــــــــــــــــــــــــــــــــــــــــــــــــــــــــــــــــــــــــــــــــــــــــــــــــــــــــــــــــــــــ ـــــــــــــــــــــــــــــــــــــــــــــــــــــــــــــــــــــــــــ‬

Q47. 7 years old child complaining of 3months period of


cough and wheeze. He received multiple antibiotics with
little response. He can't keep up with his friends in the
playground. He has eczema when he was infant and his
mum has hay fever . What is the investigation that will
lead to a definitive diagnosis?
1 - Spirometry. 100 respiratory For diagnosis of BA

2 - Immunological study.
3- Skin prick test.
4-Chest x- ray
5 - Viral serology

Q48. 28 wks PT, came with loud murmur, difficult to


wean from ventilation, what is the ttt you will give ?

All FOP Recall till June 2021 with note 51 of 378


1. Spironolactone.
2. Furosemide. Give if with HF

3. Ibuprofen. 100 neonate,pharma


4. PG
‫ـــــــــــــــــــــــــــــــــــــــــــــــــــــــــــــــــــــــــــــــــــــــــــــــــــــــــــــــــــــــــــــ ـــــــــــــــــــــــــــــــــــــــــــــــــــــــــــــــــــــــــــ‬

Q49. 11 yrs boy, came after playing football with sever


LF scrotal pain. What is the Dx?
1. Hematocele.
2. Testicular torsion. 100 urology
3. Torsion of appendix testis.
4. Epididymo-orchitis
5. torsion of hydatid cyst of Morgagni.
‫ـــــــــــــــــــــــــــــــــــــــــــــــــــــــــــــــــــــــــــــــــــــــــــــــــــــــــــــــــــــــــــــ ـــــــــــــــــــــــــــــــــــــــــــــــــــــــــــــــــــــــــــ‬

Q50. 6 yrs old boy came with limping & Internal rotation
with restricted movement (abduction). What is the Dx?
(same like S. paper).
1. Perth’s dis. sample paper
2. SUFE.
3. Reactive arthritis.
4. Septic arthritis.
Q51. 3 yrs Baby, came with fever & inability to bear
WT. O/E: the knee was hot , tender. What is the Ix
confirm the Dx?
1. X- Ray.

All FOP Recall till June 2021 with note 52 of 378


2. FBC & ESR.
3. Joint aspirate 100 MSK
‫ـــــــــــــــــــــــــــــــــــــــــــــــــــــــــــــــــــــــــــــــــــــــــــــــــــــــــــــــــــــــــــــ ـــــــــــــــــــــــــــــــــــــــــــــــــــــــــــــــــــــــــــ‬

Q52. Which indicate NAI from the following?


1. Bruises in ear in 4mns old baby. 100 safeguarding
2. Bruises in the back in 5mns baby.
3. Bruises in forehead in 3yrs baby.
4. Quadriplegic CP with thigh bruises in 12 yrs.
5. Shin bruises in 2 yrs old.
‫ـــــــــــــــــــــــــــــــــــــــــــــــــــــــــــــــــــــــــــــــــــــــــــــــــــــــــــــــــــــــــــــ ـــــــــــــــــــــــــــــــــــــــــــــــــــــــــــــــــــــــــــ‬

Q53. 2 yrs Baby in foster care, came toxic with high


fever 39 , intercostal & subcostal recession, soft stridor
and drooling what to do ? [Epiglottis.]
1.Neublized Normal saline.
2.IM adrenaline .
3.IV Cefotaxime.
4. Urgent ENT referral 100 ENT, emergency
5. X-ray Neck
Q54. School age child, he was in school trip, which
involve lot of activities like clamping mountains and
cannoning. The trip needed to be cut short due to floods
& … rain. He came with malaise, joint pain, intermittent
fever, rigor & night sweat for 14 days. O/E: Jaundice,
HSM, conjunctival injection, macular rash in trunk.

All FOP Recall till June 2021 with note 53 of 378


There is a Hx with Travel to Malta 1yr ago. what is your
diagnosis?
1. leptospirosis. ? no one got 100 in infection

2. Typhoid Fever.
3.Lyme disease.
4. Hep A.
5. Malaria
‫ـــــــــــــــــــــــــــــــــــــــــــــــــــــــــــــــــــــــــــــــــــــــــــــــــــــــــــــــــــــــــــــ ـــــــــــــــــــــــــــــــــــــــــــــــــــــــــــــــــــــــــــ‬

Q55. Female with morbid obesity BMI 99.6 centile,


no FH of DM or cardiac disease. What is the Mx?
1. Dietary Mx.
2. Give Orlistat.
3. Refer for local WT Management program. 100
endocrine,diabetes
4. check lipid profile.
5. Screen for Dm.
‫ـــــــــــــــــــــــــــــــــــــــــــــــــــــــــــــــــــــــــــــــــــــــــــــــــــــــــــــــــــــــــــــ ـــــــــــــــــــــــــــــــــــــــــــــــــــــــــــــــــــــــــــ‬

Q56. Scenario of child playing by bicycle, hit by a car


and car didn't stop, presented with brasses in elbow &
knees, he was couscous and well ,observed for 6 hrs &
discharged home. What is the most potential
complications that can occur?
1. Extradural hemorrhage. 100 emergency
Lucent interval then manifest after
2. Subarachnoid hemorrhage. compression on the artery
3. Intra ventricular hemorrhage.
4. Necrotizing fasciitis

All FOP Recall till June 2021 with note 54 of 378


‫ـــــــــــــــــــــــــــــــــــــــــــــــــــــــــــــــــــــــــــــــــــــــــــــــــــــــــــــــــــــــــــــ ـــــــــــــــــــــــــــــــــــــــــــــــــــــــــــــــــــــــــــ‬

‫‪Q57. 6wks NB, presented with Progressive vomiting.‬‬


‫?‪what is most investigation indicative of Pyloric stenosis‬‬
‫‪1.Hypo K‬‬
‫‪2.Hypo CL 100 GIT‬‬
‫‪3.Hyper NA‬‬ ‫‪Hypochloremic‬‬
‫‪Hypokalemic‬‬
‫‪Metabolic alkalosis‬‬
‫‪4. high urea‬‬
‫‪5. Metabolic acidosis‬‬
‫ـــــــــــــــــــــــــــــــــــــــــــــــــــــــــــــــــــــــــــــــــــــــــــــــــــــــــــــــــــــــــــــ ـــــــــــــــــــــــــــــــــــــــــــــــــــــــــــــــــــــــــــ‬

‫‪Q58.2 yrs child had loose stool since infancy. what is the‬‬
‫?‪finding indicate further investigation‬‬
‫‪1.Recurrent chest infection. 100 respiratory‬‬ ‫‪Cystic fibrosis‬‬

‫‪2.Wt & HT on 2nd centile .‬‬


‫‪3. Fecal incontinence .‬‬
‫‪4. Eczema.‬‬
‫ـــــــــــــــــــــــــــــــــــــــــــــــــــــــــــــــــــــــــــــــــــــــــــــــــــــــــــــــــــــــــــــ ـــــــــــــــــــــــــــــــــــــــــــــــــــــــــــــــــــــــــــ‬

‫‪Q59. known case of type1 Dm, admitted for control of‬‬


‫‪her blood control, at 12:05 pm she get pale, sweating‬‬
‫?‪and shaking, her RBS 3.5mmol/ l. what you give next‬‬
‫‪1-Iv dextrose‬‬
‫‪2-Oral glucose 15 gm 100 diabetes‬‬
‫‪3- Adjustment the time of the lunch‬‬
‫‪4. Orange juice and bread toast.‬‬
‫‪5. IM glucagon.‬‬

‫‪All FOP Recall till June 2021 with note‬‬ ‫‪55 of 378‬‬
‫ـــــــــــــــــــــــــــــــــــــــــــــــــــــــــــــــــــــــــــــــــــــــــــــــــــــــــــــــــــــــــــــ ـــــــــــــــــــــــــــــــــــــــــــــــــــــــــــــــــــــــــــ‬

Q60. 2 yrs, case of febrile convulsions, Midazolam given


by paramedic , IV lorazepam prepaid to be given, child
is still seizing, no signs of focal infection. What is next to
do?
1.Check glucose 100 neuro, emergency
2.LP
3. check serum Calcium.
4. Blood culture.
5. CT brain

Q61. Boy developed meningiococcal meningitis, his


sister 14 yrs on OCP.
What is Prophylaxis for meningitis?
1. single dose ciprofloxacin. 100 pharma
2. Ceftriaxone.
3. Rifampcin.
‫ـــــــــــــــــــــــــــــــــــــــــــــــــــــــــــــــــــــــــــــــــــــــــــــــــــــــــــــــــــــــــــــ ـــــــــــــــــــــــــــــــــــــــــــــــــــــــــــــــــــــــــــ‬

Q62. 7 yrs old, presented with polyurea, polydipsia & Wt


loss. What Ix you will do?
1. C peptide.
2. Islet cell A.bodies.
3.Blood glucose. 100 endocrine
4. HBA1C

All FOP Recall till June 2021 with note 56 of 378


5. Specific gravity
‫ـــــــــــــــــــــــــــــــــــــــــــــــــــــــــــــــــــــــــــــــــــــــــــــــــــــــــــــــــــــــــــــــــــــــــــــ ـــــــــــــــــــــــــــــــــــــــــــــــــــــــــــ‬

Q63. 2 yrs child coming for elective inguinal hernia


repair after premedication, the child developed
convulsion, Past medical Hx irrelevant apart from Hx of
afebrile convulsion at 9mns of age. What is the possible
Dx?
1.MCAAD 100 neuro, genetic
dt developed hypoglycemia

2. Idiopathic epilepsy.
3. Febrile convulsion
4. B6 deficiency
‫ـــــــــــــــــــــــــــــــــــــــــــــــــــــــــــــــــــــــــــــــــــــــــــــــــــــــــــــــــــــــــــــ ــــــــــــــــــــــــــــــ ـــــــــــــــــــــــــــــــــــــــــــــ‬

Q64. 2 wks NB, referred by midwife because of


jaundice, T. Serum Bilirubin 215, direct 92, GGT "little
high". What is Dx? > 20 %

1.Neonatal hepatitis.
2. Breast milk Jaundice.
3. Physiological Jaundice.
4. Bellary Artesia 100 GIT
5. Hemolytic dis. Of newborn
‫ـــــــــــــــــــــــــــــــــــــــــــــــــــــــــــــــــــــــــــــــــــــــــــــــــــــــــــــــــــــــــــــ ـــــــــــــــــــــــــــــــــــــــــــــــــــــــــــــــــــــــــــ‬

Q65. 6 yrs old boy, presented with joint pain in ankle &
knee with abdominal pain, purpuric rash in LL . no
macroscopic hematuria, urine dipstick showed
proteinuria + , RBC +, what is Dx?

All FOP Recall till June 2021 with note 57 of 378


1. HSP. 100 nephro, MSK
2. Meningococcal septicemia.
3. IgA nephropathy.
‫ـــــــــــــــــــــــــ ـــــــــــــــــــــــــــــــــــــــــــــــــــــــــــــــــــــــــــــــــــــــــــــــــــــــــــــــــــــــــــــ ــــــــــــــــــــــــــــــــــــــــــــــــــ‬

Q66. 14 yrs female had fatigability, fever 2wks duration ,


night sweating, Hx of pricing . O/E: diastolic murmur,
splenomegaly & finger clubbing. Urine analysis show
micro hematuria What is Dx?
1. IEC. 100 cardio
2. RH. Fever .
3. ALL.
4. NHL
‫ـــــــــــــــــــــــــــــــــــــــــــــــــــــــــــــــــــــــــــــــــــــــــــــــــــــــــــــــــــــــــــــ ـــــــــــــــــــــــــــــــــــــــــــــــــــــــــــــــــــــــــــ‬

Q67: 12 yrs female, had day & night symptoms, her


night symptoms improved with pelvic floor exercise, but
still she had dripping of urine with laughing & coughing.
What will you give?
* < 5 year —> reassuring
1. Desmopressin. * > 5 year motivation then
alarming
2. Alarm .
3. Amitriptyline. If only night time
Tricyclic antidepressant
4. Oxybutynin. 100 nephro, pharma Daytime dripping, severe
symptoms
‫ـــــــــــــــــــــــــــــــــــــــــــــــــــــــــــــــــــــــــــــــــــــــــــــــــــــــــــــــــــــــــــــ ـــــــــــــــــــــــــــــــــــــــــــــــــــــــــــــــــــــــــــ‬

Q68. 18mns, fully immunized, toxic appearance, Temp


40 , came with purpuric rash on LL, CRT 4 sec , anterior
fontanelle is flat. What is the Dx?

All FOP Recall till June 2021 with note 58 of 378


1. Meningococcal septicemia.
2. HSP.
3. ITP.
4. NAI.
‫ـــــــــــــــــــــــــــــــــــــــــــــــــــــــــــــــــــــــــــــــــــــــــــــــــــــــــــــــــــــــــــــ ـــــــــــــــــــــــــــــــــــــــــــــــــــــــــــــــــــــــــــ‬

Q69. 4 wks diagnosed as unstable hip, referred for


orthopedic surgery, mother asked about Mx?
1. Abduction splint. 100 MSK * < 3 mo. —> by pediatric , double diaper
—> by surgery , abduction splint
* 3-6 mo. Pavlik harness
2. Adduction splint. * 6 mo. - 2 yr —> close reduction
* > 2 yr —> open reduction
* > 3 yr —> femoral shortening
3. Open reduction. * > 4 yr —> acetabulum surgery

4. Acetabuloplasty.
5. Close reduction.
‫ــــــــــــــــــــــــــــــــــــــــــــــــــــــــــــــــــــــــــــــــــــــــــــــــــــــــــــــــ ــــــــــــــــــــــــــــــــــــــــــــــــــــــــــــــــــــــــــــــــــــــــ‬

Q70. Child presented with generalized body swelling


more prominent in LL, he had Hx of skin infection , what
is Ix you will do?
ASGN
1. Serum Albumin * 6 weeks after skin infection
* 2 weeks after sore throat
2. Urine analysis. 100 nephro
3. Urea & Electrolytes.
4. Complement assay
‫ـــــــــــــــــــــــــــــــــــــــــــــــــــــــــــــــــــــــــــــــــــــــــــــــــــــــــــــــــــــــــــــ ـــــــــــــــــــــــــــــــــــــــــــــــــــــــــــــــــــــــــــ‬

Q71. Mother known case of SLE, had Hx of recurrent


miscarriages, what is the most serious complication
could affect the baby?
SLE anti ro anti la
* anti ro —> complete heart block
* anti la —> skin rash

All FOP Recall till June 2021 with note 59 of 378


1. Complete heart block. 100 MSK,cardio
2. 1st degree heart block.
3. Carditis.
4. Thrombocytopenia.
‫ـــــــــــــــــــــــــــــــــــــــــــــ ـــــــــــــــــــــــــــــــــــــــــــــــــــــــــــــــــــــــــــــــــــــــــــــــــــــــــــــــــــــــــــــ ــــــــــــــــــــــــــــــ‬

Q72. 9 yrs argue with her mother after step – father came
to home, then she went for her bed , when she stand up
got pale , fall to ground, twitching of her hands, rapidly
recovered, she remember she was dizzy before, there is +
ve Hx of Epilepsy?
1. Complex partial.
2. Syncope. 100 neuro
3. Pseudo seizure.
4. Vertigo . * syncope ; After triggering —> syncope —> then
abnormal movements
* complex partial ; after automatism aura —>
5. Reflex anoxic seizing —> then loss of consciousness

‫ـــــــــــــــــــــــــــــــــــــــــــــــــــــــــــــــــــــــــــــــــــــــــــــــــــــــــــــــــــــــــــــ ـــــــــــــــــــــــــــــــــــــــــــــــــــــــــــــــــــــــــــ‬

Q73. Female teenager, her mother think that she induce


vomiting, but she is denying. What is the evidence
support the mother thinking? Bulemia

1. Multiple cuts in the wrist.


2. Erosion of teeth enamel. 100 adolescent, psychatric

Q74. Child with Hx of fever then after 5 days, urine


analysis show microscopic hematuria, proteinuria++ &
?acetone .What is the most likely Dx.?

All FOP Recall till June 2021 with note 60 of 378


1. IgA nephropathy. 100 nephro
2. Glumerlonephritis.
3. Renal failure.
‫ـــــــــــــــــــــــــــــــــــــــــــــــــــــــــــــــــــــــــــــــــــــــــــــــــــــــــــــــــــــــــــــ ـــــــــــــــــــــــــــــــــــــــــــــــــــــــــــــــــــــــــــ‬

Q75. 7 yrs old boy C/O headache. O/E: pubic hair , testis
3 ml , penis stage 3, high BP, facial acne, adult ouder.
What is the Dx.?
1. CAH.
2. Adrenal tumor. 100 endocrine High BP

3. Premature adrenarche.
4. idiopathic central precious puberty.
‫ــــــــــــــــــــــــــــــــــــــــــــــــــــــــــــــــــــــــــــــــــــــــ ــــــــــــــــــــــــــــــــــــــــــــــــــــــــــــــــــــــــــــــــــــــــــــــــــــــــــــــــ‬

Q76: Nurse give double dose of Gentamicin when she


was preparing in a side room, she was interrupted by
phone call, After trough level done, they found it is high.
What you will do?
1. Don't inform the parents unless harm has occurred.
2. Inform parents& tell them investigations are going on.100 patient
safety
3. Inform parents and tell them who is responsible.
4. Don't tell the parent at all.
5. Inform medical team governance.
‫ـــــــــــــــــــــــــــــــــــــــــــــــــــــــــــــــــــــــــــــــــــــــــــــــــــــــــــــــــــــــــــــ ـــــــــــــــــــــــــــــــــــــــــــــــــــــــــــــــــــــــــــ‬

Q77: 2mns old with inconsible crying 2 -3 hrs after


vaccination. What to do?

All FOP Recall till June 2021 with note 61 of 378


‫‪1. Continue as same schedule as planned. 100 patient safety‬‬
‫‪2. Provide the next vaccination under hospital care.‬‬
‫‪3. Delay next vaccination.‬‬
‫ـــــــــــــــــــــــــــــــــــــــــــــــــــــــــــــــــــ ـــــــــــــــــــــــــــــــــــــــــــــــــــــــــــــــــــــــــــــــــــــــــــــــــــــــــــــــــــــــــــــــــــــ‬

‫‪Q78: child with URTI (O.M), torticollis, stridor. What is‬‬


‫?‪the Dx‬‬
‫‪1. Mastoiditis.‬‬
‫‪2. Retro pharyngeal abscess. 100 ENT, resp‬‬
‫‪3. Sternomastoid tumor .‬‬
‫‪4. Cervical lymphadenopathy.‬‬
‫ـــــــــــــــــــــــــــــــــــــــــــــــــــــــــــــــــــــــــــــــــــــــــــــــــــــــــــــــــــــــــــــ ـــــــــــــــــــــــــــــــــــــــــــــــــــــــــــــــــــــــــــ‬

‫‪Q79: Baby developed bleeding after circumcision, you‬‬


‫‪are in ward found baby with tachycardia, CRT prolonged‬‬
‫?‪. what you will do‬‬
‫‪1. call Surgical team.‬‬
‫‪2. Give normal saline bullos. 100 emergency‬‬
‫‪3. give factor 8 .‬‬
‫‪4. Fresh frozen plasma.‬‬
‫‪5. Give human Albumin 4.5‬‬

‫* كل الشكر والتقدير والعرفان لمن شارك في هذا العمل الجماعي جعله‬


‫هللا في موازين حسناتكم ونسأل هللا عز وجل أن يكون هذا العمل خالصاً‬
‫لوجه الكريم‪.‬‬

‫* ونسألكم الدعاء بالنجاح والتوفيق دائما وأبداً ‪.‬‬


‫قــــــروب فـــــــــــبراير ‪ 2021‬م‬

‫‪All FOP Recall till June 2021 with note‬‬ ‫‪62 of 378‬‬
Fop October 2020

EMQ
I- Hematology Diagnosis
1. Hereditary spherocytosis
2. G6PD / thalassemia major
3. Thalassemia trait
Microcetic hypochromic anemia 1. Thalassemia HSM
4. Hemophilia A 2. IDA. |_ no HSM
5. ABO 3. G6PD |

a. Turkish 18 months old boy FTT with hepatosplenomegaly with Hb =


6 microcytic hypochromic anemia, No antenatal screening Dx.
thalassemia major 100%
Inv; Hemoglobin Electrophoresis

b. Malazian boy with pallor and dark color urine without


hepatosplenomegaly took herbal ttt. Intravascular hemolysis —> dark urine
G6PD 100%
Inv, G6PD assay
c. North Europe 3 years with mild jaundice for 1 day during neonatal
period he had two sisters are normal. Now he presents with pallor
and splenomegaly. May give in scenario —> father has cholecystectomy
Hereditary spherocytosis 100%
Inv; osmotic fragility test
II- Neuro. Diagnosis

1- Benign childhood epilepsy (Rolandic epilepsy)


2- Reflex anoxic seizures
3- Complex partial seizure
4- Infantile spasm
5- Febrile convolution
6- Rigor.
Grunting
a. Child 6 years mother heared abnormal noise in the child room.
She found the child has abnormal twisting movement in one side of
face with abnormal eye movement, drooling saliva ,sweating at night
during sleep and feeling tired and can not speek with mother ,attack
repeated 2 times before.
Benign childhood epilepsy (Rolandic epilepsy) 100%
EEG during sleep —> cenrotemporal spike ,, ttt carbamazepine

All FOP Recall till June 2021 with note 63 of 378


b. Scenario about toddler got pale and loss of consciousness for seconds
after hit his head by table with muscle twitching.
reflex anoxic seizures 100%

c. Scenario about feverish child 39 c with abnormal body movement that


stopped when you touch him.
Rigor 100%

III – Infectious Disease

1-CMV /
2- HIV /
3- HBV
4- Rubella
5- Varicella
6- Parvovirus B19
7- HSV
**Q about maternal infection lead to baby congenital defect.
a. Congenital heart disease.
Rubella 100% Peripheral PS , PDA
b. Dermal scaring and limbs hypoplasia.
Varicella 100% 381 guidelines
c. Q about hydrops fetalis.
Parvovirus B19 100%

IV - List of investigation.

1- Bronchoscopy
2- FBC and film
3- CT chest
4- IgE level
5- IgG subsets
6- esophageal PH study
7- peak expiratory flow reading
8- pulmonary function test
9- short synacthen test
10- US chest . (like s.paper)

a- 2 years old boy has been admitted to hospital 4 times in the last 6
months with Dx of croup, on one occasion he required intubation

All FOP Recall till June 2021 with note 64 of 378


and ventilation. He was born at 26 weeks gestation and was
oxygen dependent until the age of 4 months.
Bronchoscopy 100% Subgluttic stenosis , see SOP

b- 9 years old girl present with a 10 days history of fever and cough.
She had responded initially to oral amoxicillin but now look ill and
has stony dullness on Rt lower chest. Dull chest ?? Effusion
US chest 100%

c- An 8 years old boy is taking 750 microgram/day inhaled


fluticasone propionate. He has poor controlled cough and last
week has become very lethargic after short – lived episode of
vomiting. Other children in his class had similar illness but they
less severity affected. He has finger clubbing, his height on 25th
centile. High cortisone , adrenal insufficiency , see akp case
short synacthen test 100%

V – GIT Investigation

1- Colonoscopy with biopsy


2- upper GIT endoscopy with biopsy
3- anti tissue transglutaminase
4- abdominal US
5- barium swallow
6- abdominal CT.

**All scenarios have same family history of father has peptic


ulcer and other member with crohn disease (I think aunt?).

a. 12 years child with weight loss and diarrhea and Rt.Iliac


Fossa pain and tenderness.
Colonoscopy with biopsy

b. 2 years child with pallor and faltering growth.


anti tissue transglutaminase To differentiate celiac and crhon,
celiac early age

c. 14 years girls with epigastric pain which awaken her from


sleep at night.
upper GIT endoscopy with biopsy

All FOP Recall till June 2021 with note 65 of 378


VI – Palliative
1- Ketamine
2- IV morphine
3- intranasal morphine
4- fentanyl patch
5- no drug ttt.
6- femoral Nerve block
7- Diclofenac
8- Ibuprofen

a. Girl had RTA come with large hematoma over temporal


area with thigh deformity (fracture) and hypotensive and
GCS 13. IV cannula inserted and CT arranged, orthopedic
want to do splint and child sever thigh pain.
femoral Nerve block 100%

b. Scenario about child was cold with burn by hot coffee on


his Lt arm and anterior chest they apply tap water for 20
min and put cling film but the child still distressed and
crying and have pain.
intranasal morphine 100%

c. 13 years old girl was complaining banding headache and


was took regular paracetamol and ibuprofen in maximum
dose for weeks but no effect on pain now.
no drug ttt 100%

BOF
1. 10 years old girl came with mother partner with acute abdominal pain
which Dx as acute appendicitis need operation within 4 hours,
mother outside the city and aware about her condition.
Whom to take the consent?
a. Mother partner.
b. Daughter. 1. Married husband even after divorce
2. After December 2013
c. Surgeon doctor in charge. 3. Before 2013 if his name in the certificate
4. If his name in the certificate —> mother give to him
d. Mother by phone. 100% —> by court order

All FOP Recall till June 2021 with note 66 of 378


2.Child with meningococcal infection what prophylaxis drug do you give to
his sister?
a. Ciprofloxacin. 100%
b. Azithromycin. Meningococcal, cipro
H influnza, rifampicin
c. Amoxicillin. Pneumococcal, no drug
d. No prophylaxis needed.

3. Boy with cystic fibrosis. What is the chance of his sisters to be carrier of
cystic fibrosis?
a. 1/4 . b. 2/3 .100% c. 1/2 . d. No carrier.

4. Mother had varicella 5 days after delivery of full-term baby, the baby
well and good feeding and has no sign of varicella what to do.
a. Check maternal and baby IgG and IgM and tack action according the
result.
b. Give IVIG to baby and give acyclovir if develop rash. 100%
c. Give oral acyclovir.
d. No need for any action.
BMI
5. How to Dx overweight? > 91 % overweight
a. Weight 99.6%. > 98 % obese
> 99.6 % morbid
b. BMI = 99.6% . See guidelines
c. BMI = 91% .100%
d. Waist circumference > 75 % centile.
e. BMI >25

All FOP Recall till June 2021 with note 67 of 378


Hint 4-6 week , should exclude biliary atresia Clay stool
6. 6 weeks old boy from Nigerian parents brought by his mother with poor
feeding and lethargy with jaundice and dark color urine the mother has
changed his feeding to formula milk 2 weeks ago due to poor weight
gain. Biliary atresia dt viral or autoimmune induced , confirmed by US or HIDA test
What initial investigation for Dx?
a. Urine culture.
b. Conjugated bilirubin. 100%
c. Electrophoresis.
d. G6PD level.
e. Mantoue test

7. Girl with tachycardia narrow complex more than 300 b/m which drug
you give?
a. Propranolol.
b. Digoxin.
c. Adenosine. 100%
d. Amiodarone

8. Child with facial swelling, pallor and stridor 1 hour after eating nuts butter
what to give him? Anaphylactic, many 2 systems affected
a. Oral antihistamine.
b. IM adrenaline. 100%
c. No Rx.
d. Hydrocortisone.

9. 14 days old baby with mucopurulent eyes discharge not respond to


chloramphenicol, what the causative organism?
a. Neisseria gonorrhea. 0-5 days , ceftriaxone
b. Chlamydia trachomatis. 100% 5-14 days, single Iv azithromycin , eye drops azithromycin
c. Streptococcus.

10. Child with Hx of RTA (hit by car on 50 km / hr velocity) came with


shock and there were bruises over Lt Upper abdomen. 2 IVcannulas
were inserted and she received NS 0.9% 40ml /kg still CRT = 4 second,
poor peripheral perfusion, tachycardia Surgeon in the way. What to do
next?
a.10 ml / kg packed RBCs
b. 20 ml /kg NS 0.9 % bolus.
c. Human albumin.
d. In tropes through peripheral infusion or inotropes
e. 10 ml/kg normal saline 0.9%.

* I known cardiac disease, deteriorated after IVF by tachycardia —> cardiogenic shock —> inotrope mainly milirinone
* evidence of bleeding, —> hypovolemic shock —> mainly fluid , PRBC not delay
* septic shock , if no evidence of bleed —> inotrope after 40-60ml/ kg
All FOP Recall till June 2021 with note 68 of 378
11. Child came with History of Otitis media and Fever with muffled
respiratory sounds, weak crying and torticollis.
Best next step? Retropharyngeal abcess —> refer to ENT for drainge
a. Oral antibiotic.
b. Nebulized adrenaline.
c. Urgent refer to ENT. 100%
d. X ray lateral view
e. Neck us.

12. 13 years old girl with breast bud and scanty pubic hair. Asking bout
tanner stage?
a. Breast stage 1 pubic stage 1.
b. Breast stage 1 pubic stage 2.
c. Breast stage 2 pubic stage 2. 100%

13. 13 months boy come with head circumference on 98% centile with
normal development no other abnormalities. There were no previous
recording in his Redbook. What is the best action now?
a. Cranial US.
b. Fundus examination.
c. Brain MRI.
d. Fallow up after 3 months.
e. Assess parents head circumference. 100%

14. Question about 12 days old baby girl with 2 days deterioration and
lethargy poor feeding lymphedema in (hands or feet) weak femoral
pulsations murmur 2/6 with normal heart sounds. CRT = 3, on 100% O2
but no oxygen saturation can be recorded. What drug to give?
a. Prostaglandin infusion. 100% Turner’s $ With Coarctation of Aorta
b. IV antibiotic.
c. IV acyclovir.
d. Dopamine. Any neonatal deterioration 1. Sepsis, 2. Cardiac, 3. Metabolic
e. Lasix.

15. Boy 12 years old obese small testis size 2.5 ml, ask about 1st sign
of puberty?
a. Testicular enlargement. 100%
b. Axillary hair.

All FOP Recall till June 2021 with note 69 of 378


c. Penis enlargement.

16. Girl with clinical feature and recent Dx of rheumatic fever, what
investigation to roll out or in the Dx?
a. Blood culture.
b. Throat swab.
c. ASO titer. 100%
d. Echo

17. 10-years old child with primary nocturnal enuresis last year he
took Desmopressin when he went for camp and he was ok on it.
Now the child is very distressed about his condition, behavioral
motivation failed asks about what is next step?
a. Course of Desmopressin.
b. Enuresis alarm.100% * below 5 year, nothing to do
* above 5 year , behavioral motivation, alarm
c. Amitriptyline. * going trip , desmopressin
d. Imipramine. * soiling laxative for constipation
* dropping, urgency , anticholinergic
e. psychiatry

18. 5 years old girl with day and night enuresis and drippling ask
about possible US abnormalities.
a. Duplex kidney. 100%
b. Horseshoe kidney.
c. Rt kidney mass.
d. Dilated renal pelvis

19.15 years old boy with 3rd attempt to suicide now come with cutting
injury in wrist, the boy wants to go home and reluctant to admission but
was calmed down by a nurse he knows from previous admission.
What is the best action for the doctor to do now?
a. Telephone social service.
b. In patient Urgent assessment by psychiatric child and adolescent
service. 100%
c. Permit him to return home.
d. Discharge him and ask GP doctor to fallow him.
e. Combansory admission to pediatric and adult with security guard.

20.8-year-old child with 1st attack of convulsion for 5 minutes now he is


ok. He had Hx of febrile convulsion when he was 3 years, mother ask if he
need anticonvulsant therapy, ask about indication to start AED?

> = 2 unprovoked seizure

All FOP Recall till June 2021 with note 70 of 378


a. Family Hx of epilepsy.
b. Abnormal EEG.
c. Further occurrence of convulsion. 100% Guidelines p 144

d. The AED protect against brain damage


e.Prevent learning disabilities

21. Child with epilepsy on carbamazepine come with community acquired


pneumonia, he had allergy to penicillin, what to give to him?
a. Amoxicillin.
b. Cephalexin.
c. Ciprofloxacin.
d. Erythromycin. enzyme inhibitors
e. Co amoxiclav.

22. Scenario about child with Lt eye swelling and protrusion with hypertension, with
fever ask about Dx ?
a. Orbital cellulitis. b.Neuroblastoma.100% c.Nephrotic syndrome.
Abdominal swelling 80%, bone metastasis, pancytopenia

23. 6Days old baby came with ambiguous genitalia with


hyponatremia and hyperkalemia, Presented with shock, ask about Dx
test?
Cong adrenal hyperplasia
a. 17 hydroxyprogesterone. 100% 1st day —> karyotyping
b. Aldosterone. 2nd day —> electrolytes
3rd day —> 17 hydroxyprogestrone
c. Cortisol.
d. Karyotype

24. Child with eczema ask about the most potent topical steroid? ‫كان كاتب اإلسم العلمي‬
‫والتجاري‬
a. Clobetazol (demovet). 100%
b. Betamethasone.
c. Clobetasone.
d. Hydrocortisone.
e. Mometazone

25.Child referred from GP with chronic constipation with abdominal pain and
fecal impaction up to umbilicus. ask about management?
a. Admit and do dis impaction in hospital.
b. Prescribe laxative and send him home.
c. Oral disimpaction at home with close review.100%

Movicol 7 days —> laxative,senna


Microenema —> macroenema —>
manual disimpaction under Anastasia
All FOP Recall till June 2021 with note 71 of 378
26.13 years old girl with HR 55 she loss 5 kg weight she isolates herself
from family and friends not eating with family she goes every day to
school when doctor ask her she said she is all right and she was quite
during assessment ask about Dx? Anorexia Nervosa as she going school ,
depression not going anywhere
a. Eating disorder.100%
b. Depression.
c. Hyperthyroidism.

27. 3 years old child with repeated abdominal pain and vomiting and
pallor weight on 0.4 centile, with history of no organomegaly normal
abdominal examination with hypertension. What Dx?
a- Wilms tumor. No abdominal pain —>
accidentally discovered
Renal artery stenosis
High blood pressure that is not controlled by
b- renal dysplasia.100% medications and lifestyle changes, including diet and
exercise, may be a symptom of renal artery disease.
d- Coarctation Symptoms of renal artery disease may also include
fluid retention or congestive heart failure. Some
e- Renal artery stenosis cases, renal artery disease entirely asymptomatic
(not associated with any symptoms).In severe
f- Essential HTN cases, renal artery disease may lead to kidney
failure. Symptoms of kidney failure include
weakness, shortness of breath and fatigue

28. 4years old Girl with type 1 DM on basal and bolus insulin regimen
with HBA1C 7.9%. Mother found her in her room with convulsion and
sweating what the cause?
a. Epilepsy.
b. b. Hyperglycemia.
c. c. Hypoglycemia. 100%

29. 12 years old girl was previous healthy. Hx of hurting her Lt shoulder
and she did ear piercing which made infection to skin and responded
to topical antibiotic ttt. She is presented now with fever and murmur
on apex. Ask about investigation?
a. Blood culture. 100%
b. Echocardiography.
c. Urea and electrolyte.

30.ACase of unilateral facial palsy he can wrinkle his forehead, he had


mild response to steroids (Scenario of upper motor neuron lesion).
Ask about next step?
a. Refer to neurology. Upper motor neuron
b. b. CT brain Stem lesion spares upper face

c. c. MRI brain. 100%

All FOP Recall till June 2021 with note 72 of 378


31. Infant with NEC (preterm 29 weeks and was discharged on oral
feeding after period of intubation in NICU then developed abdominal
bloating, lethargy and poor feeding). Ask about 1ST step in Mx.
a. Stop feeding. 100%
b. Surgical consultation.
c. Change to formula milk.
d. I v antibiotics

32. 8 years girl from Sudan come to you for ttt of UTI, she refused to be
examined then she told you that her aunt was cut her from down
(genital mutilation) ask about your action?
a. Call police.100%
- if done already mutilation —> call police
b. Contact social service. - if decisions to do mutilation —> arrange meeting
c. Do nothing.

33. Mother came to you, she is worried about her 24 months old Child with 3
attacks of otitis media with effusion (glue ear) in last 4 months and he finished
3 courses of antibiotics. Mother is worried about hearing problems associated
with glue ear which she heard about. By examination the child has red
tympanic membrane and –ve diffuse light reflex. What is your next step?
a. Give a new course of amoxicillin 125 mg TID for 7 days and analgisa and
review after 3 weeks When to refer with
recurrent OM
Treatment OM , 1. Below 6
month, 2. Bilateral
b. Analgesia on need without review.100%
c. Give decongestant drug and refer to ENT Urgent for grommet
insertion.
d. Ask the nurse to do hearing assessment and if more 30 hz Refer to
ENT.
e. Give Leaflet to family to educate about hearing loss.

34. Baby on breast milk, mother changed to formula milk after starting
work, after then the child developed skin allergy and not gaining weight
what to do? Cow milk protein allergy
a. Continue same formula.
b. Extensive hydrolyzed formula milk. 100%
c. Alginate.
d. Emollients.

35.14 years old girl with hypothyroidism due to hashimoto thyroiditis on


levothyroxine 125mcg. She came to you with high TSH and normal T4,
she said she feels good, what to do?

Any child start thyroid treatment, first to correct , T4

All FOP Recall till June 2021 with note 73 of 378


a. Check T3,T4, TRH
b. Check thyroid AB.
c. Check compliance with her GP. 100%

36. The best way to assess cognitive development of 3 years old child?
a. Draws with crayon.
b. Obeys simple instruction.
c. Points at common objects
d. Brings her shoes from cupboard
e. Puts the car toy under table.

37. Child came with history of recurrent macroscopic hematuria with sore
throat RBCS +3 protein+2 on dipstick then after one week dipstick test
showed trace RBCS and no protein .Ask about which will be abnormal
during infection. IgA nephropathy
IgA nephropathy (Berger disease) is the commonest
a. Serum IgA level. 100% glomerulonephritis in children. Males are affected twice
as commonly as females. Gross haema-turia (and more
b. C3,C4 !arely microscopic haematuria) occurs shortly after an
upper respiratory tract infection (or during) d.
c. Abdominal US post-streptococcal disease where haematuria occurs
generally 10-14 days after infection. There may be
d. Cystoscopy associated loin pain. It is self-limiting. Between 10 and
30% of children will go on to develop renal failure
e. Double stranded DNA

38. Child came with jaundice and hepatitis B ask about best test to
assess the severity of hepatitis?
a. High ALT.
b.ALP.
c. HBe Ag +ve.
d. Prolonged PT.
e. albumin

39. A child came with dehydration and GE came to ER after few days.
What is the best way to assess the degree of dehydration?
a. Skin turgor.
b. Dryness of the mouth.
c. Compare with previous weight.100% *?GIT*
d. Hypotension.
e. HR 120 beat/min

40. Infant with poor feeding, lethargy, cough for 3 days with RR66 /min and
intercostal, subcostal recession with changed just one nappy per day. By = oligourea
examination wheezes and crackles on auscultation, O2 sat 96%. Mother
wanted him to return home within 2 days because of Christmas.

All FOP Recall till June 2021 with note 74 of 378


What the most important next step in Mx? Bronchiolitis

a. IV fluid hydration.
b. Feeding milk via NG tube. 100%
c. Nebulized salbutamol.
d. Oral prednisolone

41. 6 weeks infant boy with unilateral translucent scrotal mass baby Hydrocele
was calm, well and has good feeding, what is yours next action? painless
a. Scrotal US.
b. Refer to pediatric surgeon.
c. Reassure and follow up if present in 1 year. 100%
d. No follow up needed
e. Reassurance and discharge

42. Child returned from India to London and developed wt loss, pallor and
limping for 3 weeks. One knee and ankle joints .bone x- ray showed
osteolytic lesions ask about Dx?
a. Leukemia. 100%
b. Chronic osteomyelitis.
c. TB
d. osteosarcoma
e. Histocytosis

43. 3 years boy with obesity BMI = 99% centile with small penis
(surrounding fat pad) no other abnormalities, parents Ht in 50% patient
May add bone age increase
Ht in 75 % and the mother said he has normal appetite. for one year, simple obesity
The family is concerned about him. What to do to assess his condition?
a. DNA analysis.
b. Diet and activity assessment. 100%
c. Abdominal or scrotal US
d. Glucose tolerance test
e. Thyroid profile.

44. CP Infant with gastrostomy tube feeding came with decreased level of
consciousness with high Na = 190 with normal K and urea and
creatinine ask about the cause?
a. Fabricated or induced illness. 100% Or salt poison
b. Nephrogenic DI. Should add in scenario, polyurea
c. Hyperaldosteronism. Should low K
d. SIADH. Low Na

All FOP Recall till June 2021 with note 75 of 378


e. Gastroentritis

45. Vegetarian mother with exclusive breast feeding of her baby for 1 year,
the child has poor appetite. Which nutrient is deficient? "NO B12"
a. Iron. 100%
b.Calcium.
c. Zinc.
d. Folic acid.

46.3 years old boy came with yellow colored skin and white sclera, mother
is vegetarian and she said that he likes to drink too much squash juice,
what the Dx ?
a. G6pddef.
b. Thalassemia.
c. Carotenemia.100%
d.Hepatitis

47. Child with small VSD needs elective tonsillectomy. The ENT doctor
needs pediatric consultation about prophylactic antibiotics?
a. Give him oral antibiotic.
b. No need for prophylactic antibiotics. 100%
c. Iv antibiotics.
d. Chlorhexidinemouth wash.

48. 15 years girl took 20 tablets of paracetamol when admitted to hospital


she escaped, you called the family and they said she did not come to
home, what is your action?
a. Tell the hospital security to find her.
b. Call the social service.
c. Call the police to find her. 100%
d. Call the pediatric consultant in charge for child protection
Infantile spasn
49. 8 months baby with EEG (chaotic appearance), Knoking of head and
flexion of limbs over the trunk ask about Rx? (scenario of infantile
spasm).
a. Vigabatrin. 100% Ifgive in scenario mother hypertension,so
vigabatrin for tuberous sclerosis
b. Carbamazepine.
c. lamotrigine
(no steroids in options)

All FOP Recall till June 2021 with note 76 of 378


50. 8 weeks infant came with excessive crying, irritability, and refusal of
feeding, bulging anterior fontanel. Mother said he rolled from sofa and fell
on carpet 1day ago. BP = 140/80, HR= 80, RR= 25. What to do?
a. IV saline.
Fell down lead to brain hge then develop
b. Call anesthesia. 100% ICP , Suspected for impending herniation
NAI
c. IV acyclovir.
d. IV broad-spectrum antibiotics.

51. Junior doctor came from Thailand developed diarrhea and vomiting
Just before coming to UK had a duty shift on weekend .
What he should do?
a. Stay home till 48 hours free of symptoms. 100%
b. Do work and keep good hand hygiene (extra precautions).
c. Use PPE (wearing mask and gowns when dealing with patients)
d. Do investigation and take loperamide.
e. Go work when he feels very well.

52. Child with fever for 6 days and red tongue and eyes with cervical
lymph nodes with swelling of the hands.
What to give which will affect the outcome complications?
a. Oral Aspirin.
b. IVIG. 100%
c. IV antibiotics.
d. ibuprofen.

53. 3years old child with temper tantrum, stole chocolate from
supermarket, the relative tilling the mother advise about changing his
behavior, there was a younger sister of 10 months age what to do?
a. Praise him for good behavior.
b. Mother tells him that everybody loves him as much as his
sister.100%
c. Till his father to punch him.
d. Send him to live with grandparents
e. Sind him to his room when he has bad behavior

54. 13 years old girl want to have HPV vaccine but her mother refuses, the
girl read about the vaccine and understand (competent) what to do?
a. Wait until she 18 years.
b. Follow mother’s wish.
c. She can have the vaccine if she is Gillick (Frazer) competent.100%

All FOP Recall till June 2021 with note 77 of 378


55. 20 months old girl her mother said she found her arching her back,
profuse sweating, sea- saw respiration for 2-3 min. then she fell
sleep not repeated in the day, if interrupted she got angry. She had
normal development. Ask about Dx?
a. Reflex anoxic seizer.
b. Sandifer. Should associate with feeding
c. Self-gratification. 100%
d. Temper tantrum.

56. Scenario about child with difficulty in writing clumsy and difficulty in
walking. Refer to occupational therapy Can’t tie his shoes
a. Dyslexia.
b. Developmental coordination disorder (dyspraxia). 100%
c. ADHD
d. Autism

57. 14 years old girl deteriorated in school performance with weight gain
and cannot do well in gymnastic session and felt tired during day. she
had scanty period. Mother thought she has been bullied at school.Dx
a. Hypothyroidism.100% Increases TSH, lead to Increase prolactin, lead inhibition in FSH & LH
b. Cushing. Central Obesity , no affecting on periods
c. OSA. No affect of period
d. Anorexia nervosa. Weight loss

58.Yellow card used for what? Yellow card; penicillin allergy


Red card: wt, ht, head circumference
a. Record Penicillin allergy. 100% Green card; vaccinations
b. Weight of child.
c. Fever after new Vaccine. G I
R
d. Oral thrush after steroids inhalation. Y
M
D
e. Tremor after Salbutamol

59.18 hours old boy with lethargy and poor feeding with inflamed umbilical
skin with 3 blisters on the buttock. Ask about causative organism?
a. Staph epidermis.
b. Group A strep.
c. Staph aureus. 100%
* HSV; skin scar , vesicles
d. Chicken pox * chicken pox ; skin scar , limb abnormality, eye abnormal
*

60.In DDH screening, which test indicates dislocatable rather than

All FOP Recall till June 2021 with note 78 of 378


dislocated hip?
a. Positive Ortolani test.
b. Positive Barlow test. 100%
c. Hip Click.
d. Affected limb is shorter (length asymmetry)
e. Failure of abduction of the affected side.

61. 9 years old boy with cystic fibrosis on ciprofloxacin syrup for treatment of
chest infection, came on Saturday. He refuses taking the suspension
because of its taste what to do?
a. Stop treatment and refer to Cystic fibrosis team.
b. Give IV antibiotic.
c. Ask him to try ciprofloxacin tablet. 100%
d. Give amoxicillin
Should receive
e. Nebulized Tobramycin treatment

62. What warrant you for referral to pediatric surgery?


a. Supra umbilical hernia at 1 year.
b. Undescended testis at 6 weeks. Undescended
undescended
unilateral palpable testis —> review at 3 months, if still
—> review at 6 months, if still undescended —>
C. Until 3 year c. Umbilical hernia. refer to surgery , Either started on trial hormonal therapy if fail go
therapy
unless ,
encarserated d. Non Retracted foreskin at 3 y.
e. Hydrocele at 6 months Refer at age of one year

63. Female patient with generalized maclopapular purpuric rash eruption


for 2 weeks. Rt.knee pain without redness nor swelling, had also pain
in ankle joints in one-week history with normal blood Pressure and
normal lab. work. Asking about dx.
a. HSP 100%
b. systemic onset JIA
c. Reactive arthritis Proceed by URTI or GIT
d. Septic arthritis. Redness , swelling in joint
e. SLE Face rashes

64. Adolescent girl 15 years old came to ER with tachypnea, H.R 80


beat/min difficulty of breath, tingling sensations in hands; O2 sat was
99% what to do initially?
Tachypnea without tachycardia, washed CO2 , low ionized Ca,
a. Rebreathing bag 100% hypocalcemia ( tingling) , best to do rebreather lead to CO2 retention
b. Benzodiazepines improved resp . Alkalosis , but the bag may cause hypoxia

c. O2 Hyperventilation syndrome, best management if frequent occur ,


d. Iv calcium breathing exercises, guided by psychological,,, during attack give
benzodiazepine,,,if chronic, give antidepressant

All FOP Recall till June 2021 with note 79 of 378


65. 2 months old infant came with cough and cyanosis with red dots in the
eyes. With vomiting following coughing . Asking about ttt.
a. Erythromycin 100% Pertussis , red spot, apnea

b. Ampicillin c.

66. 3 years boy with recurrent wheezy chest, cough increases with
exercise and his condition improves with inhaled salbutamol.
Which advice best given to parents?
a. Trial of ICS 100% Treat of chronic asthma

b. Peak flowmetry
c. Skin prick test

67. Child complains of recurrent attacks of macular erythematous rash with


white area appear suddenly and disappear in 24 h with Eyelid swelling
and associated with recurrent RTIs, with history of facial eczema in
infancy and take soya-based formula. Mother has allergy to mite/Pollin.
Diagnosis? By exclusion

a. Chronic non specific urticaria. 100%


b. Erythema multiform Post infection or drug side effects
c. Hereditary Angioedema Has systemic symptoms
d. Ig E syndrome / hypersensitivity

68. 18 months child came with acute (24h) deterioration, vomiting and
In gastroenteritis bloody diarrhea with intermittent abdominal pain and pallor,
stool content
blood, but here investigations:
bloody stool = PH 7.2 BE -10
red current jelly
HB 12 (within normal range given)
WBCs 20 Intususseption,, pain is on & off ,,
Plat 145 treatment by barium enema or surgery

What to do initially?
a. Abdominal us Target sign
b. Abdominal x-ray
c. Urea & Electrolytes
d. Stool C&S
e. Clotting test

69. 3 years old boy came with speech & language delay.his mother
complaining of abnormal behavior (sameness, hand flapping when he Autism
is happy with something he like). Mother is worried about school and
there is no facility for behavioral assessment in your unit, what should
you do? ‫عليه خالف‬

See fop cases,

All FOP Recall till June 2021 with note 80 of 378


a. Refer to community pediatrician for assessment.
b. Refer to educational psychologist for preschool assessment. If associated depression
c. Speech and language therapy referral
d. Refer to health visitor for behavioral assessment.
e. Hearing assessment

70. Which warrant you for urgent referral to hospital?


a. fever 40 C
b. Reduced skin turgor 100% Pallor , skin turger, tachycardia, tachypnea

c. Reduced urine output


d. Dry mucous membrane
e. Ear discharge

71. 7 years old boy complains easy bruising, clumsy walking. The child
doesn’t want to participate in sports. He is presented with bruise over
his knee, he didn’t remember any trauma and by examination there
were multiple bruises over and below his knees. History of delay
walking and father also had history of delayed walking. Diagnosis?
a. Hemophilia A. Father not affected,

b. Ehler Danlos syndrome 100% Autosomal dominant, skin streachable, joint


hypermobility, cardiac aortic dissection, myopathy,
c. Marfan syndrome No bruises platelet aggregation ( bruises )
d. Non accidental injury
e. Won wellbrand disease

72. 4 years old girl came with dysuria with low grade fever and mild
abdominal pain. Urine culture shows E coli responded to antibiotics in
48 hours. What will you do next?
a. reassure and give advice about preventive measures 100%
b. do ultrasound
c. MCUG
d. DMSA

73.A child 8 years with 6 weeks history of waking at night, crying in leg pain.
The child has mild ligament laxity. Examination was normal. What is the
Diagnosis?
a. Growing pain 100% Night pain mainly, symmetric bilateral, associated with benign
hypermobility, ( pallor, HSM, pancytopenia)
b. Restless syndrome
c. ALL
d. EhlerDanlos syndrome

All FOP Recall till June 2021 with note 81 of 378


74. 3 years old girl came to A&E with minimal vulvaler bleeding, she was
playing in the garden with her elder brother who is 15 years old and
witnessed her falling off the swing on her bicycle. What would you do?
a. Examine her under sedation
b. Call senior pediatrician for assessment 100% He is responsible to refer to all other
staff,, first to call senior, then call
c. Refer to gynecologist social workers
d. Skeletal Survey
e. Call child protection

75. A case of congenital hypothyroidism was prescribed levothyroxine in wrong


dose (higher dose). Parents are concerned about side effects.
What is the most 1Stsign of overdose? Sleep disturbance first thing parent will observe at home,,
a. Tachycardia other sign , the Dr will observe

b. Weight loss
c. Diarrhea
d. Hyperthermia
e. Sleep disturbance See SOP

76. A child came with recurrent attacks of URTI with intermittent stridor and
In foreign tracheal tug with mild grade fever. Ask about diagnosis.
body &
croup a. Laryngotracheobronchitis 100% Recurrent croup , twice per year, bronchoscope to
b. Epiglottitis. exclude subglottic stenosis

c. Bronchiolitis
d. FB inhalation

77. Child came at Friday night with mother complain of difficulty of feeding,
Examination show bruises on cheeck, mother is a teacher and father is
accountant. What is next step? ?? NAI, example, humerus spiral fracture, metaphyseal fracture in
femurs, post rib fracture, callus formation > months, bruise cheek bruise in
a. Call social service by phone earpinna, cigarette burn , sign of bite , circumference hand and foot
b. Admission for observation. 100% Friday is holiday , admit then when start working days , to be
seen by social workers
c. Do coagulation profile and review on Monday.
d. Reassurance and discharge home

79. Female Baby 6 weeks, non dysmorphic presented with lethargy and poor
feeding ,during examination there is Lymphedema in LL and systolic murmur
in 2/6 in the Lt. sternal edge. Liver is palpable 1 cm below the costal margin.
What is diagnosis? Turner Syndrome
a. AVSD
b.VSD
c. PDA
d. CoA 100%

All FOP Recall till June 2021 with note 82 of 378


‫‪e. Pulmonary Outflow obstruction.‬‬

‫شكرا جروب أكتوبر ‪2020‬‬


‫شكرا لكل من ساهم في هذا العمل‬
‫شكر خاص ل د‪.‬حازم‬
‫ال تنسونا من صالح الدعاء‬

‫‪All FOP Recall till June 2021 with note‬‬ ‫‪83 of 378‬‬
FOP FEB 2020

EMQ:

Genetics: multifactorial, karyotype XO, X


linked recessive, X linked dominant,
Autosomal recessive, Autosomal dominant
(100%)

1. Baby 3 weeks hungry, vomiting Pyloric stenosis , may


congenital or
environmental

forceful, and history grandparent with


operation in young age (multifactorial).

2. Short girl, learning difficulties, webbed


neck (Karyotype XO).

3. Baby Boy 21 months old pull to stand


hold on furnisther,cannot walk tone
normal, history of cousin with disability (X
linked recessive) Duchine

All FOP Recall till June 2021 with note 84 of 378


Infection: acyclovir, fluocxilline, co-
amoxiclave, benzyl
penicillin,trimethoprime,cefotrixone

1. 7 days ill Baby with vesicle in chest


(acyclovir).
2. 7 weeks baby Bullous skin infection
and umbilicus redness and secretions with
fever ( fluoxcilline ). Bulle impetigo, see guidelines, if penicillin allergy, use macrolid

3. 3 years old well girl with urine +ve for


nitrate and leucocytes ( oral
coamoxclave)

Rheumatology: SUFE, Perthus, reactive


arthritis,Oligoarthritis JIA,Septic
arthritis(100%)

1. 3 years Baby with history of diarrhea


and Gastroenteritis in Nursery infection
and limp ( reactive arthritis ) After UTI or GIT

All FOP Recall till June 2021 with note 85 of 378


2. 6 years Boy with history limp for 3
Inv, hip X-ray, ttt, rest
months after exercise ( perthes).
3. 14 years old Afrocaribian boy limp
after exercise with sever hip pain trying to
reduce weight.(SUFE) Ttt, surgery

Palliative: oral morphine ,Patient Control


analgesia(PCA), intranasal morphine, Fentanyl
Patch ,IM morphine, rectal paracetamol, Oral
paracetamol

1. 9 years post-surgery perforated


appendicitis and developed peritonitis
(pca)
2. 3years chubby accident with fracture
Clavicle ,screaming ( intra nasal
morphine) Severe pain Chubby child , difficult cannula, intranasal

3. 14 years old girl with Chronic regional


pain took paracetamol and ibuprofen, still
in pain (Fentanyl Patch)

All FOP Recall till June 2021 with note 86 of 378


git: gastroenteritis , UTI, malrotation, ps,
overfeeding, constipation, all tacks 150 ml per
kg, (100%)

1. 7 week increasing non bilious


vomiting, lethargy fever 38.5 C not well
,not feeding well ( UTI).
2. 7 weeks well baby 48hrs intermitted
bilious vomiting ,baby ,Tachycardia
,capillary refill time 4 Sec (malrotation )

3. 7 weeks old baby boy with non-bilious


vomiting , allows hungry (pyloric stenosis)

Blood: G6PD Deficiency, Osmotic fragility test,


HB electrophoresis, sweat test, Abdominal
U/S
Direct Bilirubin.(100%)

1. 3 weeks with prolonged Jaundice with


pale stool (Abdominal U/S).
Biliary atresia, Haida scan

All FOP Recall till June 2021 with note 87 of 378


2. 3 weeks with consanguinity parents,
Used nitrufurontoin in UTI ,will lead —>
anemia, UTI infection (g6pd assay) G6PD

3. 4 months consanguinity no ethnicity


with pallor parent with spleen +++ (
Child pallor , parents with
osmotic f test) Heridetary spherocytosos
splenomegaly

Recently accurate test ema test

bof:

1. Teenager Female was active and


playing Gymnastic she gain wt. with
decrease school performance
hypothyroidism Acquired

Polycystic ovary
Obesity
2. 2 months old Baby with 2 weeks
cough history not immunized and no
isolation room.(100%)
Admit to SCBU in Isolate
Refer to another hospital
Admit to adult ward

All FOP Recall till June 2021 with note 88 of 378


Discharge
Admit and inform senior
Admit and inform Infection Control

3. Child coughing for 2 weeks CBC show


lymphocytosis WBCs 17.000
Pertussis(100%)
TB
Measles
3 c , corysa, cough, conjunctivitis

4. Child 10 months with fever and cough


with conjunctivitis , fever ,start rash from
face and spread to all body. His brother
newly admitted to Nursery (Same as
Sample Paper) (100%)
Measles
Roseola infantum
Scarlet fever
Kawasaki

5. Patient with tonsillitis and history


penicillin anaphylaxis

All FOP Recall till June 2021 with note 89 of 378


Clindamycin
Ciprofloxacin
Cephalexin
Clarithromycin
6. Child live with grandmother under
protection program came with ruptured
Appendicitis came with grandmother

Who can consent? 1. Child if he is competent if above 16 year


Scotland or above 18 in British
-Grandmother 2. Mother even by phone
3. Father even not married as long as his name in
birth certificate

-Social service 4. Surgery if it is emergency


5. Court if not surgery

-surgeon who do the Operation (100%)


-Doctor of child protection program???

7. 3 years old girl with night and day


coughing. Mother is warred the baby may
be asthmatic
What to do
- peak inspiratory flow and follow up
-Spirometry after b2 agonist trial
-try of inhaled steroid for 4 weeks and
follow up.
- Spirometry

At age of 6 years —> can use spirometer

All FOP Recall till June 2021 with note 90 of 378


-Fraction of inhaled FeNO

8. 2 month baby came to clinic with


fever 38.5 Continues crying and poor
feeding and take paracetamol ,become
well
What to do?(100%)
- full septic screen and start IV Cefotaxime
-Admit to the ward and observe.
-Send home and if sick came again
-CBC, urine culture and observation all
over the day.
-Oral Antibiotics

9. Child 10 years old on 2 puff preventer


q 12 hours on treatment for asthma was
controlled on 2 puffs twice daily.
There are new local guidelines cheaper
15%, the old guidelines recommended to
wean if controlled.

- give 1 puff of old preventer,


-use the new preventer same dose

All FOP Recall till June 2021 with note 91 of 378


-use the old preventer same dose
-Stop Spacer
10. Child with speech problem and
improved now has problem in playing and
unbooting. He has problem in writing and
copying but good in reading and
math(100%)
Dyspraxia Coordination

Autism
DMD

11. 5 years boy with symmetric crawling


,normal tone walk at 15 months, his uncle
23 years on weal chair(100%)
DMD Average age 5 year

Backer dystrophy After 10 year

Dyslexia
DDH
CP

12. Female with learning difficulty and


lens dislocation (100%)
-Homocystinuria

All FOP Recall till June 2021 with note 92 of 378


-Marfan Syndrome No learning difficulties

-Prader- willi Syndrome


-Soto Syndrome
13. Child with swelling in the eye lead and
ophthalmoplegia (no Ceftriaxone)100%
-IV Cefotaxime Peri orbit cellulitis—> oral coamoxilav
Orbital cellulitis —> ceftriaxone or cefotaxim

-Clindamycin
-Oral Co Amoxiclav
-Metronidazole
14. 6 years old boy was day time
controlled and night time bed wetting2
times /Week (100%)
What to do?
-Motivational 1. Motivation and behavior
2. Alarm
-Alarm ?? 3. Desmopressin

-Oxybutynin
-Desmopressin
-
15. Child 6 years in the school reduce fluid
intake with soling and teethed at school
= constipation
because his odor

What is the initial management?

All FOP Recall till June 2021 with note 93 of 378


-Increase fluid intake
-laxative (100%)
-refer camps
16. Parents with learning difficulty, the
child is not taking or waling is talking 3
wards and neurological exam is free.
- Neglect
- CP
-Sexual abuse

17. 5 months baby smiling at 8 weeks and


not follow his mother now
What is the more concern for doctors?
-absent red reflex Cataract Red flag
1. White reflex = absence of red reflex
2. No smile at 6 weeks
3. No eye to eye contact
- H.C 0.4 Centile 4. Squint at 3 months
5. Nystagmus

-present Moro reflex Disappear in 4-6 months

18. When to refer to Surgery old at 6


months?
-Inguinal hernia (100%) IfIf reducible —> early surgery appointment
not reducible —> emergency surgery

-Epigastric hernia
-Umblical hernia At one & half

-Phymosis
Hydrocele — > at 2 year

All FOP Recall till June 2021 with note 94 of 378


19. Child 2.5 years the mother is concern
about his development
What is concerning not doing at this age?
-say 3 words only (100%) Supposed at age of one year

-draw line but cannot copy circle


-cannot pick role in a play
-Climb star step by step

20. Newborn with palpable gonads and


hypospadias
What investigation leads to diagnosis? Exclude CAH

- 17 Hydroxeprogesterone (100%) 3diagnosis rd day but it will lead to

-karyotype 1st day

-Abd us Also 1st day

-S Electrolytes 2nd day

21. Child with UTI and has renal stones by


U/S urine analysis +ve leucocytes and
nitrates what is the cause Hematuria + flank pain —> proteus

-E Coli
-protus infection (100%) Urease screaming org. Split
ammonia in urine , Chang urine
acidic to alkaline
-Klepsillia

All FOP Recall till June 2021 with note 95 of 378


17 record

22. Baby 3 months old with sever reflux


refractory for 1 st line treatment
Consultant ordered Omeprazole, mother
is Pharmacist and saw the leaflet and ask
you about the drug

- Unlicensed drug safe in Adults and not


studied in this age group (100%)
-Ask the consultant to write licensed drug
-till them that this drug is not
experimentally approved but with safe
effect
-till them you do not know and the
consultant will change it
23. 15 years old girl has period 2 weeks
ago with flank pain with fever, sweaty and
irregular cycle with pain score6/10 and
distressed.
What is the initial investigation?
-( kub and xray, UTI

-urine analysis
- abd, us
- blood culture)

All FOP Recall till June 2021 with note 96 of 378


24. 10 years old Tall Boy with advance
bone 1year his height 90% and wt. 98%
blood pressure 91 %and his sister and
mother were normal weight
No other abnormality and not started
puberty after 2 months wt and hight still
increasing

-simple obesity (100%)


-Cushing Central obesity weigh more than high

-Adrenal tumor Dark skin

25. 14 years Obese Girl with wt loss and


poly urea with family history of DM
What to do next?
-Random Capillary blood sugar (100%)
-HA1c
-Fasting Blood sugar
-C Peptide
-Glucose tolerance test

All FOP Recall till June 2021 with note 97 of 378


26. 11 years child diabetic was visting
sister in the hospital mother found him
shaking , she measuere RBS it was 2.5
mmol/L
What best action? Or gluco gelid
-give 100 ml sugary fluid (100%) persisted—> admission
-peac of Bescket
-par of cholcite
-IM Glocagon
-Toast
27. DKA Scenario 14 years old pulse
120/min Tachycardia —> impending shock

What is the initial maintenance?


- 0.9 sodium 500 ml 20 mmol KCL (100%)
-0.45 NS 500 ml with 20 mmol KCL

28. Baby 5 months old started formula


feeding then developed Ecezema and
diarrhea
What to do?

- Extensive hydrolyzed formula (100%)

All FOP Recall till June 2021 with note 98 of 378


-Soya milk formula Prone
Heating
-Lactose free formula Head coverings
Bed sharing
-Amino acid formula Smoke , PT

29. What is the most risk factor for SIDS


-Parent bed sharing (100%) The first prone position

-Room sharing
-Dummy Decrease sids

-Smoking

30. 14 years old Girl lack of interest , not


refusing investigation or toxicology
screening and screening was normal , she
was refusing eating with wt loss
-Depression (100%)
-Autism
-ADHD
-Anorexia nervosa Refuse any investigation or argue for treatment
Age between 15 - 25 wt = 17.5

31. Girl her patents quaral with her she


disappers for 5 hours and was found with
vomiting and she takes 18 paracetamol
tablets

All FOP Recall till June 2021 with note 99 of 378


What to do?
- iv acetylcystien infusion (100%)
-Charcole For 2 hours

-Oral actelcystine
- chek the level now and after 4 hours
-chek the level now and after 2 hours
32. Child came with cyanosis life
threatening asthma take nebulized
salbutamol and Oxygen
What you do next
Call Anesthesia (100%)
IV Mg sulphate
IV salbutamol
IV corticosteroid severe asthma
Oral Pridnzolone
33. Pt with generalized Oedema with
protein ++++and –ve nitrate and
leucocytes puffy eyes
What is the treatment
- steroid 100%
-Lasix
-Albumin

All FOP Recall till June 2021 with note 100 of 378
34. Term delivered with thick meconium
stained liquor seen in the Oropharynx and
baby was flat ,white and cyanosis and HR
100/min what is the next step
-suction under direct visulation 100% ABC

- Inflation with bag and mask


- Dry and stimulate ‫غلط‬
-Endotracheal intubation

35. Female Child came with recurrent


abdominal pain high ESR and arthralgia,
diarrhea with mucous and mother with
vitilgio
What is the most diagnostic investigation -
-colonoscopy 100%
-Celling screen (TTGA)
-Barium enema

36. Girl 6 years Carrabin with 2 weeks fever


and lethargic now has limping with Low Hg
7 and platelet 22000 and WBC 6000 and
Splenomegaly with fatigue.
Diagnosis

All FOP Recall till June 2021 with note 101 of 378
-ALL 100%
-ITP
-NHL
-Sickle cell anemia

37. 11 years boy Obese with pigmentation


in the neck and glycosuria and RBS 7mmo/L
and with mild elevated liver enzymes
What is the diagnosis?
-Fatty nonalcoholic liver disease 100% Obese
insulin
, resistant to

-Hepatitis
-Type 2 DM
-Cushing disease

38. Term Baby born and developed Jaundice


on day 3 of life and received Phototherapy
for 2 days then discharged home on day 4
of life came on day 6 with fever 38.3 and
metabolic acidosis PH 7.3 and
hepatomegaly 2 CM below the costal
margin and hypoglycemia 2.2 mmo/L

All FOP Recall till June 2021 with note 102 of 378
-sepsis 100% Late

- Renal tubular acidosis


- Collagen storage disease
-Cong.Heart disease
-Kernicterus

39. 12 Month old infant came with his


mother 17 years baby with GCS 6 and with
bruises on the Abdomen and blood form
mouth
Suspected NAI,, Head injury,, post
-CT head 100% traumatic injury with GCS

-Skeletal survey
-Coagulation profile

40. Well Baby delivered his mother has


bruises his brother 3 years in child
protection
- Mid wife of safe guarding 100%
- Keep baby with the grandmother
-Call Police No threatening to call police

-Admit and do investigation

All FOP Recall till June 2021 with note 103 of 378
41. Child 14 years old play Rugby
developed Sudden tender scrotum in left
testicular
-Lt.Testicular Torsion 100%
-Torsion Morgagni
-Inguinal hernia
42. 5 years old Girl developed skin rash
after eating Ice-cream , no difficult
breathing with history of asthma and
eczema, other examination were normal
-Oral Antihistaminic 100%
-IM adrenalin
- Oral Steroid
-IV Adrenalin

43. 3 years old Baby boy came by


Ambulance from restaurant with
drooling of saliva and Stridor and difficult
to swallow and refuse Juice offered with
history of asthma
-FB inhalation 100%
-Anaphylaxis
- Epiglottitis

All FOP Recall till June 2021 with note 104 of 378
44. Diabetes table
Pt takes Take Long acting in the Night 20
IU and 1 unit for 8 gm carbohydrates
before breakfast and 1 unit insulin for
each 10 gm carbs before lunch and 1
unit for 10 gm carbs in the evening.

(The readings are high before bed and


evening meal) 100%

Monday Tuesday Wednesday Thursday


Pre Bfst 4 4.4
breakfast 7
Before 5
lunch
Post Lunch 12 10 9
Pre Dinner 12 13 18 15
Post Dinner 15
Bed time 18 21 20

- Increase short acting Lunch and dinner


-Increase long acting alone

All FOP Recall till June 2021 with note 105 of 378
-increase short acting of lunch and
dinner and long acting
-Increase short acting of dinner
-Increase short acting of lunch
45. Child 3 months old with mild
dysmorphic features His O2 saturation
95% with loud systolic murmur all over
pericardium not cyanosed with FTT

-VSD 100%
-TGA
-ASD
-PDA Muchinary main site sub clavicle but can be 2nd intercostal space

-TOF
46. Neonate came with heart failure
What is the sure sign of Heart failure?
-hepatomegaly 100%
-Cyanosis
-Weak Peripheral pulses
-Pedal Edema
47. PT 32 weeks wt. 1.3 kg admitted to
NICU was ventilated and developed E
Coli meningitis received IV Antibiotics for

All FOP Recall till June 2021 with note 106 of 378
3 weeks with Increased H.C from 35 cm
to 37 cm.
What is the most important long term
complications?
- Retinopathy of maturity
-deafness 100%
-Hydrocephalus Need to follow up to put on centile

-CLD

48. Child 6 years old with history of


repeated ear infection and otitis media
in group activity became angry but can
enjoy art lessons
What is the possible cause ?
-ADHD
-Secretory Otitis media 100%
-Autism
49. Pt came with vomiting and
examination has petechi received
metoclopramide then has neck stiffness
and rolled eye

-oculogyric crisis 100%

All FOP Recall till June 2021 with note 107 of 378
-Rolandic seizures
-Myoclonic seizure
-
50. Child in a Camp with congenital
neutropenia then developed fever 38.7
C the mother call you asking for Advise
What to tell parents?
-Advise her to give paracetamol and
observe
- Come to hospital if he became unwell.
- Go to Hospital as soon as possible 100%

51. Boy 7 years came with Absence


seizure and 3 Spike wave /Sec and
become staring for few seconds multiple
times every day. 1st choice ethoxomide

What to give?
-lamotrigine In girl

- Sodium Valproate 100% In boys

-Phenobarbitone
52. 2 years old came with 6 weeks history
swollen Lt Knee Join and 1 metatarsal
joint with no rash or fever , and no

All FOP Recall till June 2021 with note 108 of 378
history of HLAB 27 +ve and maternal
aunt psoriatic arthritis Ankylosis got spondylitis
Psoriasis
Reactive arthritis

What is the diagnosis? IBD

-systemic onset juvenile arthritis


-Oligoarticular JIA 100% See FOP

-Polyartecalura arthritis
-Psoriatic juvenile arthritis Need score
Casper

53. Child 4years with intermittent stridor


and difficult swallowing lumpy foods
What is the diagnosis?
- Vascular ring 100%
-NHL
- Heamingioma
-Larnigomalacia
54. Baby 2 weeks what is the most
important indication to refer to
Audiology assessment?

- When parents concern he doesn’t


startle to noise 100%
-If he missed neonatal screening
-If there is per auricular skin tag
See SOP

All FOP Recall till June 2021 with note 109 of 378
55. Girl with abnormal repetitive hand
movement and regressed learning, she is
withdrawn
What is the diagnosis?
-Retts Syndrome 100% Good in math

- Autistic Spectrum disorders


-Global developmental delay
-
56. Child came with swelling since 3
months in the Lt Anterior triangle with
discoloration and U/S shows clear fluid
Born in UK fully immunized
- Atypical mycobacteria adenitis
-Infected branchial cyst
-TB
57. Child came with low Ca ,Phosphate
and High Alkaline phosphate and High
PTH
-vit d deficiency 100%
-Hyperparathyroidism High Ca
-CRF High Phosphorus

All FOP Recall till June 2021 with note 110 of 378
58. Mother found OCPs in her 14 years old
daughter room, she ask you not to write
OCPs * if 13 or below ; - if emergency —> give ocd and inform self
regard like consultant or social worker police
- if not emergency —> don’t give
What to do? * if 13-16 ; give ocd and advise social worker
* if 16 ; giv

-Advise her to discuss with her daughter


100%
-Stop prescribing the medications
-
59. 1 month boy his brother has
meningitis what to do for prophylaxis?
-single dose ciprofloxacin
-Rifampicin All age ciprofloxacin

-Ceftriaxone Only if H influenza or child below 10 year


give rifampicin

-Cefotax

60. Child came with end stage


adrenodystrophy to another hospital
with respiratory symptoms then his
condition become more worse
What to do?
-ask parent if they have end life plan
-Call consultant
-Intubation with ventilation

All FOP Recall till June 2021 with note 111 of 378
61. 3 years Child came diarrhea and
undigested food particles and his wt. on
9th centile
- refer for dietitian
- Reassurance 100%
-Antispasmodic
62. Pt 3 years has multiple mollascum
contagosum not annoying him
What to do?
-reassurance 100%
-Crytherapy
-Sclerotherapy
63. Patient came after upper Respiratory
tract infection with ecchymosis and
purpuric rash and low platelet no HSM
and small cervical LN
-ITP 100%
-HSP
-ALL

All FOP Recall till June 2021 with note 112 of 378
64. 8 years girl came with morning
headache and poor sleep and new onset
squint
What is the diagnosis?
-Space occupying lesion 100% Night terror

-Meningitis
-migran
65. 10 years old boy, he is awake at Night
and screaming and go back to sleep, in
the morning he cannot recall.
What is the management?
-Reassurance 100%
- Sleep Hygiene
-Melatonin
-keep light opened
66. Girl 14 years her length 147 cm and
her mother 162 cm she ask if she will be
tall as her mom
Listen 51

-8 cm increase in the last year


-Tanner stage B2
-Tanner A2
-she has Menarche

All FOP Recall till June 2021 with note 113 of 378
67. Child admitted in the ward with
bronchiolitis he has fever and convulsion
for 2 minutes and nurse give him O2 and
RBS was 6 mmol/L and saturation 94%
what is your next action?
-observe 100%
-Rectal midazolam
-Insert Cannula
-Buccle midazolam
68. 4years Child has Epilepsy newly
diagnosed
What to do?

-The parents should agree with the plan


of treatment 100%
-Educate the student about the epilepsy
in childhood
-teach the hall school staff about basic
life support
-Exclude form school Activity
-the class assent should be with her all
the time

All FOP Recall till June 2021 with note 114 of 378
69. Child 13 month old with nephrotic
Syndrome on Oral Cortistorid dose what
is contraindicated vaccine?
-MMR 100%
-IPV
- DTP
-Hepatitis B Vaccine
70. Child 7 years has infrequent SVT
Attacks mother is worried if he has
attack outside.
What to do?
-immerse face in ice water ‫غلط‬
-Inflate in New balloon 100%
-Ocular Pressure
-propranolol
71. Boy 14 years old. He has parotitis
7 days and is athletes since 11
years his wt on 0.4 centile drop
from 50% height on 25 % by
examination he has prepubertal
testicle

All FOP Recall till June 2021 with note 115 of 378
What is diagnosis?
- Testicular atrophy
-Anorexia nervosa100%
-Pituitary tumor
-Cushing
-Hypopituitarism
72. Father of patient find a file of pt in
the resurant and give to you and
you find it is the hand writing of 1
of your college
What is the most appropriate action?
-Till the father not to till any one
-talk to your colleague100%
-Inform the GMC
-Report to the government
-inform your consultant

All FOP Recall till June 2021 with note 116 of 378
73. Mother has lymphoma and she
don’t like any one to know she has
conflict with the father and child has
FTT and there is meeting for child
Condition. He is the family doctor.
What is the GP role in the meeting?
-read growth chart
-give information about the family
-give information about the child
medical history. 100%
-Advocate for the parents
-give report about family
accommodation
74. Baby born with ambiguous
genitialia family want to give baby

All FOP Recall till June 2021 with note 117 of 378
name For assignment of sex , need multidisciplinary

What you will till the parents?


- wait for chromosomal results
-Give name that can be fit for male
or female
-give male name it is most probable
male.
-Do not assign name and wait for
the full investigation results 100%
75. Pt 28 weeks was ventilated wt 0.94
kg was feeding through NGT has
abdominal distension and bloody
stool and lethargy.
What is diagnosis?
-Necrotizing enter colitis 100%
-Hurshsprung disease

All FOP Recall till June 2021 with note 118 of 378
-Intussuptation
-meconium illus
76. Baby was under child protection
came with smooth plitrum and
learning deficility and thin upper lip.
What is diagnosis?
-Fetal alcohol syndrome 100%
-Brader willi Syndrome
-Phynotonin syndrome
77. Pt 6 years has recurrent ear infection by
examination has mass behind ear and
increase upward and laterally and
pushing ear downward has Mastoiditis

What to do?
-CT head 100%
-Swab ear discharge
-U/S
- Blood Culture
-X ray mastoid bone.

All FOP Recall till June 2021 with note 119 of 378
78. Pt has hot and tender Knee Joint and
has fever(Septic arthritis)ESR 40
What is best next step?
- Joint aspiration 100% See illustrates 492

- Knee U/S ‫غلط‬


-X ray
-IV antibiotics

79. FBC with target cell anemia


consanguinity,Microcetic hypchromic
anemia Thalassemia or iron deficiency anemia

(Hb electrophoresis) 100%


High performance liquid chromatography

‫دعواتكم لنا بالنجاح والتوفيق‬

All FOP Recall till June 2021 with note 120 of 378
October FOP 2019
1.EMQ: Same as Sample paper
laryngomalacia
subglottic stenosis Associated with intubation
subglottic hemangioma
viral croup
vascular ring Dysphasia , stridor, wheezy chest , increase with rest

1-One week old has stridor since birth,incrase with sleep and feed
laryngomalacia Increase with cry and supine sleeping

2-weeks old operated for inguinal hernia when he was 4 weels old ,suffering
from stridor ,improved partially with steroid ,now biphasic stridor --
subglottic hemangioma Sometimes mentioned child has navus in face

3-infant went to a party and had two courses of antibotics but no improvement
had a chest xray with lesion in the right lower ,after the second course the
same xray
Foreign body Foreign body scenario in party or restaurant
Investigation; CXR , bronchoscope which diagnostic and therapeutic
2.EMQ:
educational therapy
occuptional therapy
orthotist
speech therapy Speech therapy job
1. Hearing test
Refer to dietitian 2. Pronounce of little
Physiotherapy 3. Coordination between speech and swallow

1- 9 monyh old with development delay has problem with solids and take long
time to feed speech therapy

2- 4 year old has tip toe orthotist

3-child with hemplegia ,leg discrepancy has hip,knee pain and xray was
normal Physiotherapy To release spasticity by physiotherapy

3.EMQ:
listeria
GBS
nisseria
staph aureus
staph epidermids

All FOP Recall till June 2021 with note 121 of 378
Early sepsis; GEL,
1. GBS
2. Echoli
3. Listeria
1- term infant after 24 hours developed infection,culture was postive cocci
GBS

2-baby wiyh meconium stained liquor c\sculture postive for gram postive
bacillus listeria

3-baby with parentrral nutriton staph epidermids

4.EMQ:
decrease long acting insulin
decrease short acting insulin
continue same dose insulin with oral rapid acting glucose fluid
continue same dose with oral fluids
extra short acting insulin

1-pt. diagnosed with DM has voimiting and nauseated RBS 4.6(normal)


continue same dose with oral fluids
continue same dose insulin with oral rapid acting glucose fluid
2-14 years girl diagnosed with DM came with morning hyperglycemia
RBS at 2 am 2.9 (low)
RBS at10 am 10-15 decrease long acting insulin

3-child with DM vomited once ,has abdominal pain ,RBS21,ketone ++


extra short acting insulin * bms < 4 —> go hospital—> if drowsy give glucagon or surgery fluid if good oral intake
* bms (4-10) normal —> if good oral intake —> same insulin dose + sugary fluid monitor every 2 hrs
—> if no oral intake —> decrease insulin + sugary fluid
* bms (10-14) —> same dose of insulin + plaint of fluid to avoid dehydration
* bms > 14 —> if +ve ketone ; go hospital + extra insulin
—> if -ve ketone ; extra insulin at home + check bms after 2 hours
5.EMQ: If not improved after 2 dose of insulin —> go hospital

VSD/ASD/PS with normal heart sound /PDA/innocent murmur

1- 4 or 6month with ejection systolic murmur the lt upper sternal


PS with normal heart sound

2- 34 week at 10 days with long systolic murmur BP 70/30 mmHg rt


arm and Lt leg no deference (Wide pulse pressure) PDA
3-pt. with loud systolic murmur at lt lower sternal VSD
6.EMQ: affected Not affected Carrier
1/4 1/4 2/4
1:4/ 1:2/ 2:3/ 1:100/ 1:10/ population risk
1-A case of sickle cell and asking of risk of having another affected child AR
1/4
2-father with hemophilia A what are the risks for having an affected boy XR
population risk

If question chance of transferring the case from father —> zero


All FOP Recall till June 2021 with note 122 of 378
3-risk of having a carrier sibling for a family with affected cystic fibrosis child -
2/3

7.EMQ:
EBv
ALL
Reactive lymphadenitis
Atypical mycobacterium
Kawasaki disease
1-child with fever,then lymph node periauricular soft,mobile non tender1-1.5
cm has facial oedema and well child reactive lymphadenitis

2-child with fever for one week ,red tongue,cervical LN,erythrmtous rash
Kawasaki disease

3-child with fever,hepatospleenomegaly,cervical LN EBV


8.EMQ:
gardiasis
rota virus
cow milk
celiac Miserable , flat buttocks, abdominal bloating
1- 3year old child well apart from having diarrhea with food particles
toddler diarrhea
2-baby on breast feeding ,mum shiffted to formula and he is well and thriving
has eczema,developed watery diarrhea 24 hours ago rota virus

3-baby ,mum changed to formula under pressure of her family diagnosed with
colic at 5 weeks of age,has nappy rash,eczema and red scalp cow milk

9.EMQ:
reassurance
oral grisofluvin
topical steroid

1-girl with pneumonia had hair loss with unequal hair loss reassurance Telegon effavum,
dt stress
2-Boy with batch of hair loss (Alopecia) topical steroid

3-girl with scaly lesion in the scalp (Tenia) Crusty


oral grisofluvin

10.EMQ:
Syncopy
See SOP

All FOP Recall till June 2021 with note 123 of 378
Complex partial Loss of consciousness Tired, can’t remember
Simple partial
Day dreaming
Absence
1-boy looks frightened,pale ,repetitive swollwing for 15-30sec after which he is
tired Complex partial
Or loss of consciousness
2- girl at the assembly lost her consc.
Pale and twitches after she regain consciousness she is confused Syncopy

3-child noticed at school by the teacher go still for 1-3 minuets Day dreaming Only in the
school
Absence both school and home ,also occur within seconds

BOF:
1.somali mother want to breast fed her baby what vitamin deficiency will
be the most :
1.vit D. 2.vitA. 3.vit K. 4.vit C
2.vaccination for 2 month old baby(Know vaccination by Heart)
1.Dtap+RV+MenB+pcv
2.Dtap+RV+Men B+ppv
3.Dtap+RV+Menc+ppcv

3.Cystic fibrosis pt,first time to have pseud.aurgionosa ,doesn’t want to


take the oral cipro hates the taste and baby was asymptomatic what to
do:
1. Admit for IV antibodies
2. Nebulized to tobramycin or ???
3.oral tablet ????
4.stop cipro and wait for the C.F team on Sunday

4.you are junior doctor and young parents brought their baby with spiral
fracture of Humeros what to do: Fracture
1.call senior registerar) Spiral fracture of humerus
2.skeletal survey Metaphyseal fracture of femur
Posterior rib fracture
3.Till parents you are suspectiog child Abuse Callus formation > 42 days
4.Call senior Orthopaedics Fracture of clavicle in age > 1 month

Skin
Oval shaped burn
5.one sign for urgent CT head: Bruises behind ear, pinaa bruises
Human bite
Frenulum tear
1.bruises around the eyes Cigarette burn
Retinal hge more than 42 days
2.amensia for more than 5 minutes
3.loss of cons. For more than 5 minutes

All FOP Recall till June 2021 with note 124 of 378
6.spironlactone as unliscend drug,the junior doctor looked it up on the internet
and found its not used for this age,what’s the most important action
1. Inform the parents you will use unliscened drug
2. Write on the pt notes that you will use unlicensed drug
3. till the parents about the side effects of drugs.
4. Monitor electrolytes

7.pt had a wrong blood transfusions what can you do to prevent that:
1.check the pt. wrist id
2.check the bedside identification and needs for transfusion

8. The nurse missed a dose for vitamin k for a new born and parents is waiting
outside what is the initial steps: 1. Safety
1.inform the parents and consultant 2. Patient or parent
2.inform the consultant and make an incident report 3. Consultant
4. Documents
9. 2 month with bruises came on Friday night and you suspect child abuse
what to do :
1. Brain CT If not day off—> immediately inform social
2. admition and inform social worker worker for further investigations

10.tall boy ,secondary sexual character not will developed teased at school
with mild learning deficility .
1.46xy. 2.47xxy. 3.46xx
Kleinfter Syndrome
11.when to transfer an cystic fibrosis pt to an adult service:
1.when he is familiar with the staff in adults clinic
2.when he is aware of the dose and the way to take his medicine
3.When he is aware about his disease
4.Aware to take decisions.
5. Understand his role and decision making ‫عليه اختالف‬

12.DM pt developed hypoglycaemia after playing match the school did the
RBS and called you to tell it’s 2mmol what advice to give them
1.give him sugary drink ½ Cup If child consciousness
3.inject with Glucagon IM
3.call ambulance

13.14 year old boy with chlamydia infection what’s the treatment:
1.doxcyclin for 7 days or single dose azithromycin
2.amoxcillin
3.cipro

All FOP Recall till June 2021 with note 125 of 378
14.15 year old girl with regular peroids of which the last one was 2 weeks ago
,came with Diarrhoea,vaginal discharge,BP90/50 shaking and feverish what’s
the diagnosis: Hypotension
1.gonorrhea
2.toxic shock syndrome See survival

15.2 years old boy with polyurea ,polydepsia,urine osmorality 200


Diagnosis: normal urine osmolarity 500-850
1.DI nephrogenic normal plasma osmolarity 275-290

2.DI central
3.habitual drinking Here urine osmolarity between 200-300

16.pt taking carbemzpine and was sick took erythromycin then came with
unsteadiness what’s the cause
1.non conclusive eplipsy Erythromycin enzyme inhibitors—> no carbolic of
2.carbamezpine toxicity carbamazepine —> toxicity

17.child had measles then afterwards developed seizure what’s the


investigation:
1.cranio imaging Ct or MRI Measles developed seizure after 10 days —> ADEM
2.LP Measles developed seizure after long time 10
year —> subacute sclerosing panenchephalitis
18.sudden infant death most important risk factor:
1.parent smoking
2.co sleeping
3.bottle feeding

19.pt had coryzal symptoms,developed hematuria ,proteinurea what’s next


investigation:
1.urea,creatinine (IGA nephropathy) IgA neohropathy URTI just few days 3-4 days
PSGN ; URTI 2 weeks back
2.FBC

20.malignancy pt with bone pain ,she is taking parcetamol 4 times and oral
morphine prn what to do:
1.rectal diclofenac
In constant continuous pain
2.slow release morphine with morphine breakthrough
3.IV morphine In other scenario if child competent, can
keep her on morphine on pumps
21.14 year old girl,has a15 year old boyfriend,they had unprotected sex once
and came to ask for emergency contraception,she understands the benefits
and uses of contraception,she doesn’t want her parents to know what to do:
1.tell her you can’t issue the pills unless her parents are notified

If big difference in age eg; 15 y and 25 y , call police

All FOP Recall till June 2021 with note 126 of 378
2.perscrib the emergency pills If has atrial fibrillation—>
emergency referral
22.girl with palpitaion and tremors for 6 weeks TSH 0.02 Scenario of hyperthyroidism
T4 and T3 are high what to do:
1.send her to endocrine clinic in 2 weeks not emergency, as she suffer already for 6 weeks,
2. Give her carbmizaole ‫عليه خالف‬ can wait for clinic
3.give her propyiothiouracil Choice of carbmizole should prescribe by endocrine

23.girl had irregular cycle , hirsutism and dark pigmentation what’s the
diagnosis: Polycystic ovarian disease Polycystic ovarian disease or
(Stein-Leventhal syndrome) presents with obe-sity, hirsutism
(sometimes with acne) and irregular periods (also primary or secondary
1.PCOS amenorrhoea) usually from puberty and the symptoms deteriorate with
time. Numerous ovarian cysts are seen on ultrasound. Androgen levels
2.cushing No irregular cycle are elevated (SHBG low) and there is a raised LH:FSH ratio with a
normal or low FSH (that is, LH is high). Infertility is a common problem
later on, and treatment with clomiphene or wedge resection of the
ovary can be effective

24.pt was diagnosed withDM and his sugar level were controlled for five years
now ,his father has HTN what’s the most important thing to tell his family:
1.he will need insulin for life
2.he will need low carb diet

25-13 years.pt on the 50th centile for wt and on the 75th centile for Ht she is not
growing for the last 2years only 4 cm /year (mid parentral centile was not on
it)her school performance was acceptable tanner staging B3 P3
what to do: Choose by exclusion Growth hormone insufficiency
1.TFT. Bad school performance
2.anti tissue trans glutamates No weight affection
3.IGF ‫ عليه خالف‬IGF Level
4.LH/FSH Tanner staging ok
5.chromosomal Turner’s short since birth

26. 6 week old baby breast fed,jaundiced,had meningitis what to give:


1.cetrixone Increase jaundice
2.ceruxime
3:gentamicin
4.Cefotaxime

27.baby with jaundice ,coombs negative,total bilrubin 120


Conjucated 90 what’s the diagnosis:
1.bliary atresia
2.galactosemia

28.18 month old with vomiting and diarrhoea the last one was bloody HB
10,CRP 20 ,WBC 14 In the scenario also platelets low
1.intussception
2.bacillary dysentery
Another diagnostic item —> urea & electrolytes
Diagnosis ; HUS
Organism; E coli

All FOP Recall till June 2021 with note 127 of 378
* meningococcal meningitis—> cipro
3.Ecoli * H influenza meningitis —> rifampicin single dose
* Pneumococcal meningitis —> no ttt required

29.6years old with sibling had meningitis what’s the prophylaxis:


1.cipro
2.ceftraixon

30.what is the most important indication to do CT head for pt with headache:


1.waking up with vomiting
2.increase headache frequency
3.increase headache severity

31.pt was diagnosed with a life limiting condition condition his parents ask if
they should refer him to hospice ,he needed NG tube what to do:
1.refer them to initiate advanced care plan
2. Refer them to hospice Hospice part of holistic
3.must provide him with full care Hospice only given feed , hygiene & drugs
So holistic more facilities like ngt feed insertions
4.he is too ill
5.to refer him to holistic palliative care

32.child recently moved to new house and school,he is out of control at home
that his mother had to call the police,he gets upset for missing with his tidy
toys,at school he is quite,good at his math
1.autism spectrum
2.ADHD

33.parent with child when he cries he becomes blue and fall down what advice
to give them: Breath hold attack
1.distracte him from the trigger
2.give him O2
3.give him bag to breath in

34.baby with vomiting,plyoric stenosis what’s the most clinical sign:


1.lost wt
2.varcious hungery
3.non bilious vomiting

35.12 year old girl with depigmented spot 1mm she had 4 Patches ,good at
sports,mother says her back is deviated,when she puts her hands on the
floor,won’t be corrected what’s the cause:
1.NF
2.tubers sclerosis
3.idiopathic

All FOP Recall till June 2021 with note 128 of 378
36.child with muscle myopathy,postive gower sign what’s the diagnosis:
1.becker
2.myotonic dystrophy
3.duchenne

37.2 year old girl ,history of ventilation as nenoate ,cruise around furniture ,tip
toe walking: Preterm
Tip toe —> CP
1.DDH
2.spastic diplegia In preterm , liable of interventrecular hge, preventrecular
3.duchenne leucomalecia, —> necrosis on brain tissue spastic diplegia

38.Asian12 year old with rash on her face and joint swelling what
investigation: SLA
1.DsDNA
2.ASO titre

39.6week old baby what to do for hearing: See SOP


1.distraction test
2.ABR
3.pure tone

40.mum went to GP for her child tonsils what’s the mosta important point to
put in the referral letter: Obstructive sleep apnea for surgery
1.he has big tonsils
2.recurrent URTI
3.can’t wake him easily in the morning

41.baby brought by his mum had fever 38C mum afraid he will get meningitis
what to tell her:
1.when he develops rash go immediately to the ER Like FOP case 48 , sign of shock
2.when he becomes pale seek emergency help pallor dt circulation compromised
3.try to elicit neck stiffness,if it’s there go to the hospital

42.pt brought by his mum had vomiting and cough ,becomes blue,
What to do:
1.prenasal swab Persistent cough then blue —> pertussis; diagnosed by
paranasal swab, ttt erythromycin
2.FBC

43.2 month with Klebsila and on antibiotics what to do:


1.US prior to discharge,MSUG at 6wres,DMSA at4-6 month
2.US prior to discharge MCUG and DMSA at 6 month
3.USand MSUG piror to discharge ,DMSA at 6month

All FOP Recall till June 2021 with note 129 of 378
44.pt had fever 40,confluent rash,conjunctivitis,red tempanic membrane what
to do:
1.give Antibiotics
2.wet wrap
3.oral parcetamol

45.5 days old baby breast fed,jaundiced Na 151


What to give: See guidelines
1.150ml/kg/day NG tube or cup 1. Breast feed
2. Cup up , syringe 100 ml/ kg/ day
2.100ml/kg/day NG tube or cup 3. Ngt
3.dectrose IV 4. Ivf

46.neonate delivered,after inflation breath,HR less than 60,there’s chest


Movement,what to do :
1. Dry and stimulate
1.dry and stimulate 2. Ventilation
2.inflation breath for 30 Sce 3. Intubation
3.chest compressions 4. Chest compressions

47. Baby took methadone,his O2 sat 98 ,has noisy breathing what to do:
1.secure airway ABC
2.IM naloxone

48.pt had illness after which he had basal crepitation,HR140,RR40 ,liver 2cm
what’s the diagnosis:
1.Viral myocarditis Coxsakei virus and adenovirus can both cause myocarditis Symptoms of HF

49.boy collapsed after swimming,slowly recoverd his con.har cousine with


epilepsy Any collapse after exercise —> prolong qt interval
What to do :
1.EEG
2.ECG
3.excersice ECG

50.down syndrome 12 year old child with VSD ,developed fever,had spleen 4
cm new murmur
Urine +++ blood
What’s the diagnosis:
1.ALL
2.infective endocarditis

51.pt had recurrent broncholiotis in winter,went to a party last week,then


developed a cough,took two antibiotics course ,xray with consolidation in the

All FOP Recall till June 2021 with note 130 of 378
right lower lobe ,the same xray after the antibiotics are finished what’s
diagnosis:
1.pneumonia
2.F.B

52.pt with illness took antibiotics then developed painful nodules over shin
what’s the cause: Erythema nodosum common with
1.mycoplasma pneumonia mycoplasma pneumonia
2.staph

53.pt developed wheez and rash after eating what to do:


1.oral antihistamines
2.im adrenaline
3.im phenypherine

54.pt received MRSA infection what’s the important way to prevent the spread:
1.treat the staff and eradicate
2.isolation
3.wash hands

55.pt had consvulsion took rectal diazepam,iv lorazepam by paramedic


What to give:
1.phytoin blous
2.phytoin infusion
3.phenobarbitone bolus

56. Mother with HIV what to give the baby:


1. ziduviodine

57. Indian pt. came to UK, complain of tiredness, pale, spleen 2cm,
hepatomegaly 2cm, no facial features with/out Jaundice what to do:
1. CBC+ blood film
2. Electrophoresis Leukemia

58. Infant born 2.2 now one year on the 2nd centile what to do:
1. Previous measures

59. Appendectomy pt,refuse to eat ,tachycardia,CRT 3 what to do:


1. Saline bolus
2.glucose bolus

60.female 13 years old,down Syndrome had abdominal bloating,diarrhoea,had


her thyroid function checked last year was normal ,her mother abandoned her:

Neglect child
All FOP Recall till June 2021 with note 131 of 378
Child known with hyperlipidemia in the family
what to do ?
1. If child < 5 y —> see the father
1. celiac 2. If child > 5 y —> check lipid profile and
2. chron’s cholesterol level
3. If become competent —> do genetic
3. Hypothyroidism screen

61. Girl 10 year with all males in the family diseased, and a dead uncle
1. Check her cholesterol level If mother has ovarian cancer what to
2. Wait until she is competent screen the baby —> wait until she
competent
63. 12 week old baby not following his mum what to do:
1. Refer to Ophthalmology

64.2year old no words,not interacting with anyone what to do:


1.refer to child development team
2.refer to speech therapy For hearing assessments

65.edemtous child ,malnourished


What to give:
cow milk formula for Kwashiorkor ,, unless
1.cow formula child not shock don't give IV

66.girl came to ER with her mum’s partner with pain ,has to go to emergency
appendiectomy,the mother is away who is going to consent:
1.the partner
2.the girl
3.mothet by phone

67.pt came with eye swelling in the eyelid ,painful eye movement what’s
treatment:
1.amoxcillin oral
2.iv ceftraxione

68.pt was clumsy,enjoys playground,no one wants to invite him to their house.
1.ADHD
2.autism

69.pt transferred from small primary school to big secondary school,became


aggressive and developed learning difficulties :
1.educational therapy
2.home tution teaching
3.family therapy ‫عليه خالف‬

70.

All FOP Recall till June 2021 with note 132 of 378
FOP exam June 2019

EMQ1:palitive
decrease morphin
senna
continuous SC pump
hyocin patch
movicol
metochlopromide
diclofenac Na
domperidone

1/-3 senatioes palliative ( all taking oral slow acting morphine with breakthroughs )
First case uncontrolled pain in end stage cancer,,need to increase dose to more than 8 time ……
1/continuous SC pump

2n/ controlled pain with difficult swallowing …


.hyoscine patch ( ‫)عليه خالف يحتاج مراجعه‬
3rd controlled pain + constipation for 3day
…movicol
N.B no fentanyle patch in choices

2EMQ- -cardiology :

1/Classical VSD PAN SYSTOLIC murmer =VSD If apex , MR

Neonate with pounding pulse murmer continous =PDA


Ejection systolic murmer in 3 month well infant in Lt intercostal space ….= ASD
In the option severe PS
3-EMQ musculoskeletal :
A-3yr with limited external rotation and limping afterrecent URTI On antibiotic prophylasis there also
history of VUR with recurrent UTI …..transient synovitis “ Sample papers active arthritis

Destructive
B- 12 yr well growing with hip limited painful movement after exciercise BMI 91”( not obese )…SUFE

C-swollen red hot joint joint after chicken pox difficult to pear wt “….. septic arthritis
N.B no reactive arthritis in choices

4EMQ-neonatal jaundice : (asking about diagnosis )


spherocytosis
G6PD
physilological
ABO
CMV
RH incombitability
prolong physiological jundice
glactosemia

All FOP Recall till June 2021 with note 133 of 378
1/6 hrs Jaundice with HSM 7cm spleen liver 2cm,,with peatecheal rash,,comb -ve…..Cmv
2/Jaundice in 18 hour neonate mother o+ baby A+ coomb +ve ….ABO
3/Jaundice in 38 h female neonate baby well ,,FH of neonatal jundice need phototherpy….
physilological,,spheocytosis (there is debate) Spherocytosis in first day of life
But spherocytosis peak of jaundice in 2nd day, mandatory
all unconjugated hyper bill phototherapy

5-Emq toxicology how to treat :


A-Girl take Methadone over dose …..naloxone
B-Girl take paracetamols but refuse iv line …oral methionine
C-Take Alot of drink in party deny drug came after went home because of confused and sweaty
,,pale….give== glucose See SOP
alcohol toxicity treatment
* hypoglycemia —> give glucose
6-emq * RF —> ventilation support
lead * alcohol level
iron
3 senarios all indian has microcytic hypochromicanemia : ( asking about diagnostic investigation )
1/Healthy child + vegetarian + was breast fed up to 6 months cow milk wediling gate and bow leg
,,Delayed…..Ferritin v/s vit D Rickets —> vit D deficiency

2/Indian with HSM ….Electrophoresis


3/years eating healthy mixed food + iritability and WT loss 0.4 Centile …Anti tissue transglutimenaze
Celiac disease
7-EMQ/3 scenario about bloody diarrhea (same as TAS sample paper ):
A-low Hb and platlets …..HUS Diagnostic blood film ,, scinario of HUS , next step serum creatinine
B-Painless bloody diarrhea + well child …colonic polyp
C-8 years old + 6 month history of bloody diarrhea + lethargy +wt loss …chrons

8-EMQ 3 senario about investigation unresponsive child :


CT BRAIN
TOXICOLOGY Another scenario trumatic head most probably extramural
asprin

A- 12 year boy found unresponsive in his room with unilateral dilated pupile…..CT
B- 14yrs Girl un responsive with bilateral dilated pupil reactive & tachycardia…..toxicology screen
C- pt was having shalow irregular resp ==asprin
9EMQ resp:
antibiotic
salb inheler//with spacer
erythromycin
o2
budsonide
oral dexamethazone
adrenaline neub
1/ 18 month first attack wheeze,spo2 89%,==o2
2/ 2 yrs stridor,,==oral dexamethazone
3/ normal sat 98% difused wheeze =salbtmole inheler

All FOP Recall till June 2021 with note 134 of 378
EMQ10:DERMATOLOGY
intralesion sclerotherpy
silvernitrate cautarization

See AKP CC 42
1/ baby with molusim contagusum teased at school=silvernitrate cautrization
2/haemangioma on eye difficult to open eye=propanolol
3/eczemas skin dry his brother has eczema with dryness in the back=emolient

1st. Emollient
2nd. Steroid with potency according to severity
3rd. Tacrolimmus if > 2 year

Best of 5

1- Sudden difficult breathing


Shifting trachea Rt side
Decrease air entery lt side
After staph pneumonia(pt cystic fibrosis)
—>
1/Pneumothorax
2/Effusion
3/Empyema

2-Scenario facial(preorbital) swelling and edema in foot and large abdomen,,(skin rash history??)
anphylasis
nephrotic
hart faliure
preorbital cellulitis

3-22monthChild healthy eating well on 25 centile I think , diarrheal 3-6 time per day episodes with food
particles :
1/Giardiasis Decrease wt,
2/Toddlers diarrhea
3/Celiac

4/inspirotory stridore increase during feed and sleep and apnea (SINCE BIRTH)
1/laringomalacia
2/GOR

5-Murmer diastolic ,,clubbing ,, fever splenomegaly ,,miserable


Infective endocarditis
Kawasaki
Rheumatic fever

All FOP Recall till June 2021 with note 135 of 378
6/ conjectivitis receive chloramphilocol drop not improve ,,came with cough
1/clarithromycin oral
2/iv ceftrixone Chlamedia usually associated with cough
3/amoxycillin oral

7-On chemotherapy
Brother chicken pox
Give vzig and acyclovir when rash
IVIG
acyclovir See guidelines —> high risk —> give prophylaxis oral acyclovir

8-Scenario about
Tumour lysis syndrome Leukemia ,Low HB 68 ,High wbcs and High k ,,oncology team advice to start
alloprinolol
What to give :
1/Blood transfusion
2/Iv saline

9-Boy going to India took antimalraial proph,, after coming back ,, pain in throat , enlarged cervical
lymph node 3cm, HSM.,,constipation.. + jaundice.
1/ IMN
2/Malaria
3/heptitis A

10-Girl height 0.2% centile and wt 25% , mild learning disability


Next
Karyotyping Ear crease

11-.senario descrip. bekwithwedman ... large tongue , earcrease,,decrease wt 12% wt now 4.1kg
Next investigation ?
Glucose

12/polyurea polydepsia +wt loss Guidelines 81


RBS
OSMOLARITY
GTT

13-Boy 12 hour with very small phallus and bilateral un palpable gonad no dysmorphic
features,,NORMAL feed
No hypotonia what diagnosis :
CAH Low steroid —> hypoglycemia
Prader willi
Klinfelter syndrome
1. US and karyotype
2. U/E
3.

All FOP Recall till June 2021 with note 136 of 378
14-day time symptoms , nocturnal bladder instability enuresis still voiding on cough or exercise
Treatment
1/Oxybutinin Anticholinergic
2/dessmopressin ,,
3/ametryptaline , If nocturnal enuresis
4/enuresis alarm and star chart

15- 12 year girl with 4 weeks history of wheeze, shortness of breath.


No improvement after salbutamol and steroid inhaler, no history of asthma
History of weight loss next step
1/check inhaler technique Malignancy
2/chest x ray
3/CBC with blood film
4/echo

16- 19 month Girl bottom shuffler scenario ,,not walking ,,mature pincer grip ,,building 3 cub
need(normal up to 24month)
1/ MRI
2/REASSURANCE
3/Ck

17- amber 3 ,, signs in 5 years old child ,,tachypnea ,,tachycardia fever ,,decrease consiousness
What investigation to do(guideline)
1/CBC,urine c/c .,,,
2/CBC ,,crp,,urine c/s,,xray
3/ CBC ,,crp,,urine c/s,,xray

18-Picture of child with rash (vesicles and crusted lesions on the trunk , face and LL )
history for eczema and mother has painful mouth lesions recently:(picture)
1/eczema herpeticum
2/Impetigo
3/Chicken pox

19-Gentamicin over dose by nurse—next ?


1/Inform parents and tell them the investigation going on 1. Safety
2. Inform parent or patient if competent
2/Inform parents and the nurse is responsible for 3. Inform senior
3/goverment 4. Relative of other patient
5. Pharmacy
4/dont inform 6. Incidents report

20-12yrs Boy follow up for chrons


Told nurse he had new anal ulcers and dont want parent to know
—encourage to tell parents and keep confidentiality

All FOP Recall till June 2021 with note 137 of 378
21- 9year no constipation soiling and leukostrase++ in urine , recent mother partner with his son
15years ,,stay alway in room
1/sexual abuse
2/UTI

22-Constipation with soiling in 5 years child ,,since start school also at home
-1/fear of school toilet
2/Constipation
3/sepration anaxity
4/Child abuse

23- Scratch around anus at night /red margin around anus painful defecation (it was acute symptom
with in 24h)
1/ theadworms
2/Sexual abuse
3/group A strep
4/staph
‫عليه خالف‬

24-Facial Eczema with LN 1.5 cm ,,soft ,,mobile


1/review after 2w
2/excesinal biopsy
3/review after 3month

25- Umbilical hernia in infant


Mother said he is crying thinking he is in pain Size 1.*1.5 reducible easly what your best advise :
1/Wait for closure
2/Come to ER if vomit
3/Surgical opinion
4/reduced by coin
5/reduce manual

26-migraine in child ??
1/Usually unilateral in adult Usually bilaterally in children
2/Family history
3/Rare under 5 years
4/Common visual aura Common no aura The classic is with aura, but the common without aura

27/4wk unilateral testis palpated at inguinal canal, And other side in scrotum
1/Review 3m
2/Review 6m
3/Review 12m
4/refer to surgery

Retractile testis * Definition ; any testis palpable in inguinal canal and can be pushing the scrotum.
* Management ; need to review annual, till puberty, if can not be pushed in scrotum —> refer to surgery
* referral time 1. If cannot pushed in the scrotum
2. If reached puberty

Which age if testis still in inguinal canal , then unlike to descend, at age of 3 months

All FOP Recall till June 2021 with note 138 of 378
Parent concern
Fixed squint, any age need to refer
28- 4m squint refer to ophthalmologist More than 3 month apparently squint, need optha refer

29-Fracture tibia 5yr on pain despite regular paracetamol and brufen next?
1/Oral morphine
Fracture
2/Codiene Spiral fracture of humerus
3/Diclofenac Metaphyseal fracture of femur
Posterior rib fracture
Callus formation > 42 days
30-To whom skeletal survey should be done : Fracture of clavicle in age > 1 month
Spiral fracture of humorous in 4 month infant Skin
Spiral fracture in tibia 2 years child Oval shaped burn
Spiral fracture of femur in 23 month child Bruises behind ear, pinaa bruises
Human bite
Frenulum tear
Cigarette burn
Retinal hge more than 42 days
31- Diabetic conscious hypoglycemic 2.5
Sweaty twitches
Next ?
1/Oral 4 glucose tablets See 19 cc
2/Orange juice and slice toast
3/IM glucagon
4/Iv glucose 10%

32-Baby 4 week dehydrated loss 12% of birth weight breast feed on demand
Na 150 next ? Hypernatremic dehydration
1/Frequent regular feeding See guidelines, neonates, by syringe, cup, —> if not tolerated, ngt —> if not
2/Ivf tolerated, IVF
3/Formula feed
4/Ivf then formula

33-Child 4 mouth with bleeding from noise and brussis and petichue already in under child protection
Came with this finding ,,with upper resp infection
1/Child abuse Petichue character of ITP
2/ITP

34-Accident GCS 8 and abdominal bruised at upper abd start deteriorate conscious
Tachycardia 170/MIN
Next ?
1/Us abdomen
2/Ct brain
3/Cannula
4/Call anathesiest As PALS ABC start by appearance If appearance comatose, then intubation
5/nasophargeal tube

All FOP Recall till June 2021 with note 139 of 378
35- Unstable hip in 2week neonate referred to pediatric orthopedic in next day and mother ask what will
be his advise * < 3 mo. —> by pediatric , double diaper
1/Abduction splint See guidelines —> by surgery , abduction splint
2/Abduction traction * 3-6 mo. Pavlik harness
* 6 mo. - 2 yr —> close reduction
3/Double nappies * > 2 yr —> open reduction
4/open reduction * > 3 yr —> femoral shortening
* > 4 yr —> acetabulum surgery

36- age 11yrs breast srage 3 ,, obesity screening .. girl .. wt and height (>98%) centile BMI 91% BMI( 30 )
What is the best management Fop case = overweight management is life style
1/Wt mangement and exercise & life style Medication used only if BMI centile > 98 %

2/Statins Can be given after failure of life style


3/Lipid profile and LFT

37- 28 week loud murmur Cannot wean from mechanical vent. What to give PDA
IBrufen Guidelines
1st course —> ibuprofen
Prostaglandins 2nd course —> ibuprofen
Indomethacin Not given now 3rd course —> paracetamol
Last —> surgery

38-collapse after swimming , + family history uncle in follow up with cardio what diagnosis
1/long QT
2/HOCM

39-female need prophylaxis for meningitis , on ocp ?


Cipro Can give to all type population Cipro is given one dose for all populations in
In H influenza —> rifampicin daily for 4 days
40-large planter wart, Human papilloma viral
1/Not swim Can swim but cover lesion
Mulloscum contagiousum can swim anytime
2/refer to dermatologist? Refer to GP If fail refer to dermatologist But human papilloma viral = wart only if wear
3/Put water proof plastic water proof plastic then can swim

4/not swim until releved spontinously


Not swim 5/not sleep until response to treatment
‫عليه خالف‬

41- 7 year old car accident no loss of consciousness observed for 6 hours and discharged what's the
most complication Subdural hge in baby shake syndrome
1/Post traumatic stress Subarachnoid hge in AVM Stiffness
2/Extra dural hge Middle meninges vessel Artery tear
See illustrated
3/Necrotizing fasciitis , type of hge

42/ there was unilatral painful firm swelling after trauma


1/heamatoma
2/scrotal edema
3/tumor
4/hydrocele

All FOP Recall till June 2021 with note 140 of 378
43- Mother nervous worried because family history aunt of hypothyroidism with mental retardation -
TSH high 1st day neonate was done next ? First day reflect mother hormones
1/T4 to confirm See neonatal guidelines
2/regular blood tests,,
3/go for thyroid screening on regular date 5-7 days..
4/need urgent iso top scan
TSH < 10. —> no action
> 20 —> hypothyroidism
10-20 border line —> need to repeat < 10 —> no action
> 10 —> refer to consultant
44-high TSH ,no uptake by scan ,
1/ thyroid aplasia
2/Lingual thyroid
3/dyshormonogenisis If there is goiter TSH & T4 low

45 –p 32wk 5 month old infant with history of antibiotic in NICU and nicu,, pass screening hearing what
test to do now —>
1/ABR(aut brain stem response) Auditory,, Check hearing pathways
2/Distraction test
3/Visual reinforcement
4/outo acoustic emission Newborn Screen

46- ronaldic epilepsy drooling and gurgling during sleep parathesia in jaw < what to do
1/need EEG
ttt ; reassuring then carbamazepine
2/ca

47- ADHD with methylphenidate difficult writing association and in wearing his clothes —>
Dyspraxia Coordination disurder
Dyslaxia
Side effects of drugs

48- case with investigations


Ca low
Ph low normal
PTH low Correct high PTH
Alp high
25vit d very low
Treatment ???
1/One alfa
2/Vitamin D
3/Calcium
4/Calcium and phosporus

49- anaphylaxis Senario on a party eating cake. What to give


1/Adrenalin 150 micro im See guidelines
2/Iv hydrocortisone
2/Antihistamines

All FOP Recall till June 2021 with note 141 of 378
50- child frightened at night , crying , recurrent attacks couldn’t remember What happens next day :
Night terrors
Night mares
Seizure

51 – 5 month baby with typical uti ( Ecoli respond to antibiotic within 48 hourthe next to do :
1/ us after 6 weeks within 6 weeks
2/US now

52- iv salbutamol side effects > S/E; hypokalemia, hyperglycemia, lactic acidosis, tachycardia
1/lactic acidodosis
2/Hypo NA
3/Hypo ca
4/hypoglycemia
6/metabolic acidosis

53- Peeling flexures ...crust around eye and mouth ..no mucousal membrane affection -ve c/s from side
of bulae what diagnosis :
1/SSSS
2/Erythema multiformis Target lesions
SSSS; = Ritter syndrome, ttt by antibiotics
3/Toxic shock syndrome Effec of toxin, blood cultures negative, sterile ,,,, impetigo positive
4/epedermolysis bolosa Blisters
5/steven jonson Mucosal affected

54/

55- Girl 6 y with breast enlargement breast 2 pupic 1 axilary 1 obisce child >>
thelarche ...

56- Neonate vaginal bleeding bilat non painful breast enlargement what ur best advise to the mother :
1/It’s normal finding
2/It’s due to medications taken during pregnancy

57/

58-

59- BCG vaccine site


Intradermal Lt arm at insertion deltoid
Intradermal lt between shoulder and insertion of deltoid
Intradermal Rt arm at insertion of deltoid

All FOP Recall till June 2021 with note 142 of 378
60- 15 yrs came in septic shock.... For emergency...... the nurse tell you that she didn't take consent for
canula insertion.. What you will tell her :
1/insert after senior advice
2/take parents consent
3/ check if she competent for consent
4/Proceed without consent

Regardless of age, emergency treatment to save life or


prevent deterioration can be given without consent, source
RC Emergency Medicine

61- female student 18 years pregnant 8wk with cystic fibrosis ,,husband not affected with no family of
cystic fibrosis she want to continue the pregnancy ..wants to know if the baby will be affected ..what
advice ????
1/Wait for neonatal screening
2/Screen father for most common mutation of cystic fibrosis.
3/Amniocentesis
4/Cvs
5/DNA after dilivery

62- Scald ,Which indicates abuse : See sop


*Regular margins and equal depth lesion.
*Circumferential
abcent of splach mark

/63- GE vomiting 2times at night and one in the morning before giving insulin dose
Glucose 6 normal value given (5-9 )
Normal
Ketone +
1/Continue same dose ,Give sugary drinks and monitor glucose and ketones
2/Reduce dose and give sugary drinks If no oral intake
3/shift immediately to hospital If bms < 4 , no oral intake
4/continue same dose and encourage food and drink

64- Girl paracetamol toxixity 10tablets below level


Dysuria refuse eating in morning Hint of pregnancy Refuse eating in the morning dt. Nausea
—>
1/Ask if pregnant
2/Do hcg
See SOP —> usually pregnant
3/Refer outpatient CAMHS teenagers , more liable for depression
4/Allow to go home with suicidal attempts

5/

Any suicidal child —> refer to


psychiatric within 24 hours , if
discharge follow with CAMHS as
outpatient

All FOP Recall till June 2021 with note 143 of 378
Gower sign

65-11yrs boy go school with his sister she noticed that he had difficulty getting into school pus he had
viral Illness 1month age he had also complain of tiredness since last year wt diagnosis :

1/chronic fatigue syndrome Common in girl , should be more than 3 months symptoms
2/Becher muscular dystrophy body ache
3/-duechen muscular dystrophy Less age

66- 3yrs case of male child with delayed walking , as symmetrical muscles what initial investigation :

1/CK As case fop any delay walking—> do CK


2/DNA analysis
3/dystrophin

67- case about history of meningitis with convulsion for 10 min (not convulsing now)first ttt
1/iv lorazepam
If * focal neurological sign
2/iv antibiotic * Seizure
3/iv saline * GC scale low ——> give acyclovir with antibiotics
4/antibiotic +acyclovir

68- infant with URTI feverish 38.5


with history of febrile convulsion but more than 5 minutes( 7 minutes ) what best advise to tell her
mother if it occurs again

1/Buccal midazolam Go to hospital Illustrated if prolong > 5 minutes See guidelines

call ambulance, if febrile convulsions

69- another case of febrile convlesion,,recurent ,,known case


1/oral brufen See SOP
2/regular paracetamol * regular fluid to avoid dehydration
* ibuprofen faster action and duration
3/plenty of drink * also paracetamol and ibuprofen will give if child destress
dt. Pain

70- case of CNS infection , shocked (tachycardia + CRT 4 seconds ) what to give :

Iv antibiotic
Iv saline

‫اللهم تقبل منا هذا العمل خالصا لوجهك‬

‫بالتوفيق والنجاح للجميع‬


Way to success

All FOP Recall till June 2021 with note 144 of 378
FOP February 2019
BOF Puff

1-3 year old child on steroids pump 100 microgram


BID per day still having cough during sport and night
cough .what will be your next action?
steroids 200 mcg twice a day
Add of Leukotriene
LABA
answer: Leukotriene
2-patient 3 year old from Asian country received
BCG vaccination came for he has mass in neck for
last few week with discharge and discoloration of
skin otherwise patient is vitally stable?
Active TB Both Atypical mycobacterium & infected bronchial
cyst fluctuated —> do US
* Atypical mycobacterium is solid
Atypical mycobacterium TB * infected bronchial cyst fluid
answer: Atypical mycobacterium TB See survivor 62
3-One child was treated one week ago for Acute
upper respiratory tract infection mother noticed
mass in neck and occipital region splenomegaly
what could be dx

All FOP Recall till June 2021 with note 145 of 378
A Infectious mono nucleosis
B Non Hodgin Lymphoma
C Lymphoma
answer: Infectious mononucleosis
4- One child in school having difficulty in sports and
he cannot hold pen in good grip otherwise learning
is okay where will you refer. The problem is coordination
A Physiotherapist
B occupational
C neurologist No learning difficulties

answer: neurologist?? Occupational


5- Patient complain of body and leg pain, on exam
he has swelling on lumber region ?
Spinal tumor Pain in leg and. Body with weakness in arm and leg

answer: Spinal tumor??


6- Patient ℅ leg pain since few week but now
parents notice having echymosis on back and on
exam he looks pale and having hepatosplenomegaly
answer: ALL
7- Girl present with paracetamol over does not toxic
dose and multiple scratch marks on her hand the
what to do? Suicidal attempts

All FOP Recall till June 2021 with note 146 of 378
-obsarvasion for 4 hours then discharge
-dicharge as it's not toxic dose
-CAMHS ?? CAMHS need consent for referral

-admission
answer: admission (100% psch)
8- A refugee child from Syria with night sweats ,
thin, malnutrition, respiratory signs .. diagnosis??
TB
PCP
answer: TB
9-Girl 3 years Labial fusion .. management?
Do 17 ohp
surgery referral
topical estrogen If started then after stopped —> recurrent
answer: reasurance(sop 401) Labial fusion —> spontaneous regression
10- 2year Girl with abnormal posture occur with
stress or cry ,symptoms “see-saw” at the beginning
of sleep? hypnagogia ?
Self gratification
answer: Self gratification
11- girl was circumcised in Sudan
Police

All FOP Recall till June 2021 with note 147 of 378
FMG
Social worker
answer: police
12- 3 years girl Exposed to meningococci
answer: cipro
13-Anorexia nervosa with social withdrawal
eating disorder hr low IfIf anorexia nervosa has withdrawn —> depression
anorexia nervosa no social withdrawal—> eating disorder

depression disorder
answer: eating disorder vs depresion
14-Down syndrome 11 yr asymptomatic playing
football short stature without previous follow up?
Cervical X-ray
Thyroid function test
Blood film
Celiac screening If symptomatic
answer: thyroid function test
15-A child with ALL in remission on maintenance
chemotherapy medication. child went to 350 mile
trip and parents were forgotten to bring medication.
You spoke with tertiary care treating hospital nurse
over the phone about details of the medication.
What is the next plan?

All FOP Recall till June 2021 with note 148 of 378
Ask the consultant
Request urgent previous plan
Start the treatment
answer: Request urgent previous plan
16- 3 wk bw 3.4 current 3.7 feeding 125 ml 6 time
per day with vomiting
Overfeeding Should only 150 ml/ kg: day

GOR
answer: Overfeeding =TI 202ML/KG/DAY
17- 3 rd day baby bilious vomiting
Admit and investigate ?? Malrotation
answer: Admit and investigate
18- Sure sign of abuse 4 month?
ear bruise
answer: ear bruise
19- baby with bruises in the cheek the mother has
difficulty with breast feeding they came on friday
night what to do? Child abuse

discharge and come back on Monday


admit
social service
bleeding profile

All FOP Recall till June 2021 with note 149 of 378
answer: admition(answer of person with 100% in
ethic)
20- 13 ye GCSE exam dysuria headache nausea ?
Trimethoprim and tack culture
Broad spectrum anti biotic and go to exame
Miss the exame and admit
answer: ?? Broad spectrum anti biotic and go to
exame

21- Q ask about consent 18 year ; can refuse ttt


13 year and over if competent 16 year ; can agree ttt
< 16 year ; if competent
16 year and over if competent
Any age if competent If 11-12 year if competent

answer: Any age if competent(100% in ethics)


22- When to refer baby?
9month old with hand preference More problems dt CNS
When to refer child to hospital
40C
Reduce skin turgor
answer: 9month old with hand preference(should
not be less than 1yrs)

All FOP Recall till June 2021 with note 150 of 378
23- weakness on L side of face and was able to
wrinkle that side of the face. Improved slightly with
steroids?
Discharge with GP follow up
CT
MRI
Neurologist referral Facial palsy, spare upper —> UMNL

answer: MRI ?? More visualize midbrain

24- 1.5 month dry cough ,,vomitting afer cough(clear


cenario of pertusis)
Erythromycin After persistent cough —> cyanosis,
apnea, vomiting, red eye, petichae

Amoxicillin
answer: Erythromycin
25- Girl 12 yr chest clear RR 45 spo2 99 hart rate
normal with tingling ?
Hyperventilation
answer: Hyperventilation(tacypnia with out
tachycardia)
26- Feeding jejunostomy low GCS,, NA* 190
Fabricated
GE Other scenario CP child , no body want to
care
Salt poising Child symptomatic by low GCS , Na high dangerous

All FOP Recall till June 2021 with note 151 of 378
DI
answer: DI?
27- question about eye complaints not improving
after chloramphenicol For staph or strept
chlamydial Infection ..
Neisseria
answer: chlamydial Infection
28- 23 wk decesion of resuscitation <purposes
22 wk —> for research

22 wks & 6 days —> dnr, only if

Parental wish parent wish, so first discuss with


consultant
23 wks —> parent wish before
According to the baby condition delivery —> CPR according
baby condition

Poor out com


answer: Parental wish
29- Headache neck rigidity?
Subarachonoid hae
answer: Subarachonoid hae
30- bicycle handle hit the upper abdomen..10hrs
later vomiting and tachycardia started? Shock
pancreatic injury High WBC and serum amylase
subdural hematoma
answer: ? Pancreatic Handlebar injury like riding bicycle —> pancreatic

31- Small for gestational age pulmonary stenosis


absent red reflex ?

All FOP Recall till June 2021 with note 152 of 378
Reubella
answer: Reubella
32- Generalized mild hypotonia child with
handwriting and spelling difficulty?
Occupational therapy
neurology
answer: neurology??

33- Neonate has tachycardia and tachypnia


Hepatosplenomegaly and anemia
answer: TORCH
34- 12 yr female sesonal rhinities before exam
Cetrizine same day evening
Xylometazoline same day as needed
answer: Cetrizine same day evening
35- small VSD with plan for tonsillectomy
no prophylaxis need Annual Allergic rhinitis more

chlorohexidinmouth wash
sever with hyphae, ttt by
cortisone

erythromicn
cardiologist opinion
answer: no prophylaxis need

All FOP Recall till June 2021 with note 153 of 378
36- was 4 wks of fever, headache, malaise, rigors etc
plus spleenomegaly and grade 2 diastolic murmur?
IE
RF Rheumatic fever
malaria
answer: IE
37- Acute rheumatic fever to rule out or in ?
ESR
ASO
ECHO
answer: ASO
38- scenario eye swelling with protrusion , fever , bp
140/85 Diagnosis ?
- hyperthryroid
- neuroblastoma
- langerhans histocytosis
- orbital cellulitis
answer: neuroblastoma
39- Hyperthyroid girl on carbimazole tremor
palpitation ?
Propranolol
answer: Propranolol

All FOP Recall till June 2021 with note 154 of 378
40- Boy with cystic fibrosis, what is the chance of
carrier for his sibling?
2/3
answer: 2/3
41- Cervical lymph node with skin discoloration
over it and non tender
Diagnosis?
Mycoplasma
atypical mycobacterium
answer: atypical mycobacterium
42- 15 year Fever night sweat hepatosplenomegly
lymphadentopathy?
brucelosis
NHL Wt loss, night sweet, fever, common in UK

TB No cough in scenario
answer: ??
43- Hx of tonsillitis a month ago with palpable
cervical swellings 0.5mm?
reactive lymphadenitis
answer: reactive lymphadenitis?
44- severity of dehydration ?
Dry mucous membrane

All FOP Recall till June 2021 with note 155 of 378
wt loss wether it is 5% or 10%
prolonged capillary refills time
skin trugor
hypotension
answer: wt loss wether it is 5% or 10%
45- Worse sign of hepatitis?
pt prolonged
answer: pt prolonged
46- 9month old baby with birth WT of 3.7 kg..now,
FTT with WT below 0.4th centile.. basal crepts with
distress? Heart failure
PCP All lung crepitation
VSD
answer: VSD
47- case of DKA CR 2sec heart rate 120 PH 7.18
..what to give immediately ? Child dehydration but not shock
NS .9%
10ml/kg bolus
answer: 5% maintenance Correction, stain
48- Meconium aspiration newborn prolonged labor
36 hours PROM at second stage of labor?
Admit and start antibiotics No red flag , one risk —> observed

All FOP Recall till June 2021 with note 156 of 378
suction observation
antibiotics and observe
answer: observation
49- 13 years old girl moaning and closing her eyes
throughout?
Pseudosizure
answer: Pseudosizure

50- A mother is having difficulty feeding her child


and believes that you can not help her
Breast feeding team
answer: Breast feeding team
51- 15 y old migrant Fever+ petechiae+ palatal
petechia splenomegaly+ fever ?
Ebv
answer: Ebv
52- blister on buttock and redness around
umbilicus?
staph aureus
group A streptococcus
staph ulbus
E choli

All FOP Recall till June 2021 with note 157 of 378
haematuria
answer: staph aureus
53- Periorbital oedema after 1 week of diahrrea
hemoloytic anemia and thrombocytopenia pale. He
want investigation? HUS
IgA
Antestreptolysin o titre
Peripheral smear
answer: Peripheral smear
54- maculopapular purperic rash and arthritis
JIA
HSP
answer: HSP
55- patient on prophylaxis penicillin then develop
dental cerise ?
start another antibiotic
liase with dentist
suger free pinicilin
no treatment
check if the penicillin contain suger
answer: suger free pinicilin

All FOP Recall till June 2021 with note 158 of 378
56- Question about intermittent inguinal swelling
increase by cough?
Inguinal hernia
answer: Inguinal hernia
57- extensive eczema on the face , on hydrocortisone
mother woried about corticosteroid use on the face
- tacrolimus ointment ( 1% )
- oral steroids
- oral criprofloxacin for 3-6 month
- oral antihistamine
answer: tacrolimus ointment .3%(Over 2yrs age)
58- unvaccinated child who developed a fever,
recovered, and then developed a rash across his
back?
Roseola infantum
answer: Roseola infantum
59- Diabetic child on insulin pump presented with
DkA Not shocked But severe acidosis
Unwill, abdominal pain and vomiting
What is the next step ???
1- shot saline 10 ml/kg. Not shock infusion

2-maintainance fluid with deficit 5%

All FOP Recall till June 2021 with note 159 of 378
3-maintainance fluid with deficit 10%
4- start insulin infusion ,05 u/k/h.
5- increase dose of insulin pump
answer: ?? maintainance fluid with deficit 10%
60- And also small erythematous patch in the
forearm Doctor thought it is viral origin and
reassure the child After 1 week Pain and fever still
the same And the patch increases in size?
borrelia
Mycopoasma pneumonia.
answer: borrelia = Lyme disease

61- There was also a Q about child coming with calf


mass. Bruising after playing football with brother
History of clumsy walking. And delayed walking
with avoidance of sports in school , father also I
think had similar condition normal apgar score
_cerebral palsy
_hemophilia A
- Ehler Danlos
answer: Ehler Danlos
62- Girl with type 1 dm. On basal bolus regimen
with poor control. Hba1c given was 7.9% .there was

All FOP Recall till June 2021 with note 160 of 378
stressful condition at home Parents heard her
screaming and saw her fitting and sweating what is
the cause
- Night terror
- Hyperglycaemia
- HypoglycaemiA
answer: Hypoglycaemia

63- 4 years , full time in nursery youngest of 6


brothers , teacher concern from uncharacteristic
behavioral disruption , no concern at home
- clinical psychology assesment
- audiology assesment
- visual assesment
- developmental
- psychometric test
answer: audiology assesment
64- 7 year old boy with 100mcg fluticasone
Which step in asthma ?
- Leukotrine antagonist
- increase 200mcq fluticasone
- LABA

All FOP Recall till June 2021 with note 161 of 378
answer: - LABA
65- 43- 4 or 5 month not sure of age , breast fed ,
well baby , found to have HGB of 58 what to do ?
- blood transfusion
- oral iron
- change milk
answer: blood transfusion (pedia guide line )

66- 9 year girl have family issues ( step father come


to home ) , argue with her mother and doesn’t want
to get out of bed , then when she got out turned pale
fall down with twitching of hand , then wake up
after 2 minutes and she don’t remember except she
was dizzy ?
- reflex anoxic
- complex partial
- pseudoseizure
- ! syncope
answer: - complex partial

67-Unilateral wheeze
Rigid bronchoscope

All FOP Recall till June 2021 with note 162 of 378
Answer: Rigid bronchoscope
68-Case about plegocephally
answer:reasurance
69- Q about croup
70- Obese micropenise
answer: Normal obesity

All FOP Recall till June 2021 with note 163 of 378
EMQS
1
Diclofenac
Iv morphine
Intranasal diamorphine
Femoral nerve block
Ketamine
Nothing (as for the overuse headache
1-A child with burns, parents washed with cold
water for 15 min, now child had hypotension with
cool peripheries Severe pain interfere with daily activity 8-10

What is the drug of choice for pain management?


answer: Iv morphine Intranasal morphine or diamorphine as more
easy in ER, if no hypotension

2- 14 year old boy met with RTA had head injury


followed by swelling gradually progressive and child
also got right femur fracture. Now orthopedician
want to manipulation and put cast , so what next
management for pain management?
answer: Femoral nerve block

- Opioid lead high ICP in head injury

All FOP Recall till June 2021 with note 164 of 378
2
Discitis
transient synovitis(common in pedia after urti)
reactive(occur after git & gento urinary inf less
common)
Perth's
1-7/8 year old girl previously fit and well suddenly
unable to weight bear. Examination of lower limb
neurology is NAD. h/o upper respiratory tract
infection few days ago. O/E tenderness over lumbar
spine region remaining all examination was normal
answer: Discitis
-Leg pain at night If not awake benign , but if awake for pain malignant
answer: Growing pain

3
Atypical mycobacterium
TB

All FOP Recall till June 2021 with note 165 of 378
Rubella
ebv
ALL
- scenario generalized lymphnodes , splenomegaly
answer: EBV
- 4 year non tender LN with color change above it 1
node cxr clear- long period
answer: atypical mycobacteria
- tonsillitis one month back , 0.5 nodes ( multiple ) in
the occipital area and posterior triangle
answer: reactive lympaded
4
Choose suitable investigation
CRP
ESR
Amylase
fecal elastase
Anti-tissue transglutaminase
H pylori stool
H pylori serology
All diabetic patient and complain of

All FOP Recall till June 2021 with note 166 of 378
- scenrio with peri-umbilical pain and abdominal
distention , constipation .
answer: Anti-tissue transglutaminase
- scenario with intermittent epigastric pain , poor
apetite , wake at night , wt loss
answer: H pylori stool
- scenario with right iliac fossa pain and mass ,
diarhea for 6 weeks
answer: ESR

5
EMQ
-Baby with seizure stop by touch
answer:Rigor
-Crying+ apnea Other non seizure, breath holding
Painful stimuli or vomiting vaso cardiac stimulation
answer:Reflex anoxic vagal attack lead to pallor dt brain anoxia—> may lead
to seizure

-Neonate came after six hours with milky stool


answer:Serum billirubin
-Neonate lethrargy
answer:Hypoglycemia
6
-11 year girl breast bud developrd 2 year ago

All FOP Recall till June 2021 with note 167 of 378
answer:Normal
-Case about premature adrenarch
-8 years with breast and hair
answer: Central (tumer)
-Yellow card
answer: Penicillin allergy
redbook=follow up of growth(34 fop??)
Referral child
answer: Skin turger
-FTT with hight and weight bellow second centail
answer: Recurrent chest infection May cystic fibrosis
note:
tansient synovititis 3-8yrs common cause of
nontrumatic,, history of urti,,predomonant in
male,,48hrs of acute inf
reactive arthritis= 7 to 14day after acute inf,,git &
uti inf

All FOP Recall till June 2021 with note 168 of 378
FOP OCTOBER 2018
EMQ-PALLITIVE
Im morphine-intranasal dimorphine-syrup morphine-PCA
PUMP-FENTANYL patches-rectal diclofenac
1- 15 years old girl in chronic regional pain syndrome not releaved by
oral morphine or electric nerve stimulation......fentanyl patches Pain is chronic

2-3years old clavicle fracture in acute pain..... IM or intranasal More rapid onset
morphine Especially in ER

3- 9 Y-Post peritonitis post surgical pain.....PCA pump Child is large


Post surgery pain

EMQ-ANTICONVULSANT
ABSENCE SIEZURE—ETHUXAMIDE
REFLEX ANOXIC SIESURES-REASSURANCE and eduction
FEBRILE CONVULSIONS-re assurance and explanation
EMQ CARDIOLOGY
1) 6 months girl with BP 100/ 70 femoral pulse delayed
than brachial and low volume---coarctation of aorta
2) 3 yr old with loud systolic murmur @mid LLSB, thrill +
VSD
3) one with ESM at PA and ejection click--pulmonary
stenosis (aortis stenosis radiated to neck and pulmonary radiate to back)
Only PS , AS , some VSD ==> present with mumur

All FOP Recall till June 2021 with note 169 of 378
EMQ-Renal :
UTI-ACUTE GN-CHRONIC RANAL FAILURE-HUS-RTA
1)baby 3 months high grade fever and wt loss- hematuria
and protienuria at the time of infection—CBC HB
10.9WBC=20--UTI ??
2) 2 weeks post skin infection ... hematuria-protiein urea
acute glomerulonephritis. 2-4 weeks
After throat infection

3) 14 yr old girl with delayed puberty and short stature


Turner’s syndrome

Proteinuria +++ CBC WITH NORMOCYTIC ANEMIA Creatinine


VERY HIGH-
chronic renal failure

EMQ-hemolytic anemia
1-turkish 1 YEAR OLD FTT hepatosplenomegaly –HB 6
micocytohypochrominic anemia,---thalassemia major
2-malazia dark coulored urine without HSM take herpal
ttt—G6PD
3-NORTH EUROPE 3years with mild jaundice for 1 day
during neonatal period-2 sister are normal-presented by
spleen ++only---conginital spherocytosis
EMQ-RESPIRATORY –same as TAS sample paper
investigation?

All FOP Recall till June 2021 with note 170 of 378
1-pt on 600 micro cortisone inhelar developed hypotension
fatigue weakness after GE
-SHORT SYNCTAN TEST =ACTH IV ,,,IF CORTISON NOT
INCREASE =ADRENAL INSUF Addison, easy fatigue

LONG SYNCTAN TEST IM INJ FOR 3DAY=(SURVIVAL P 175)


CASE 14 TAS
2-child with stone dullness percussion on rt chest after
having symptom of pneumonia with high fever and increase
breathlessness-CHEST US Empyema
3-child with long hx on ventilator for 4 months AS he was
preterm 26 weeks and became oxygen dependent –
tachypniec and need oxygen presented by recurrent croup??
may be high resolution CT chest or laryngoscopy???
Premature more harm for subglutic stenosis by long ventilation

7-mother pregnant ---


--if cadiac ds fetus-rubella
Baby with hydropes- parvovirus.
Baby with skin scarring -varicella. Skin scar, Limb malformation

8-rheumatology
1-fever tender warm joint-SEPTIC ARTHRITIS
2- fever 4 week ago and rash in the leg and buttocks-HSP
3-HUS CASE

All FOP Recall till June 2021 with note 171 of 378
9-scinero about IMN-ROSEOLA INFANTUM-KAWASKI
SYNDROME
10-vomiting no diarrhea
1-wt loss 6 weeks hungrey ---pyloric stenosis
2-3 months old wt loss fever lessergic =UTI
3-recurrant bilious vomiting-malrotation

Best of five

All FOP Recall till June 2021 with note 172 of 378

1-It is a question about boy 9 months and his testes in inguinal canal
and can be milked down to scrotum??so it retractile 1-reassurance
2-surgical referral 3-review after 3 months
• 2-Child with 5% burns and wrapped by wet cloths- distressed what to
do 1-pain killer 2-measure temp 3/ underess
• 3-This Q was regard which assessment you ask to ass he has normal
intellectual function 3 years old See Q ; 7

• 1-put toy under table


After 5 minutes —> give midaz
2-draw by crayon
• 4-child was still convulsing for 2 min and is feverish=39 and nurse was
upon his head with oxygen what your next action? 1-BuccaL
midazolam 2-OBSERVE 3-rectal paracetmol 4- RECTAL
IBUBROFEN ????????????????????
6 minutes
• 5-A 15 years old boy suffered injury during rugby ball Match, or more
unconscious for 5min, amnesia for 5 min only regain conscious. What
to do next?
Box 7.2
1-go for the match 2-play if CT normal 3-play after full recovery???
Box 103 Indication for CT, GBS , 1. GCS ,1. Basal skull fracture, 3. Seizure

• 6- 17 yours old mother just delivered, mother has bruised around one
eye, baby doing good. Elder child was placed at foster home for a brief
period at 1 your old age. What to do?

• Send mother n baby to grandmother's place


• Inform local midwife to keep on watching
• Don't discharge call the police ???
• Arrange coagulation study for baby
• Contact midwife regarding safeguarding.???

• 7-3 yr old child brought for assessment. Which milestone is


appropriate?
• Drawing with crayons Finemotor

• Brings shoes when asked for Below 3 year age

All FOP Recall till June 2021 with note 173 of 378

• Puts toy car under table
• Points to known objects At age of 18 month

8-Parents with learning difficulties and their son 20 MONTHS OLD can’t
talk at all. Didn’t want to cruise FTT ?
-emotional DEPRIVIATION
-CP-
• 9- HE can walk but can’t jump,,-12 y ,,can walk but clumsy and
deteriorate in sports and falls dawn during sports??? 1-DMD
2-Fredrechs ataxia?? 3-Becker.??

• 10-Child with obesity what is the cause to be treatable 1-ht 25centile


not really microenis ,
only fat embedded 2-MICROPENIS 3-bradypnea 4-FRAGILE X 5-poor school performance
Dt OSA

• 12-12 years old WELL CONTROLLED TYPE 1 DM with RBS 0.2 after
evening snack....Islet cell tumour-EXCESSIVE INSULIN INTAKE

• 13- 12 years old thin built very active athlete with weight at0.2 centile
height 75 centile....>hyperthyroidism-anorexia nervosa

• 14-Baby not sleeping at night until 2 am autism at2yrs,will not come to
breakfast , with social and language delay.,,with hyperactivity dt tired after not
sleeping at night
.ADHD-- GLUE EARL --AUTISM- -depresion????????

• 15-ask about puberty SEQUANCE in males-


Testis,pubic hair,height
In male; Testis, pubic hair, penis enlarge , growth spurt, voice
In female; breast, pubic hair , growth spurt, menarche
• 16-Premature thelarche 2 peak, 1st. 5 months to 2 year
2nd. 5 year to 8 year
SLE • 17-Rash..Anti ADNA +ve what should we do next—slit lamp Uveitis in SLA or JIA

• 18-Rash after taking amoxicillin for tonsilitis----+> infectious


mononucleosis

All FOP Recall till June 2021 with note 174 of 378

• 19-Same sinario of TAS sample paper, but need diagnosis not


investigation. Girl with hypoglycaemia and admitted became ok all 24
h of admission mama is a nurse father is a lawer FABRICATED
ILLNESS-KETOTIC HYPOGLYCEMIA
• 20-Newborn with irregular breathing n normal heart rate. Mother
attends methadone addiction rehabilitation group. What to do?
• Cpr n iv naloxone??
• Uvc catheterization n naloxone
• Cpr n im naloxone •
• 4.ventilation breath??

21-Obese child feeling tired during school hours, has already told
parents about being bullied in school .has stria in the abdomen
diagnosis?
•Depression
• OSA?
• cushing ?
• ..
• 21. Newborn discharged with no resuscitation needed. Chest x-ray
showed gas in abdomen. At 6 hrs developed choking while feeding, at
present maintaining saturation with o2, crackles in chest.mother was
polyhydrominous
• H type TEF No polyhydrominos, frequent choking and aspiration

Most common • esophageal atresia with distal fistula Gas in the abdomen dt fistula

• 3 diaphragmatic hernia Loop in the lung

• 22-Child 6 y with limping. No fever. Hip cannot be abducted or


internally rotated
• Perthes
• Slipped capital femoral epiphysis

All FOP Recall till June 2021 with note 175 of 378

• 3...
• 4...
• 5..
• 23-2 months old unimmunized child with cough requiring urgent
admission, no cabin available in ward. What to do?
Pertussis
• Transfer to other facility
• Admit in SCBU isolation cabin
• Keep in main ward n inform infection control people
• Keep in main ward till cabin available
• Push hard to vacate cabin ....???special care unit?

26-X liked pattern SCINARIO-what to do as mother ask about fate of


her baby? 5 month pregnant lady asks for genetic testing , maternal
grand uncle died and maternal aunts daughter had an abortion who
was a male. Advice given check if she is a carrier X linked recessive , if mother
carrier so baby will be carrier, if not
do investigations

• 27-Infant with fever vomiting bloody diarrhoea with increased counts


.... (picture of septic shock ) what will u do next ?
o stool culture
o USG abdomen

o oAbd X-ray

o ourea and electrolyte HUS

• 28-Girl with breathing difficulty(stridore) , numbness of fingers , chest -


clear Hyperventilating syndrome Rise in breath, not rise in HR

SpO2 normal
orebreathing mask Old ttt better rebreather training to avoid hypoxia
If not improved benzodiazepines —> behavior
oI/v calcium therapy —> serotonin uptake inhibitors
Rapid breath, normal HR and SPO2, Titanic symptom
mimic hypocalcemia, ttt psychological therapy, breath
train in acute first onset, if not improved , give
benzodiazepines, if recurrent, give same acute ttt then
serotonin re- uptake inhibitor if no improvement

All FOP Recall till June 2021 with note 176 of 378

• 29-GP role in parent meeting

• 30-Sign of cardiac disease (Faluire)

• ocyanosis

• operipheral edema

• odistended neck veins

• hepatomegaly

• 31-2 yr Child of parents from Pakistan, FTT , breast feeding till 1 year
now on solid foods and unmodified cows milk ,
• O/e pale , HSM +
• Diagnosis hb electrophoresis

• 32-Child with cough x 2 days ... had taken herbal medicine , developed
dark urine and microscopy normal oG6PD def

All FOP Recall till June 2021 with note 177 of 378
oLead poisoning

• 33-Child with cough x 2 days ... had taken herbal medicine , developed
dark urine and microscopy normal oG6PD def oLead poisoning

• 34-Child has cough x 2 weeks , clinically chest normal


• FBC … lymphocytosis

• -TB

• pertussis

• 35 Young Boy with acne , pubic and axilary hair with HTN
• Options :
opremature adrenarch no htn
oadrenal tumour

oCAH Mainly hypotension

• 36- Ophthalmology Question 5 months old not following his mother


what is most important in examinations.
• I think absent red reflex

• 37- Girl with epilepsy what is your plan in school?? Avoid swimming

• 38- Neonatal conj hyperbili but enzymes normal...


• . breast milk jaundice.
• .n hepatitis
• -biliary atresia
In early presentation liver enzymes normal

All FOP Recall till June 2021 with note 178 of 378
If therapeutic—> do without Not urgent
consent situations

• 39- Mather want to do circumcision to her boy and divorced I think


what to do...GP must refer for surgery,, parent can discuss togather
Then do legal advise and court If emergency no need consent

• 40-Neonate discharged at 6th day and readmission on day 8 by poor


feedind and sepsis ..E coli?

• 41-Pt with torticolis ,ear pain,?!!retropharyngeal abcess?!! Urgent ENT

• 42- is in inguinal canal and can be milked down to scrotum so it


retractile just reassurance or what ??
• 43-which are contraindicated in nephrotic syndrome?!! MMR
• 44-There is one for Dka maintenance fluids

• 45- hypoglycemia after short acting insulin BY MISTAKE FROM THE
MOTHER
glucogel oral
• 46-NEONATE WITH SUDDEN DESATURATION NOT IMPROVED WITH OXYGEN
WHAT TO GIVE-IBUPROFEN ,,PROSTAGLANDIN

• 47-Infant with irritabilatity and vomiting and hx of SIDS WHICH of the
following lead to diagnosis—
• glucose MCAD confirmed by glucose

• amonia

48-Prophylaxix of meningiococcal meningitis=cipro

• -rifamicin (FOR HAEMOPHLUS LESS THAN 10YR)

(GIUDLINE) * All age ciproflo,


* use of rifampicin 1. If 2nd infection treated with Cipro, 2. H
influenza, 3.

• 49-Child 5 y with allergy to penicillin and tonsillitis what to give –


clindamycin
• -cipro
• azithromycin

All FOP Recall till June 2021 with note 179 of 378
• 50-Pt develop rash after penicillin and had sore throat-IMN
• 51-UNDESCENDED TESTES at 9 months old –refer to surgery
• Should do first year

• 52-Case of a girl with SVT first step


• vasalva maneuver
If in the hospital, ice pack
• -ice pack If outside hospitals, valsalva maneuver
• If infant, immersion to icy water

• 53-ASian 6HB with palor and cousin parents and HSM diagnosis?SCD
//THALESMIA SCD usually no hepatomegaly only splenomegaly

• 54-Pt with features of molliscum contaguism and teased by his friends


–ttt cryotherapy--- If in face —> silver nitrate
If no teasing, reassuring

• 55-Baby 3 months hypotonic and smile wt below .2 centile
• –SMA Myotonic dystrophy mask face Or mother mask face or ptosis
• 56-FEVER FOR 7 DAYS AND CRACKED LIPS AND RED EYE diagnosis
kwasaki

* If child 3 years or low —> give salbutamol spacer + face mask


* if child > 3 year —> give only spacer with salbutamol

• 57-3y old athmatic ask about method to give inhaled salbutamol spacer
WITH FACE MASC
• 58-3 y old with fever that followed by rash-diagnosis-
roseola infantum
PS, TA, EA,,,, HYpoplatic lt ,
• 59-What is duct dependenat disease -pulmonary atresia COA
• 60-Child spilled coffee over his chest in wet wrap brought to hospital
and distressed what to give –pain relief-,,AB-measure temp,, un-
dressing
• 61-Diagnosis of obesity
• BMI 91- Morbid obesity 99.5
Obesity 98
• WT MORE THAN 75- Overweight 91
• WAIST MORE THAN75

• 62-PAKASTni boy 2 ys pale breastfeed until 1 year on solid food and


unmodified cow milk-no HSM- IRON DEF ANEMIA

All FOP Recall till June 2021 with note 180 of 378
thank you

All FOP Recall till June 2021 with note 181 of 378
The question was recalled and collected then reviewed with most accepted answer from
1. a post in Facebook group MRCPCH
2. through a zoom meeting with a some candidate who attended the exam —> search for the recorded meeting
3. another meeting to review with same candidate who appeared the exam after result came to confirm the answer with doctors got 100% in each
system —> search for the recorded meeting
4. Dermatology; was direct message from doctor her friend got 100 % in dermatology

‫بسم ا) الرحمن الرحيم‬


Examination of FOP June 2018
1st EMQ

Cardiology
EMQ 1;
Option; VSD, ASD, PDA, pulmonary valve stenosis, supra pulmonary stenosis, COA, innocent
murmur.
all SPO2 > 95% and child not cyanosed No comment about change with position
a. 1 month, 2-3/6 systolic murmur left lower sternal margin.
b. neonatal on discharge ejection systolic murmur on left upper sternal margin with thrill
c. neonatal on 10 days long systolic murmur left upper sternal margin both BP in left arm and leg
high normal SBP 70

Answer; Can be innocent murmur but not load < 3/6 systolic murmur Still murmur most common 2-7 year
a. VSD; as no innocent murmur in left lower sternal .. b. PS; due to thrill association, in ASD no
thrill .. c. PDA; (long murmur represent machinery)
the majority same answer and confirm by one got 100 % in cardiology

Genetic
Hemophilia —> population risk
EMQ 2; confitrm 100 %
Options; AD, AR, X-linked dominant , X-linked recessive, multifactorial, XO, XXY
a. a boy delay walking then start to walk, apart of hypotonia other examination normal, his uncle
was using wheelchair
b. short girl her hight .4th centile mid parental hight 25th centile, no obvious physical feature
c. persistent vomiting at 3-6 week has grand father did gastro surgery at child age

Answer;
a. X-linked recessive; the case DMD .. b. XO; the case is Turner .. c. multifactorial; the case is
infantile hypertrophic pyloric stenosis
confirm by many got 100% in genetic

Dermatology
EMQ 3;
Option; measles, kawasaki, cow milk allergy, scarlet fever.
a. 10 month baby presented with fever 38.5 conjunctivitis, cough. confluent blanching rashes,
started on his face then cover entire all his body. he has history of coryza with fever few days
back, his sister 3 year brother has recently started nursery
b. 18 month girl with fever for 1 week, conjunctivitis , red tongue , swelling hands, spread
erythematous rashes, palpable cervical LN fever
c. 18 months girl 3 days unwell, feverish on & off red cheeks pale around her mouth, pus in the
tonsil, erythematous rashes all over body. her fever recorded up to 38.5 HR 150.

Answer;
a. measles .. b. kawasaki .. c. scarlet fever
exactly same in sample paper

DM
EMQ 4;

All FOP Recall till June 2021 with note 182 of 378
Option; extra dose short acting insulin, IVF with insulin infusion, continuous same dose with oral
fluid, call emergency for admission, decrease long acting insulin.
all child known to have DM type 1 on long acting insulin and short acting insulin each meal
a. came with RBS 21 ketone +, whole the day, look well. with history of fever 38, cough and sore
throat
b. came with RBS 4.6 ketone +, unwell with history of nauseating, vomiting —-> GE
c. came with low glucose 2 mmol at 2 am and in the morning hyperglycaemia

Answer;
a. extra dose short acting insulin (child uncontrolled sugar due to febrile illness .. b. continuous
same dose with oral glucose fluid vs continuous same dose with IVF ( child control sugar but sick
with GE) .. c. decrease long acting insulin (Somogy phenomenon)

Rheumatology
EMQ 5;
Options; (CBC, CRP, ESR), X-ray hip, X-ray hip & knee, X-ray frog position, Aspiration, US knee,
CT scan hip, blood film
a. child with pain in knee, nor redness no swelling, fever 38 . hip limited internal rotated what the
initial investigation Clue that no sign of septic arthritis or osteomyelitis, case may be reactive arthritis, so do routine labs work

b. child limping, limiting internal rotation , BMI 27 —> Xray frog for upper epiphyseal norma
c. child with history of frequent infection before, now came with bone ache look pale and unwell
B scenario no mention of age so either Perthes or SUFE, Illustrated book, Ttt of perthes, rest, pt, cast,surgery, ttt of SUFE,
mentioned both need X-ray including frog view only surgery
Answer;
a. (CBC, CRP, ESR); as long as not redness no swelling so no sign of osteomyelitis so the initial
investigation .. b. X-ray frog position; this case of SUFE as child is obese, normal BMI 25 .. c. blood
film; to exclude ALL

Respiratory
EMQ 6;
Option; oral prednisolone, IV salbutamol, oral dexamethasone, IV fluid, Reassurance, salbutamol
puff, salbutamol nebuliser.
a. 5 year child known BA came with acute distress, although given salbutamol nebuliser but
saturation remain 88 %,
b. child with history of cold presented with milld noisy breathing,
c. and infant with history of cough and slight decrease oral intake with anxious mother

Answer;
a. IV salbutamol; no IV hydrocortisone in the option (sever BA) .. b. oral dexamethasone (mild
croup) .. c. reassure (acute bronchiolitis)

GIT
EMQ 7; Most GIT associated with Down is duodenal atresia
Option; duodenal atresia, pyloric stenosis, Hirschsprung Disease, volvulus, intussusception
a. baby delivered at home, mother told green vomitus within 18 hr, baby was hypotonia Volvulou
b. baby around 4 week persists vomit and look hungry her vomit coming after milk feed suddenly s=
traverse across their room. bilious
c. an infant with vomiting, on and off abdominal pain loose bowel motion (not sure)
vomit
Answer;
a. duodenal atresia; hypotonic child = Down syndrome .. b. pyloric stenosis; with projectile vomiting
.. c. intussusception (not sure) On and off abdominal pain, do US for target sign
Intermittent abdominal pain ;
- Intussusception
Rehabilitation - malrotation
EMQ 8;
Option; physiotherapy, occupation therapy, Dietician, play therapy, orthotist, orthoptist,

All FOP Recall till June 2021 with note 183 of 378
a. child with CP has chocking while eating .. FTT —> physiotherapy ??? most probable speech
and language due to incoordination —> see sample paper old one Speech is first choice , if not in the choice chose
occupational
b. 4 year persistent tip toe — orthotist fro design of limb
c. child with limping hemiplegia discrepancy in the length of his leg, he has hip pain and back pain
his examination and Xray was normal—> physiotherapy

Answer;
a. refer to speech and language therapist; (chocking is due to breathing & swallowing
incoordination) repeat in old sample paper .. b. refer to orthotist (for designing of limb .. c.
physiotherapy. At age of 4-5 orthopist, at age 8 year surgery
orthoptist = physiotherapy of eyes

Neurology
EMQ 9;
Option; brain tumour, cluster headache, tension headache, migraine ,
a. child with bilateral headache banding in nature not interfere with activity
b. child with long history of headache now become more severe not relief by paracetamol and
ibuprofen and associated with morning vomiting and ataxia
c. child long history of headache some time associated with abdominal pain and vomiting

Answer;
a. tension headache .. b. brain tumour .. c. migraine feature of vomiting
confirm by one got 100 % in neurology

Haematology
EMQ 10;
Option; aplastic anaemia, ALL, G6PD, SCA, thalassmia, HS,
a. a child with history of viral infection before then he become pale Hb low, WBC normal
b. feature of SCA play football cold weather leg pain look pale
c. a child with pallor and splenomegaly ??

Answer;
a. aplastic anaemia (no parvovirus infection in option).. b. SCA .. c. HS ??

2nd BOF

Ethics & Safeguarding (9 question)


BOF 1;
an 13 year child brought by ambulance took 20 tablet acetaminophen while waiting the result of
drug level taken at 4 hr child escape . parent was called and they answered she is not at home
a. call police
b. social worker will ask emergency protection
c. parents when come back
d. security to find her
e. doctor responsible for child protection.

Answer; call police ,, child in danger emergency situation ,, confirm by one got 100 % in ethics

BOF 2;
an 11 year female just arrived form Egypt, mother show her pants with obvious blood, her brest
and axilla tanner stage 1, she refuse for local examination,
a. female genital mutilation
b. precocious puberty

All FOP Recall till June 2021 with note 184 of 378
c. vaginal infection
d. menarche

Answer; female genital mutilation , clue came from travel ,, not in puberty ,, confirm by one got
100% in ethics

BOF 3;
child has brought by the teacher with intestinal obstruction need urgent gastro surgery, parent not
around, the home was called grandmother was there .. the child register in protection plan ; who
give the consent
a. grandmother
b. surgeon who will do the operation
c. doctor on charge of care
d. the teacher
e. doctor on call in child protection. ??

Answer; ??? surgeon who will do operation as it is emergency confirm by one got 100 % in ethics

BOF 4;
13 year old girl with living with mother who known a sex worker was bringing her at night presented
with enuresis, constipation, vaginal greenish discharge, the child has change behaviour like to stay
alone, ???
a. non consensual contact with adult
b. non consensual contact with child
c. consensual sex

Answer; non consensual contact with adult confirm by one got 100 % in ethics
vaginal greenish discharge clue for STD

BOF 5;
A man found a medical sheet of admitted patient in the restaurant of the hospital . he inform you as
a doctor then you found same hand writing of your colleague
what you will do ?
a. to talk to the colleague XX
b. inform local medical team 1. Safety of patient
c. inform medical governcy 2. Talk to colleagues
3. Inform the consultant
d. call medical council XX 4. Incident report
e. call consultant

Answer; call consultant confirm by one got 100 % in ethics Talk to the colleague

BOF 6;
13 year mother found OCP in her bed room, and went to GP for advise. what the GP will do ?
a. advise to talk to her mother Daughter
b. don’t see her and keep confidentiality.
c. ask the girl if she has a boyfriend
d. prepare appointment to whole family
e. prepare appointment with girl alone
Doughter
Answer; advise to talk to her mother a advise confirm by one got 100% in ethics
she below age of confidentiality.

BOF 7; safeguarding

All FOP Recall till June 2021 with note 185 of 378
Neonatal 12 hour brought by parents with abdominal bruises, vomiting of blood. what is the best to
do ?
a. CT bran
b. skeletal survey
c. coagulation profile
d. Vitamin K IV
e. IV antibiotic

Answer; CT brain; (for NAI delay presentation, late to came to hospital .. vitamin k present with
bleeding ,, child still not moving clue is the age ) confirm with many got 100 % in ethics

BOF 8; ethic or safeguarding


A 4 month baby the nurse forgot to give vitamin K IM at birth then he was admitted with (i think
bleeding). you admitted him for 2 days and ask the nurse to give vitamin K now. the father is
waiting and asking if anything happen ?
a. complet incident report and inform consultant
b. Tel parent what exact happened and apologise to them
c. medication error due
d. Tel parent nothing happen and the baby taking his routine supplement.
e. inform parents and the consultant

Answer; inform parents and the consultant ?? confused by many


see guideline
patient safety and inform consultant
open communication
documentation

BOF 9;
mother with HIV just delivered a new born,, doctor decided HIV prophylaxis .. she came with her
partner (biological father) who is not aware about of her HIV...
a. inform the father the condition baby
b. discuss with mother to tell the husband
c. inform child care for baby protection as he is at risk
d. inform the GP about the condition of baby & its risk If mother refuse to tell
e. follow for HIV team management. If the father knew, inform HIV management father—> follow HIV team

Answer; ?? Discuss with mother to tell husband

Adolescent & Behavioural


BOF 10;
Athlete girl BMI 18 training she is follow special diet arrange by her coach every day has secondly
amenorrhea
a. athlete amenorrhea
b. anorexia nervosa
Triad ( osteoporosis, amenorrhea, feeding
Answer; athlete amenorrhea disorder)

BOF 11;
child aggressive behaviour at home after they transfer to a new home. He academically good at
school especially in math and since subject not interest in group play.
a. conduct disorder
b. ADHD
c. Autism spectrum disease

All FOP Recall till June 2021 with note 186 of 378
Answer; Autism spectrum Disease = Aspergers disease child usually follow strict manner routine if
changed the child become disruptive behaviour confirm by one got 100 % in behavioural

BOF 12; Behavioural / Psychiatry


child 6 year easy distractible fail to make friend delayed reading 2 year different to compare with
his classmate. not concentrating in his academy .in the school classmates can enjoy play with him
but no body like to invite him to his party at home, calling the teacher impolite
a. dyspraxia
b. ADHD
c. dyslexia
d. autism
e. deafness ???
Psychology and behavior assessment
Answer; ADHD

BOF 13; Gastroentology


baby with tachypnea persisted vomiting at 12 hour old CXR no air bubble in the abdomen
a. tracheo-esophageal fistula,
b. proximal oesophageal atresia without fistula

Answer; proximal oesophageal atresia without fistula majority choose it

BOF 14; Nephroloogy


child has fever no focus with urine analysis negative for bacteria, glucose, protein and nitrite, WBC
= 20 increased,
a. viral infection
b. TB infection
c. UTI
d. bacterial vaginitis

Answer; bacterial vaginitis clue is female no long history confirm by 2 doctor got 100%
sterile pyuria see guideline long list differential include TB infection with long history

BOF 15; Gastroentology


A case of child with chronic constipation for 6 month fecal mass reach till abdomen,
a. Admit for dis impaction
b. Oral dis impaction at home with close review -> senna or laxative
c. review within one week -> lactulose
d. start laxative -> rectal dis impaction
-> surgery
Answer; Oral dis impaction at home with close review ????

BOF 16; Nutrition or Gastro


infant FTT with generalised edema and underweight look dehydrated, miserable
a. milk based formula
b. cow milk formula High protein content
c. NS bolus
d. soya formula
e. albumin infusion

Answer; cow milk formula for Kwashiorkor ,,


unless child not shock don't give IV

All FOP Recall till June 2021 with note 187 of 378
BOF 17; Nutrition
preterm baby now 4 month ,, history of 10 day diarrhoea mother switch breast to formula due to
her work ,, baby has eczema
a. hydrolysate formula
b. preterm formula
c. soya formula
d. lactose free
Should be antibiotics 24 pre & post
Answer; hydrolysed formula confirm by one got 100 % in nutrition

BOF 18; Infections


baby born to mother who previous pregnancy has GBS baby, the mother afebrile during delivery ,,
she was taking IV antibiotic before ,, what u will do to the baby
a. take CRP & swab, start iv antibiotic and observe for 24hr
b. choose observation for 48 hours
c. reassure and come back to hospital if something happen
d.

Answer; take CRP & swab, start iv antibiotic and observe for 24hr (as per guideline one risk)
see fop cases one risk observation
tow risk IV antibiotic

BOF 19; Nutrition


Sudani mother live in UK want to breast feed her baby for long time for 6 month mother BMI < 18..
which deficiency baby will develop ?
a. Vitamin D deficiency
b. iron deficiency
c. folic acid
d. protein

Answer; Vitamin D (dark mother with bad nutrition) confirm by many got 100 in nutrition

BOF 20; Musculoskeletal


child pain below knee with swelling
a. Osgood shelter disease
b. perthe
c. SUFE
d. tibial tubercle
e. osteosarcoma

Answer; Osgood shelter disease.. the majority choose it

BOF 21; Respiratory


female girl long history of chesty cough,, multiple previous admission due to chest infection
has a palpable mass upper right quadrant ,, what investigation ??
a. sweet test
Sweet test - 20 ; normal
b. Ct chest
- 40-60 ; repeat
c. X ray - 60-125 mmol/ L ; positive

Answer; sweet test confirm with 2 got 100

BOF 22; Gastroentology


child eating variety of food but can't gain weight, he has low Hb, with history diarrhoea abdomen is
protuberant.
a. coeliac

All FOP Recall till June 2021 with note 188 of 378
b. rickets
c.

Answer; coeliac

BOF 23; Cardiology


child ejection systolic murmur on right sternal margin radiate to neck with thrill
a. AS
b. PS

Answer; AS confirm by one got 100 in cardio

BOF 24; Gastroentology


how to diagnose the sure persistent vomiting (diagnosed pyloric stenosis)
a. Hpochloremia
b. hypokalemia
c.

Answer; hypochloremia the majority choose it First electrolytes disturbance, chloride

BOF 25; Nephrology


2 month baby with UTI klebsilla on antibiotic not improving for 3 days
a. Do US now and MCUG at 6wk and DMSA at 6month
b. Do US now and MCUG and DMSA at 6 wks idid it wrong i guess its dmsa after 6month

Answer; US now MCUG 6 week DMSA 6 month


see guideline

BOF 26; Safeguarding


baby of twins by mistake he receive wrong blood transfusion then develop blood reaction
how to avoid blood group mismatching
a. check the identification of the patient bed side
b. check the name
c. check the ID number
d. to follow blood bank strategy

Answer; check the identification of the patient bed side confirm by many got 100 % in safeguarding

BOF 27; Dermatology


female with swelling raised edge fluctuate in the thigh blue colour bleed after mother scratch
accidentally .. what you will do ?
a. observation
b. check platelet level
c. laser
d. steroid

Answer; check platelet


(Hemangioma if bleed you have to check platelet to exclude low platelet due to consumption that is
Kassaback merit syndrome see survival reference ) confirm by one got 100 % in Dermatology

BOF 28; Respiratory


child with recurrent RTI during past winter after attending a party he developed sudden onset
cough not resolve by 2 course of antibiotic CXR twice persistent consolidation finding in right side
a. FB
b.

All FOP Recall till June 2021 with note 189 of 378
Answer; FB .. confirm by tow got 100 in respiratory

BOF 29; Neurology


toddler went with her mother to shopping she want to buy sweet but the mother refused then she
cry too much cyanosed then seizing
a. vagal attack
b. expiratory anxiety apnea
c. partial complex
d. psudo seizure

Answer; expiratory anxiety apnea = breath holding spell

BOF 30; Neuro development


a male with macrocephaly (i cant remember more detail) what is initial to do ?
a. review after 3 month
b. measure parent head circumference
c. see previous measurement In red book

Answer; measure parent head circumference ?? the majority ,, vs see previous measurement ,,

BOF 31; Neonatology or Endocrine & Growth


baby birth weight 2.2 faltering thrive at 2 year 8.4 kg what to do ?
a. past record previous measurement If maintaining same percentile so no problem
b. discharge with check up by health care
c. review after 3 month
d.

Answer; review after 3 moth

BOF 32; Endocrine & Growth or Gastroentology confused chapter


An obese girl BMI . 99.6 centile
a. exercise and decrease calorie
b. dietitian referral
c. orlistat
d. exercise alone
e. weight management program

Answer; weight management program many different answer


orlistat prevent lipid absorption it use if with hyperlipidemia
review guideline

BOF 33; Ethics


A 10 year girl mother DM type 2 on Asthma management ,, her father had heart attack, and uncle
of father died young Familial
a. lipid profile hypeelipidemia
b. no worry she is girl
c. Measure lipid and follow up until puberty If > 2 year —> lipid profile with follow till puberty
> 2 year —> do lipid profile for father
d. wait until the girl competent

Answer; Measure lipid and follow up until puberty majority


risk of FAP familial adenomatous polyposis at age

BOF 34; Pharmacology


A girl with epilepsy on carbemazpine and allergic to penicillin came with community acquired
pneumonia what you will give ?

All FOP Recall till June 2021 with note 190 of 378
a. erythromycin
b. doxycycline
c. amoxicillin
d. cephlexin 5 % allergy to penicillin
e. ciprofloxacine

Answer; cephlexin (carbemazipine enzyme inducer, erythromycin inhibitor) confirm by one got 100
in pharma

BOF 35; Ophthalmology


child with inflamed eye lid painful limitation of movement
a. oral amoxicillin
b. IV ceftriaxone

Answer; IV ceftriaxone for orbital cellulitis confirm by many got 100 % in ophthalmology

BOF 36; Neurology


What most imp sign to do ct brain
a. vomiting 3 times
b. Fall of 3 meter
c. Amnesia
d. loss of consciousness > 5 min
e. bruises around both eye

Answer; bruises around both eye panda sign

BOF 37; Diabetic M


Girl with type 1DM take 20 unit basal insulin and bolus insulin (can’t remember the dose) each
meal with so many reading given in a table something like below

day 1 day 2 day 3 day 4 day 5 day 6

breakfast 7 6 7.5

lunch 7 7 6

dinner 20 21

befor sleep 19 20
a. increase short acting lunch and evening
b. increase long acting insulin

Answer; increase short acting lunch and evening

BOF 38; Cardiology


down with lethargy fever 1 month and tachycardia tachypnea and VSD and fever HR = 190 with
liver 4 finger below costal margin, splenomegaly, haematuria.
a. infective endocarditis
b. viral myocarditis

Answer; infective endocarditis confirm by one got 100 % in cardio

All FOP Recall till June 2021 with note 191 of 378
BOF 39; Dermatology
New born 5 days with vesicle on trunk and lesion on rt toe redness after 2 days, baby febrile high
grade and vomiting
a. herpes infection
b. epirdetmolysis bulosa
c. staph skin syndrome
d. varicella

Answer; staph skin syndrome confirm by one got 100 % in derma

BOF 40;
a boy poly depsya and bolyurea with urine osmolarity 200 and specific gravity 1008 i guess well
baby,
a. habitual polydipsia
b. DI nephrogenic
c. DI central
d. DM
Habitual polydipsia urine osmolarity 600, after
Answer; habitual polydipsia .. DI nephrogenic deprivation test 750
clue look well no water deprivation in scenario

BOF 41; Nutrition


A girl Ca low phosphate low ALP high
a. hypophosphatemic rickets
b. Vitamin D
c.

Answer; Vitamin D the majority choose it

BOF 42; Endocrine & Growth


new born 24 hour with ambegous genetalia you assure the parent investigation in process.. what to
do to aid diagnosis within 24 hour?
a. chromosomal analysis
1st day: chromosomal analysis
b. 17 hydroxycholesterol 2nd day : urea and electrolytes
c. FISH Hydroxyprogestrone 3rd day : 17 hydroxycholestrol
d. urea & electrolyte

Answer; chromosomal analysis = karyotyping .. confirm by one got 100 in endocrine

BOF 43; Neonatology


the most risk factor for SIDS?
a. Bed sharing
b. separate room from parent

Answer; bed sharing confirm by many got 100 % in neonatology


cot death

BOF 44; DM
DKA child with hyponatremia child HR high, capillary refill > 3 second
a. 20ml/ kg 0.9% NS
b. 10ml/ kg 0.9% NS
c. dextrose
d. insulin IV

Answer; 10ml/ kg 0.9% NS majority 20 ml/ kg NS

All FOP Recall till June 2021 with note 192 of 378
see guideline
steal
BOF 45; Psychology
3 year temper tantrum toddler still chocolate from market change behaviour after the new born
sister arrived , living together with grandmother
a. praise him for good behaviour
b. mom to tell him she love her
c. assure him every body love her

Answer; assure him every body love her ???

BOF 46; Ophthalmology


squint by 6 month
a. refer to ophthalmology
b. refer to orthopist visual acuity correction
c. = physiotherapy of eye

Answer; refer to ophthalmology confirm by many got 100

BOF 47; Ethic


consultant prescribe methotrexate for management of JRA, the parent read methotrexate is used
of treatment of cancer .. what to do
a. ask rheumatology consultant help
b. start methotrexate and monitor CBC
c. stop methotrexate
d. some drug can be used off licence
e. inform parents the use of methotrexate after fail of other management

Answer; start methotrexate and monitor CBC Ask rheumatologist consultant

BOF 48; Respiratory ?????


chest physiotherapy is best for
a. bronchiolitis
b. impact foreign body
c. broncheactesis
d. SMA

Answer; broncheactesis vs. SMA No antibiotics needed

BOF 49; Infectious Disease


a young child 3-4 year with bell palsy and skin erythema
a. IV doxycycline XX
b. amoxicillin
c. penicillin
d. ceftriaxone
< 9 year amoxicillin
e. > 9 year doxycycline
Cardiac complications, ceftriaxone
Answer; amoxicillin for lyme disease

BOF 50; Musculoskeletal


child with tip toe venu varus at age of 4 year and When reassuring
a. reassurance - tip toe —> 3 y
- venu Varus —> 6 y
Answer; reassurance
it is norma till age of 6 year

Tip toe —> orthotist


In toeing —> inpatient orthodontist

All FOP Recall till June 2021 with note 193 of 378
BOF 51; Gastroenterology
A girl 14 year delay puberty has of abdominal pain loose motion 3-7 time ,, the mothers has
vitiligo, what initial investigation to do ??
a. colonoscopy
b. marker of celiac disease
c. fecal calprotien non invasive ?? see SOP
d. MIBG scann ???

Answer; fecal calprotien vs colonoscopy different answer

BOF 52; Musculoskeletal


a female age walk at 24 month she born preterm she can stand easily from sit position
a. diplegic cp
b. DMD
c. DDH

Answer; diplegic cp vs DDH

BOF 53; Dermatology


3 year child severe eczema using twice daily hydrocortisne and there was thickening of skin, what
the next ?
a. clobetazone moderate
Severe eczema —> most potent steroid
b. dermovate most ptent
c. oral steroid
d. oral tacrolimmus
e. 6 hour hydrocortisone

Answer; dermovate confirm by one got 100 in derma

BOF 54; Immunology


mother of a baby developed rash 2 days before labour .. what you will give
a. IV acyclovir
b. VZIG
c.

Answer; VZIG the majority choose it


see new guideline —> mother has rash develop 7 days before or after — give IVIG

BOF 55; Gastroentology


a child 8 month brought to the hospital for investigation due to jaundice found to have yellow hand
chest, and abdomen and white eyes mother like to drink vegetable purred juice pale
a. caroteinemia
b. Gilbert
c. breast milk jaundice

Answer; caroteinemia the majority choose it


clue no jaundice

BOF 56; Emergency


baby boy haemorrhage from circumcision i guess with tachycardia HR ~ 180 and capillary refill
time > 3 sec cold extremities .. what is the initial step to do ?
a. blood transfusion
b. FFP
c. IV NS

Answer; IV NS the majority choose it

All FOP Recall till June 2021 with note 194 of 378
BOF 57;
child generalised oedema ascites what best investigation to do ??
a. serum albumin
b. urine analysis the initial investigation
c. ASO titre

Answer; serum albumin vs urine analysis (for nephrotic syndrome


deferent answer even with doctors who got 100 in nephrology or since of practice

BOF 58; Respiratory


7 year traveller chronic cough for 6 month CXR reticule nodular shadow investigation ??
a. tuberculin test
b. CT chest high resolution
c. HIV Gastric lavage in active TB
d. sweet test

Answer; tuberculin test confirm one got 100 in respiratory

BOF 59; Infectious / Immunology


child know case of congenital neutropenia has fever was measure at home 38. 3
a. ask to go to hospital if another spike
b. go to hospital as soon as possible
c. wait till fever subside
d. measure temperature after one hour if still high go to hospital

Answer; go to hospital as soon as possible confirm by one got 100 in infection

BOF 60; Ophthalmology


Neonate 12 week don’t follow the mother
a. referee to ophthalmic
b. review later

Answer; referee to ophthalmic confirm by many got 100 % in ophthalmology

BOF 61; Palliative


child with cancer Increasing bone pain .. On paracetamol regular and morphine PRN, has kidney
problem, platelet low
a. Morphine pump
b. morphine SC
c. oral morphine slow release twice daily
d. ibuprofen

Answer; oral morphine slow release twice daily confirm by 2 got 100% in palliative
see case fop

BOF 62; DM vs Psychiatry


12 year DM on insulin .. Poorly controlled with frequent admissions due to DKA .. The parents are
very meticulous about her treatment .. What is the best for compliance of the child ?
a. Psychology referral
b. insulin pump
c. involve her in weekend diabetic camp
d. let’s the child to take the insulin

Answer; involve her in weekend diabetic camp

All FOP Recall till June 2021 with note 195 of 378
BOF 63; Hematology
child from Bangladesh has bone pain in knee for long time feature of pancytopenia, X ray
osteolytic lesion in many site of bone
a. Osteosarcoma
b. leukaemia
c. TB
d.

Answer; Leukemia

BOF 64; Musculoskeletal


14 year obese limping
a. slipped upper femoral epiphyseal
b. perthe’s disease

Answer; SUFE the majority choose it

BOF 65; Immunology


baby 2 month continuous crying for 2 hours baby after immunisation of
a. continuous same schedule immunisation
b. stop to give one of immunisation (not remember

Answer; continuous same schedule immunisation in guideline if > 3 hours red flag

BOF 66; Hematology


2 month child diagnosed with SCD what you will give - 3 months —> give antibiotic + vaccinations
a. iron - 6 months —> folic acid + succynile urea
b. folic acid
c. pneumococcal vaccine
d. Antibiotic prophylaxis Given at 2 year

Answer; folic acid (pneumococcal vaccine regular given see guideline prophylaxis vaccine)
Antibiotic prophylaxis

BOF 67 Respiratory
BMI obese headache , at sleep produce sound ,not concentrate at school
a. OSA
b. Day dream
c. absence seizure

Answer; OSA

BOF 68; Neurodevelopment


when to refer the child
a. not sitting not support at ?? month
b. not walking by 15 month
c. convergent squint at 6 month
d. tip toe walking by 18 month
e. syllabus bubble at 6 month

Answer; convergent squint at 6 month

BOF 69; Nutrition


mother want to give breast feeding which contraindication

not sitting not support at ?? month

All FOP Recall till June 2021 with note 196 of 378
a. HIV
b. hepatitis B

answer HIV

BOF 70
about growth of on infant and the answer reassurance (sorry can’t remember the answer)

‫بالتوفيق للجميع‬
‫ تنسونا من صالح دعائكم‬:

All FOP Recall till June 2021 with note 197 of 378
FOP
February 2018

Kindly note that the I’ve put the


probable answers, they are not
confirmed as true answers

EMQs:
1/Genetic risk calculation

1:4
1:2 Hemophilia is X-linked
Population risk no male to male transmission
1 no 0 in the option
2/3 population risk

- Risk of having an affected baby for a well parents


who have a son with sickle cell disease ?
-Risk of having a baby (boy) to a father who has
haemophilla A ?
-Risk for a daughter to be a carrier of cystic fibrosis ,
she has an affected sibling ?

Answers:1:4, population risk, 2/3

2/Neonatal resuscitation

All FOP Recall till June 2021 with note 198 of 378
Dry and wrap in a warm towel
Naloxone
Endotarcheal tube under direct visualization
Inflation breath
Bag and mask ventilation
But in neutral position

-Term, meconium stained liquor


GCS9 at5 min cried immediately ,pink and well
Apgar score 9
-Term,bradycardic, mother took naloxone 2
hours bf labour, dried and stimulated but no response
- Term,floppy ,mecoinum seen in nasopharynx
HR > 100
Answers : Inflation breath
Dry and wrap in a warm towel , Bag and mask
ventilation Endotarcheal tube under direct
visualization if meconium seen can insert ETT Suction under direct
visualization

4/ (GIT)

Upper GIT endoscopy with biopsy


Colonoscopy with biopsy
IgA anti tissue transglutaminase
U/S abd
Serum amylase
Stool for occult blood

All has a Father with duodenal ulcer, aunt with croh's

All FOP Recall till June 2021 with note 199 of 378
-14 year old with diarrhea and blood and weight loss
RIF
-3 year old no wt gain miserable with vomiting and
abd distension.
-14 year old with upper abd pain and wake in night. Gastritis
Answers: Colonoscopy with biopsy
IgA anti tissue transglutaminase
Upper GIT endoscopy with biopsy

5/(Dermatology)

Emollients
Propranolol
Cryotherapy
Silver nitrate
Reassurance
Steroids
Intralesional scklerotherapy

-Baby with dry skin on his trunk, sister has eczema ? Emollient
-Baby has a large haemangioma which prevent his eye
opening ? propranolol If ophthalmology in options , still choose
propranolol
-Child has mulloscum contagiosum and is teased by his
friends bcz of it? Cryotherapy

6/(Haematology)

Nurtitonal iron def anaemia


hereditary shpetocytosis

All FOP Recall till June 2021 with note 200 of 378
Thalassemia major
Self limited, after viral
transient erythrocytopenis of childhood infection parvovirus
Thalssimea minor Black fan in infancy, TEC in child , both retic 0 but
Blackfan need steroid

-4 month old, breast fed baby,Hb low , MVV low ,


reticulocytes normal?

-Child required phototherapy in the neonatal period ,


no has jaundice , splenomegaly:
Hb low
Retics: high

-3 month old had viral infection few days before:


relics 0.1
MCV normal = 89
HB lower limit of normal = 10. 9

Answers: Nurtitonal iron def anaemia


hereditary spherocytosis
transient erythrocytopenis of childhood Typical normocytic
normochromin
7/Investigations anemia retic 0.1

Sweat test
Bronchoscopy
Dairy for peak flow meter
PH studies

-Baby with FTT , recurrent chest infections , mother


said he has piles taking steroids & salbutamol but not
improve

All FOP Recall till June 2021 with note 201 of 378
-Child on inhaled salbutamol, day symptoms improved
, still wheezes after running and has notctunal cough
3 year -A toddler developed sudden shortness of breath when
they were in a party, he has a localized wheeze on the
right middle lobe?

Answers: Sweat test


Dairy for peak flow meter
Bronchoscopy

8/(Jaundice)

Cephalohaematoma
G6PD
Failure to establish enough breast feeding
ABO incompatibility

-Baby born term weighing 3.4 kg, at 5 day presented


with drowsiness and jaundice, mother blood group A+,
Child O + , sodium 150, urea10 Hypernatremic dehydration

-Large swelling on head after ventose extraction


mother blood group O+,babyO+ , high un conjugated
bilirubin AB+ A+

Turkish boy has viral infection and developed


jaundice,, positive FH family history Retic 5 %

Answers: Failure to establish enough breast feeding,

All FOP Recall till June 2021 with note 202 of 378
Cephalohaematoma
G6PD

9/(Seizures)

Generalized tonic clinic epilepsy


Hypocalcemia
Hypoglycemia
Meningitis
Tuberous sclerosis
Sturg Weber

-Baby flexes his legs and cries and becomes pale ,


his mother has hypertension Infantile spasm
13 y -Girl developed fits , she was treated by
carbamezapine, had temp 38.7and vomiting since-
last night, has blotchy rapid spreading rash.
- Asian boy with fits , his mother has a limp and
previous f# Fracture

Answers: Tuberous sclerosis


Meningitis
Hypocalcemia

10/ GIT

Hurshsprung
Intussupsion
Pyloric stenosis
Gastro-osophageal reflux GOR

All FOP Recall till June 2021 with note 203 of 378
Gastroenteritis
Ulcerative colitis

-Baby on amoxcilin, has fever and vomiting6 times


miserable
irritable crying , which contained blood in the last two = blood vomit

episodes If in scenario bloody stool —> intusseption

5 month -Baby can’t gain weight, has bilious non bilious


vomiting every time he feeds, he is always hungry.
mother after meal projectile
GOR no FTT
-Down with bilious vomiting. but GERD has FTT

Answers:Gastroentritis
Pyloric stenosis
Hirshsprung -- duodenal atresia at first day
Hirshprung at 6 days
If intermittent abdominal pain —>
malrotaion , valvular

BOF

1/ 14 year old with chlamydia urethrithritis

-Docxycycline for 7 days


-Amoxicillin
-Trimethoprim
-Cephalosporin

All FOP Recall till June 2021 with note 204 of 378
Answer:Docxycycline for 7 days

2/A girl with viral infection and rash temp 40


conjunctivitis and pink TM, what to give? Measles
-Erythromycin
-Antihistaminic
-Paracetamol
others all antibiotics
Answer:Paracetamol

3/oral prophylaxis of 3 year old whose brother has


meningitis?
-Ciprofloxacin
- Ceftraxone
4/7 year old boy with 3 spike and wave,ttt ?

-Lamotigine
-Sodium valporate first line in guidelines
*no ethuxamide in the options First line —> ethosuximide
Second line —> boy ; valproate ......... girl ; lamotrigine

Answer:Sodium valporate

5/ ADHD scenario child is not sleeping at night , he


watches TV till late night and then falls asleep but
he is tired next morning , management?

-Sleep hygiene
-CAMHS
-Melatonin not license in pedia

All FOP Recall till June 2021 with note 205 of 378
Answer:Sleep hygiene

6/ADHD scenario , child is having difficulty in


writing and refused by school teams, messy eater,
can’t tie his shoes he is on methylphenidate , what
is he having? coordinating problem

-Dyspraxia
-Methylphenidate side effect
-Dyslexia

Answer:Dypraxia Referr to occupational therapy

7/ADHD cenario, symptoms reported during the last


year both at school and home, mother thinks he
need treatment,what is the most appropriate
action?

-Ask for report from school If scenario only at home , so the school report

-Assessment by educational psychologist


-Perscribe methylphenidate

Answer:Assessment by educational psychologist

8/4 month old has E. coli UTI Which responded to


treatment,what to do next?

-U/S after 6 wks

All FOP Recall till June 2021 with note 206 of 378
-MCUG * US at 6, 12 month —> 1. If increasing size should remove, or sometime

-DMSA
involuted... 2. to detect any hydronephrosis then do MCUG
* MCUG Should do dt associated visicoureteric reflux
* DMSA to assure the other kidney
Blood pressure yearly
Answer :U/S after 6 wks

9/2 or 3 year old girl with hypertension ,abd pain


and fever,normal abd examination? Antenatal
DMSA
diagnosed, do

-Wilms tumor by exclusion others no


-Dysplastic kidney fever
-Renal artery stenosis

Answer:Dysplastic kidney Recurrent UTI , fever


10/13 year old girl wants to have HPV vaccine and
her mother refused?
-She can have it if she is Gillick competent
-Wait until she is 18
-Follow mother’s wish

Answer: She can have it if she is Gillick competent

11/10 year old came to the ED with her mother’s


boyfriend, she is due to appendicectomy after 4
hours, her mother is outside the country and can’t
get back ,who will consent ?
- Mother after coming
-Mother by the phone no consent through phone
- The doctor on behalf of child
-The boyfriend
Just inform parents through
phone then the doctor will
sign the consent
All FOP Recall till June 2021 with note 207 of 378
-The Child

Answer:The doctor Mother by the phone

12/Child has adrenoleukodystrophy ,came to a new


hospital in which he has no register, he developed
respiratory failure , what is the best action?

-Intubate and ventilate


-Ask the parents if they have end of life plan
-Call his consultant in his original hospital
- transfer to previous hospital
Answer : Intubate and ventilate
* first ask the parent if child for DNR —> if yes don’t intubate as here - no chance -
situation. even contraindications for intubation due - unbearable suffering
* If no yet decisions from parents —> u have to intubate and ventilate
13/A nurse prepared double the dose of gentamicin
for a pt and it was given by mistake?

-Tell the parents what happened and that the


investigation is going on.
-Tell the parents what happened and who is
responsible
-Start invest and then tell the parents
-Don’t tell the parents.

Answer: Tell the parents what happened and that the


investigation is going on.

All FOP Recall till June 2021 with note 208 of 378
14/ A doctor gave a child caffeine which is
unlicensed , what should he do about this?

-Write in the file why you used it


-Tell the parents that you gave their child an Clear
message
unlicensed drug Correct

-Tell the parents the expected SE.


-Tell the parents that you’ve used all the therapies
with no response.

Answer: Tell the parents that you’ve used all the


therapies with no response.

15/ Girl with peculiar feeding habits, lost wt,


mother thinks she induces vomiting , what is the
clinical finding most suggestive of this diagnosis?

-Teeth erosion
-Secondary amenorrhea

Answer:Teeth erosion

16/3 days old with breast enlargement and blood in


the nappies?

-Normal finding Hormonal withdrawals


-Drug during pregnancy

All FOP Recall till June 2021 with note 209 of 378
not include in heart lesion cyanotic,
Answer:Normal finding prosthetic vulve... see guide

17/ Case of AS for septic operation? Previous cured IE


-Operate and don’t give prophylaxis No surgery done before
-Give prophylaxis in the high risk procedures
-No prophylaxis and come if Pt developed symptoms

Answer : No prophylaxis and come if Pt developed


symptoms

18/Teenager has ear piercing after which she


developed local infection, pale, has apical
murmur,night sweating , investigation? IE

-Blood film
-Blood culture
-Echocardiogram

Answer:Echo or repeated culture

19/Boy collapsed during a hockey game, then slowly


regained consciousness , his cousin has epilepsy,
investigation?

-ECG
-EEG
-Blood glucose

All FOP Recall till June 2021 with note 210 of 378
Answer:ECG

20/ The site of BCG vaccination?

-Intadermal in the left arm between the shoulder and


deltoid
-Intradermal in the right arm
“”””””””””””””””””””””””””””
-Intradermal in the left arm at the insertion of deltoid
ms
-Subcutaneous in the lt arm between shoulder and
deltoid

Answer: Intradermal in the left arm at the insertion of


deltoid ms

21/ Long scenario of acute case of DKA ,then at the


end of the scenario they asked what is the best
maintenance fluid in DKA?

-0.9% saline with 20 mmol KCl in 500 ml


-0.9% saline + glucose

Answer: -0.9% saline with 20 mmol KCl in 500 ml

All FOP Recall till June 2021 with note 211 of 378
22/An obese girl with headache. Examination show
bilateral papilloedema, she can’t abduct her Squint
eye ,Whats the next step? Idiopathic intracranial hypertentiin
has false localized sign Is diagnosed by
exclusion on MRI
-Cranial MRI Correct Fop — MRI with venograpgy
-LP with manometry Then do LP
Review
Answer:LP with manometry

23/ Fixed squint at 4 months, mother has history of


squint in the past ....? any squint with > 4 month age refer to
-Refer to ophthalmology ophthalmology 1. Fixed squint
If fixed squint at 3 month —> refer any age
-Review at 6 month ophthalmology need early referral 2. Apparent squint > 3
If transit squint reviews 6 month months
3. Loss red reflex
Answer :Refer to ophthalmology Pastest any age
4. Not flollow his
mother

24/ Staph aureus pneumonia pt became more 5. Parents


concern

distressed suddenly ,diminished air entry on left


side, trachea deviated to Hyper resonance
right? percussion

-Pleural effusion
-Pneumothorax

Answer:Pneumothorax

25/ Scenario of a case mostly nephrotic syndrome


and came with vomiting, cold peripheries, CRT 3
Sec , ttt? Child in shock

All FOP Recall till June 2021 with note 212 of 378
indication of albumin in nephrotic
- albumin 20 % # indication of albumin 1. scrotal edema 2. massive
acsitis 3. nutrition deficiency
-Saline bolus
-Oral predenisolone
- Maintenance IV Fluids

Answer:Saline bolus as child in shock

26/Discolored swelling in anterior triangle of neck fluctuate


since birth, not hot nor tender, Pt born and lived in
the UK , fully vaccinated , U/S showed clear fluid?
-Infected branchial cyst Since birth infection is surrounded by wall
No clear fluid, is
lymph node -Atypical mycobacterium in lateral site Appear in child
cell not give sign of infection
since birth.. painless
-Tubercolous adenitis

Answer:Infected brachial cyst

27/ Case of ALL on chemotherapy has contact with


varicella patient? Has IgG negative.. not immunized before
-VZIG and come back if rash appears
- VZIG and Acyclovir if rash appears
-VZIG and varicella vaccine

Answer: VZIG and Acyclovir if rash appears

28/ 7 year old has knee pain and a limp especially


after sports ?
- Perth’s dz Look to the age
- Slipped femoral epiphysis Above 10 year

All FOP Recall till June 2021 with note 213 of 378
- JIA

Answer:Perth’s dz

29/Bay with mild dysmorphic featured, this upper


lip and smooth phlithrum? Child was taking from poster care

-Fetal alcohol syndrome

30/Child in social care register, presented with


extensive bruises in his shin,epistaxis and purpura
in the trunk, he had URTI,diagnosis?
-ITP
HSP
Child abuse

Answer :ITP due to epistaxis

31/8yr old previous viral infection...now pain in


ankle joint...calf muscle tenderness present on
passive dorsiflexion.....all are normal apart from CK
>1000?

-Achilis tendonitis not give high CK Should tenderness over Achilles

-Benign myositis

Answer: Benign myositis

All FOP Recall till June 2021 with note 214 of 378
13 year
32/ A girl with coughing for 3
weeks,vaccinatedPetechia over face, shoulder and
chest,diagnosis ? no petiche in lower body

-Pertussis
-ITP

33/ Case of febrile convulsions he was 2 years I


think turning blue mother is nurse father teacher,
advise in next attack? After given buccal midazolum
Rectal paracetamol —> blue lips—> ambulance

-Buccal midzolam - 5 min for airway, oxygen, glucose —>


-Ambulance - buccal midaz (better) or rectal diaz —>
-Rt position - call ambulance for further IV Mange
Call ambulance
1. After 10 min. of seizure
2. Unable to give medication

Answer :buccal medazolam 3. Turn blue or not breath


after given midaz buccal

12 year girl
34/ One boy with liver enzymes high and
pigmentation of skin glucosria
obese,gyenecomastia,RBS normal,diagnosis?
4-6
review
-Type 2 DM
-Fatty non alcoholic liver
-Cushing High glucose , no cortisone in history , not
asthma
Answer:Fatty non alcoholic liver

All FOP Recall till June 2021 with note 215 of 378
35/ Boy was in a restaurant , developed rash and
swollen face after he ate an ice cream , no
difficulty in breathing nor stridor, ttt?
-Oral antihistamine If no stridor no destress
-IM adrenaline —> no need to give
-IV antihistamine adrenaline & steroid

Answer:Oral antihistamine

36/ 14 years old female 147 and her mother is 167,


she is concerned that she is short , which of the
following will indicate that she will be at least as
tall as her mother?

-Breast development stage 2 Spurt start after breast development stage 2 ( most
accurate)

-Pubic hair 2 review


-Menache period in past 2 years ago
-8 cm increase in height last year

Answer : 8 cm increase in Ht last year


4-7 year every year
37/A girl 14 years with brown pigmentation 3 in
number 1 cm each in her back and scoliosis , when
she bends with both her hands on the ground the Rt
hips still raised more than the LT,Diagnosis ? Hypermobility

-NF rash more than 5


-Bone tumor
-Idiopathic Late onset

If corrected with bending —> posture

All FOP Recall till June 2021 with note 216 of 378
Answer:Idiopathic The majority chose it by exclusion

38/A boy with normal fine motor


Motor development sat unsupported at 7 months
Rolled at 5, he is now 18 months not walking, Reach
—> DMD
18 month boy

hypotonia, normal reflexes, diagnosis ?


-DDH
-DMD
-Bottom shuffler
-Becker’s muscular dystrophy
may complete normal till 2 year (survival)
Answer:DMD The majority then start to progress weakness

39/A child has hypothyroidism, came for follow up,


has normal T4 but raised TSH, best next step?

-Measure free T3
-Call the GP to ask about Pt compliance.

Answer: Call the GP to ask about Pt compliance.

40/A girl at 3 months not following with eyes.


Reviewed after another 3 months still not improving
iris free fundus free, diagnosis ?

-Optic atrophy fundus is pale


6 month
-Toxoplasma
-Delayed maturation if visual axis

By exclusion
All FOP Recall till June 2021 with note 217 of 378
-Birth asphyxia If mild it will not affect

Answer: Delayed maturation of visual axis

8 year
41/A boy from Bangladesh came to the UK a year Pale
ago presented with liver and spleen ,jaundiced and
tired in the school, no symptoms in his country?
Either thalassemia or malignant
-Mantaux
-CBC with film Malaria vs. ALL
-Stool for parasites
-HB electrophoresis for thalassemia

Answer :CBC with film

42/A Boy having problems with C KR and tongue tie


but when he was an infant no problems with
feeding.
He has problems in school coz he is not saying
words right ,referral? if diagnosed —> optimum
In tie tongue problem pronounce ( t n d l s z r ) Surgery management surgery simple
brief can be done in clinic no
-Paediatric surgery policy for wait and see
-language and speech therapy to avoid any early social
obstacle to the child

Answer:Language and speech therapy

43/Case of a boy 18 months had normal hearing


test, what would warrant further testing ?

All FOP Recall till June 2021 with note 218 of 378
-Temper tantrum Normal till 3 year
-Not obeying mother review in
-Watching TV at high sound pastest
-2 acute otitis media in 3 months

Answer: -2 acute otitis media in 3 months


no history of
44/A boy in a restaurant developed sudden couch
shortness of breath, presented with drooling and
noisy breathing, he has eczema and asthma, past history
diagnosis? of asthma &
eczema
Only
-Anaphylaxis
respiratory
sign
-Foreign body inhalation * Anaphylaxis either mild to moderate —>
mainly skin sign and slow progression
-Epiglottis Or severe —> drooling with hypotension
both extremities not in scenario
* FB if chocked child not produce couph

Answer:Anaphylaxis vs foreign body

45/Most common complication of infant of diabetic


mother?

-Sacral agenisis
-Diaphragmatic hernia Most cardiac complications, HOCM

-Doudenal atresia Most GIT complications duodenal atresia


Global most complications sacral agenisis

Answer: Sacral agenisis

All FOP Recall till June 2021 with note 219 of 378
46/Baby with ambiguous genitalia and absenct
gonands, what test will lead to the diagnosis?
Urgent karyotype for gender
-Karyotyping 17 oh progesterone delay 5 days later
see guidelines
-17 OH progesterone CAH
-U/S
As the question want
Answer:17 oh progesterone to lead to diagnosis

47/Regarding the epidemiology of RSV?


season
-Incidence of asthma will increase after RSV infection
-15%will require hospital admission 20-30 % required admission

48/4 month hearing test ? he pass autoacustic

-Brain stem evoked potentials


-Tympanomety

Answer: Brain stem evoked potentials = ABR

49/12 year old presented with acute sudden onset


of vomiting ,pain in the hemiscrotum and
erythema,diagnosis ?

-Testicular torsion
- Incarcerated Inguinal hernia
-Torted hydatid cyst of Morgagni No erythema

All FOP Recall till June 2021 with note 220 of 378
Answer: Testicular torsion Fit with age

50/3 month old baby girl developed blood in stools


after her mother started work and gave her bottle
feeds,diagnosis?

-Cow milk allergy

51/A boy with limb pains, fatigue,URTI and pallor, I


think there were investigation showing low Hb and
thrombocytopenia, diagnosis ?
Platelet 18
-ALL
genu varum
52/Child with short stature ,Low ca, Ph and 25
hydroxycholecalciferol, ttt?

-Vit D
-1 hydroxycholeciferol

Answer :Vit D

54/A boy developed seizures after routine hernial


repair,had hypoglycemia in the past during a
gastroenteritis episode, forgot the rest of the
scenario, but it was towards MCAD

fasting before surgery ??

review
All FOP Recall till June 2021 with note 221 of 378
55/ Indication of skeletal survey from the
following?
-2 year old with spiral femure #
-21 month with spiral tibial# Fob cases & guideline
-Sibling with a non accidental injury skeletal survey < 1-2 year
-4 month with spiral humorous #

Answer: 4 month with spiral humorous #

56/Term baby born with C/S due to fetal distress


developed tachypnoea at 4 hours,what is the cause?

-Transient tachypnoea of the newborn


-Sepsis

Answer:TTN

57/8yrs girl developed 2ndry enuresis and fecal


siloing after her mom brought her partner with his
15 yrs old son to the house, she stays more hours
in her bedroom, diagnosis?

-Constpation
-Child abuse

Answer: Child abuse

58/A boy passing stool without feeling, teased at


school because of this, history of neglect, ttt?

All FOP Recall till June 2021 with note 222 of 378
-CAMHS
-Laxative
-Star chart

Answer:CAMHS

59: 4 year old with tonsilar old enlargement,


snoring, what is next to do to guide to mgt?
can’t awake in the morning OSA
-Ploysomnography 1st

-Lateral X-ray
-Over night oxygen monitoring 2nd

Answer:Polysmnography = sleep study

60/ Diabetic with vomiting and diarrhea for 24 hrs


looks miserable.RBS 6 mmoml..
urine ketone +
what is your management..?
-Give oral sugary fluids May the answer, child not DKA
-Refer to emergency department Guidelines
-Give his insulin and encourage oral fluids
-Stop insulin Omit insulin & observe

Answer: Refer to emergency department


due to vomiting miserable —> emergency
61/When to refer to pediatric surgery from the
following?

All FOP Recall till June 2021 with note 223 of 378
Risk of malignancy
-Undescended testis at 6 month review pastest
-Paraumblical hernia at 8 month
- Non retractable foreskin at 3 years Phymosis no need referral

Answer: Non retractable foreskin at 3 years Pastest


Phimosis foreskin balloon
Non retractile foreskin = balanitis xerotica —> surgery
62/ Child has 3 Amber scores in the traffic light
system for emergency assessment, what to do?
Guidelines
-Blood and urine cultures,LP,Cxr for fever
-Blood and urine cultures,LP
Other options…..

63/case of headache and 6th nerve palsy asking


about diagnosis ? 6 month duration headache

-Space occupying lesion Due to Tumor

64/Child has abd pain and rash in his legs and


buttocks, history of viral infection?

HSP

65/one pt having convulsion for 20 min, at home


receive midazolam at home then lorazepam in
ambulance .what is your next step?

-phenytoin bollous
- Phenobarbital

All FOP Recall till June 2021 with note 224 of 378
If drug given over 1 hour —> infusion
If drug given over 20 min —> bolus
-phenytoin infusion
-Lorazepam

Answer:Phenytoin infusion

66/A girl came with vulval bleeding ,she was playing


with her teenage brother who said she got the
trauma while she was ridding a bicycle
What to do first?

-Call senior
-Call Gyne
-Suite the wound

Answer:Call senior

67/ Most potent steroid ?


-Clobatisone
-dermovate (clobitasol)
-betnovate

Answer: Dermovate(clobitasol)
68/Baby has jaundice , Coombs rest weakly
positive ? 5 days un conjugated bilirubin very high ,
mother O , baby AB
ABO

All FOP Recall till June 2021 with note 225 of 378
69/ 4year old girl with asthma on salbutamol with
controlled her daytime wheeze, but she still has
wheezes at night and when she runs, next step?

-Inhaled corticosteroids
-Monteleukast

Answer:Inhaled corticosterid

70/ 13 year old girl has Down syndrome with wt


Loss 5 kg in the last month , pale, proximal ms
atrophy , didn’t start puperty , abandoned by her
mother when she was a baby, HR 120,
agitated,diagnosis ?

-Hyperthyroidism
-Anorexia nervosa
-Psycholgical deprivation

Answer:Hyperthyrodism

‫ﺗﻢ ﺑﺤﻤﺪ ﷲ‬
‫ﺑﺎﻟﺗوﻓﯾق ﻟﻠﺟﻣﯾﻊ‬

71/ Child hypothyroidism on treatment


came
normal T4 and high TSH

answer call GP for non complaint

All FOP Recall till June 2021 with note 226 of 378
EXAM FOP - 2017 October
Extended Match Questions ( EMQs)

* CARDIO
Options ( ASD - PS - PDA - VSD - TOF - TGA - AS - innocent
murmer ) 

EMQ 1 :
- loud pansystolic murmur on lower sternal edge
- bounding pulses with continous murmur
- 8 week old ejection systolic murmur second left ics. PS transmitted to the back +
thrill
* PS the correct answer as ASD is asymptomatic at this
Answers : VSD - PDA - ASD age 2 month no murmur yet
PS * one got 100 % in cardio with answer PS in Facebook
EMQ 2 : * majority in Facebook —> PS

- scenario baby cyanosis and no murmer when he pull his leg up


-
- ejection click 2nd ics to the right radiate to neck
- Scenario with viral infection then 1/6 mid-systolic murmer

Answers : TOF - AS - Innocent Murmer

* Lymphnodes
EMQ 3 :
Options ( Reactive - Atypical mycobacterium - TB - Rubella - ebv
- ALL )
- scenario generalized lymphnodes , splenomegaly
- 4 year non tender with color change above it 1 node cxr clear-
long period neck node
- tonsillitis one month back , 0.5 nodes ( multiple ) in the
occipital area and posterior triangle
Review
Answers : ALL - Atypical Mycobacterium - Reactive Lymphadinitis

* Thrombocytopenia :
EMQ 4 :
Options ( CMV - Alloimmune - Group B strep - ParvoVirus ,
ITP .. etc )
- 15 year old pregnant lady give birth to IUGR baby by c/s due to
transverse lie at 35 weeks presented with fits, brusies and
spleenomegaly of 3 cm . 

- 15 year old pregnant lady give birth to baby by c/s due to
transverse lie at 35 weeks presented with fits, on examination

All FOP Recall till June 2021 with note 227 of 378
mottled, tachypenic , plt 120 , wbc 2 
 GBS sepsis
- 15 year old pregnant lady give birth to baby by C/s due to
transverse lie at 35 weeks presented with fits. Antenatal scan at
20 weeks showed acistes , HGB low , plt low .

Answers : CMV - Group B Strep - Parvovirus severe anemia


hydrops fetalis
* GASTRO
EMQ 5 :
List of Causes of Gastroentestinal bleeding ( Same as Sample so sure
paper ?TAS ) answer
Options ( Anal fissure - crohns - Angiodysplasia - Colonic Polyp
- Cow’s milk Allergy - Diverticulitis - HUS - HSP - Sexual abuse -
Yersenia

- Scenario 2 day hx of bloody diarhea and fever - non blanching


rash on his trunk , HGb 7 , Plt 20 , WBC 15 , albumin 38 .
- Scenario 8 yrs boy - 6 mnt hx of bloody diarhea ass. Wight loss
and lethargy . HGB 9 , Plt 524 ( high esr , crp ) .
- Scenario 12 yr - intermittent painless bleed with normal stool ,
no diagnostic clinical feature , normal labs .

Answers : HUS - Crohns - Conlonic Polyp More in elder child


Anal fissure painful & diverticulitis intermittent pain
EMQ 6 : Mickel deverticulum painless bloody stool More blood, younger age
Choose suitable investigation fecal elastase for
Options ( CRP and ESR , Amylase , fecal elastase , Anti-tissue malabsorption of
transglutaminase , H pylori stool , H pylori serology ) pancreas
amylase for
All diabetic patient and complain of
acute
- scenrio with peri-umbilical pain and abdominal distention , pancreatitis
constipation . Antitussue transglutaminase Celiac It’s mean
- scenario with intermittent epigastric pain , poor apetite , wake relegated to
feed
at night , wt loss Serology of h pylori , better in stool serology Gastric ulcer gastritis Increase
- scenario with right iliac fossa pain and mass , diarhea for 6 with sleep —> h
weeks pylori
Or chronic
Answers : Anti-tissue transglutaminase - H pylori serology Vs malabsorption for
fecal elastase
Amylase - Crohn’s ) CRP & ESR
Explanation : 1st scenarion celiac - 2nd scenario peptic ulcer or
pancreatitis - 3rd scenario crohn’s

Pancreatitis continue severe


Gastritis Increase with sleep

All FOP Recall till June 2021 with note 228 of 378
* PALLIATIVE
EMQ 7 :
Options ( ondansetron - metoclopramide - increase morphine
frequency - increase morphine dose - decrease morphine -
movical - hyosine patch - senna - s/c Pump morphine )
- scenario of patient with mst ( slow release morphine ) and prn morphine
oromorphin and pain in the last few days reached 8 extra prn sulphate tab
doses max in last 48 hours .
- scenario patient with neuroblastoma with mst and prn morphin
and pain is controlled but she has developed difficulty in
swallowing review
- scenario patient on chemotherapy didn’t pass normal bowel
habit since 3 days . Hyoscine putch

Suggested Answers not sure : S/c morphine pump - Decrease Due to side
morphine - Movicol effect morphine
fentanyl pach If mentioned in scenario not controlled pain
* Neurology
EMQ 8 :
seizure management
Options ( IV lorazepam - IV phenobarbitone - Rectal ibuprofen
- PR paraldehyde - oral phenobarpitone - Vigabatrin -
Carbamezipine - oral NA valproate )
Review
- scenarion of a neonate and develop frequent fits
- scenario with temp of 39’c and develop seizure half an hour ,
given 5 mg diazepam PR by paramedics , still seizing ?
- 2 years old with generalized tonic clonic fits. 3 episodes in the
last 1 month

Answers : Phenobarbitone * oral Vs IV not sure - IV lorazepam -


Oral Valproate .

* POISONING
EMQ 9 :
OPTIONS ( Acetylcystine iv , diazepam , Glucose , Naloxone ,
Flumazinel ,methionine oral,,, etc )
- methadone 

- 14 yr old boy presented after night out with friends (deny any
drug only drinking ) . Pale and sweaty .confused . Key words in alcohol sweaty —>
sign of hypoglycemia
- Girl took 25 tablet of paracetamol after 2 hour irritable and
refusing canulation

All FOP Recall till June 2021 with note 229 of 378
always exclude hypoglycemia c/p sweet, confusing, shaky pale

Methionine capsule same efficacy of NAC (see Oxford)

Answers: Naloxone - Glucose - Acetylcystine Iv. Vs Methionine


oral “ because she is refusing “

* RESP
EMQ 10 :
Initial Management
Options ( oral prednisolone , oral dexamethasone , nebulized
adrenaline , salbutamol inhalor , oxygen , neb. Ibratrobium )
- mild croup scenario coryza
- 18 mth . wheezy chest first attack
- long scenarion can’t remeber but sp02 - 89%
high grade fever& other symptoms of pneumonia
Answers : Oral dexamethasone , Salbutamol inhalor , Oxygen
* Laryngomalecia decrease with sleep.
EMQ 11 : * if early age vascular ring increase with sleep and
Stridor eating dt pressure from esophagus
Options ( Croup - Laryngiomalacia - subglottic stenosis -
subglottic hemangioma - vascular ring - inhaled foreign body ) 

- Scenario of Infant with stridor since age of 2 days increased
with sleeping and feeding . 

- Scenario of infant with progressive stridor since he was 2 wks Stridor before
and now stridor is biphasic ( hx of hernia repair at 4 wk - hx of operation
stridor with some improvement with steroids )
- Scenario of child with hx of sudden cough then progressive
stridor with resp distress .
Laryngomalecia Increase with inspiration
Answers : Vascular Ring - subglottic haemangioma Vs Stenosis -
Inhaled foreign body Differential diagnosis of persistence biphasic stridor
1. vascular ring (increase with feed & sleep)
2. subglottic hemangioma rapidly progress dt. Increase size
* DERMA may improve with steroids
EMQ 12 : 3. Vocal cord paresis ( horses cry)
question same as previous exam 4. Tracheolaryngomalecia ( rare if severe)

Options ( Reassure - topical steroid - oral steroid - injectable


steroids - oral gresfolvin )
- Scenario with alopecia areata
- Scenario with a girl has scaly lesion with crust and used
antidandroff no improvement
- Scenario of hair fall ( different hair longs ) , hx of pneumonia ,,
now she is fine .
Dt. Stress

Answers : Topical steroids - Oral Grisofulvin - Reassurance

All FOP Recall till June 2021 with note 230 of 378
* Rheumatology
EMQ 13 :
Options ( septic - sle - jia  scleroderma ..
detmatomycitis ..Lyme ?
- joint pain , lymphnodes , HTN
- Rash in the face , eyelid and back of fingers
- child with swelling of knee joint. I think there was raised skin
lesion of ankle with central pallor.

Answers : SLE - Dermatomyositis - Lyme

* Emergency
EMQ 14
Investigation
Options ( Blood glucose - Blood pressure - Serum urea and
electrolytes , Urine albumin and creatinine , Ct Brain , Abdomen
us etc ) Pancreas big —> hypoglycemia —> convulsions
- scenario of a baby with convulsion . large tongue BWS
- 14years girl was well , history of vuR when was infant now came
Hypertensive
with confusion from many weeks and hyper reflexia encephalopathy
- hx of fall , unilateral pupils
Scenario 2 frequent UTI —> uremia —> high BP
Review
Answers : Blood Glucose - BP - CT brain
Explanation : Beckwith wedman more prone to hypoglycemia ,
2nd scenario may be encephalopathy so will measure BP .

Best Of Five ( BOF )

1- Adolcent girl 4 week hx of fever , night sweet headache wt


loss splenomegally heart murmur grade 2/6 clubbing 

-Non Hodgkin lymphoma

-Sarcoidosis


All FOP Recall till June 2021 with note 231 of 378
-Infective endocarditis 

-Reumatic fever
-SLE

Answer : Infective Endocarditis due to new heart murmur

2- There was also a Q about child coming with calf mass. Bruising
after playing football with brother

History of clumsy walking. And delayed walking with avoidance of
sports in school , father also I think had similar condition normal
apgar score 

_cerebral palsy No bruises
_hemophilia A
 X linked, father will not affected
- Ehler Danlos EDS Father affected AD
but hemophilia X linked no male to male transmission

Answer : Ehler Danlos Syndrome ( autosomal dominant )


3- chronic cough investigated thoroughly normal spirometry . 14
yr Girl coughs during day but no night time symptoms.
Management
- Otolarngeal referal
- say to her no serious problem and reassure
- placebo

Answer : say to her no serios problem and reassure ( habitual


cough )

4- Girl with type 1 dm. On basal bolus regimen with poor control.
Hba1c given was 7.9% .there was stressful condition at home
Parents heard her screaming and saw her fitting and sweating
what is the cause
- Night terror ppt factor to hypoglycemia
1. stress
- Hyperglycaemia 
 2. miss meal
- Hypoglycaemia

Answer : Hypoglycemia

5- A girl with fever, increasing headache for 4 days. Develops


upper limb fits. HTN , Bradycardia . Best management before
investigation ?

Iv mannitol
 Cushing triad requires emergency ttt

All FOP Recall till June 2021 with note 232 of 378
Iv antibiotics 

Iv acyclovir and iv antibiotics

Answer : Iv acyclovir and IV antibiotic ( golden hour to start


antibiotic ) Vs Iv mannitol ( due to increase ICP with cushing triad
)

6- Child with dysuria , developed lately 2’ry enuresis , us done


showing renal stone.
Dipstick
Proteinuria +++
Blood +
Leukocyte +
Nitrate - Proteus mainly
What is the Cause
 with flank pain

 and hematuria
- Hyperparathyoidism

- Cystionuria
- Distal Renal Tubular acidosis 

- Proteus uti
 the majority proteus as 2ry enuresis to infection
- E. Coli uti

Answer : Proteus UTI ( will caus stones ) Vs Distal renal tubular


acidosis ( also causes stones and proteinuria )

7- An 18 month asylum seeker migrates from africa to uk . No


details of his vaccination available.
What are you going to Do ?

- Normal uk schedule with rotavirus

- Normal uk schedule without rotavirus

- Bcg followed by uk schedule
- Vaccinate according to country of origin schedule

Answer : BCG followed by UK schedule

8- Tanner Staging for a girl presented to You with Breast Bud and
sparse pubic hair ?
- Breast Stage 1 pubic stage 1
- Breast Stage 1 pubic stage 2
- Breast Stage 2 pubic stage 2

All FOP Recall till June 2021 with note 233 of 378
- Breast Stage 2 pubic stage 1
- breast stage 2 pubic stage 3

Answer : Breast Stage 2 Pubic Stage 3 if sparse & spread over junction of
pubis or mons pubis

9- 7 month old with coryza and cough given amoxcilin next day
developed vomiting and profuse diarrhea and managed with oral
rehydration what is the Cause?

Clue age is less than 2 year
- Rotavirus 
 common is rota viral
- RSV 

- Antibiotic induced entritis not cause vomiting

Answer : Rotavirus ( Gastroenteritis ) profuse diarrheal = watery diarrhea

10 - scenario with NAI ( non accidental injury )


What is most indicative
- Spiral fracture
- asymmetrical nappy rash
- pariatal haematoma —> cause accident injury if isolated

Answer : Spiral Fracture

11- patient with hx of headache which of the following is


significant alarm ?

- Increase in severity

- Increase in frequency 

- Wake from sleep with vomiting

Answer : Wake from sleep with Vomiting

12- 9 months old with gcs of 6. Mother says he rolled off sofa
while 3 yr old was watching him. Most appropriate step after
stabilisation?

- Ct brain

- US brain

- Skeletal survey suspect NAI
- Clotting profile

All FOP Recall till June 2021 with note 234 of 378
Answer : CT Brain ( indication in less than 1 year with low GCS )
13- Most serious complication in a SLE mother
- complete heart block
- Thrombocytopenia

Answer : Complete heart Block Irreversible

14- 14 year old girl deteriorate in her school performance , with


weight gain and can't do well in gemnastic cession . scanty
periods Autoimmune thyroditis —> proximal myopathy
- hypothyroidism review
- cushing Pigmentation, hairshotism
- bullying
- anorexia n. No gain weight

Answer : Hypothyroidism

15 - scenario with 4 weeks baby with DDH ( unstable left hip ) for
pediatric surgery tommorow and mother asks you what he will do
?
- Open reduction
- Close reduction
- abduction traction double nappies after first screen at birth
- abduction splint but 2nd screen at 6 wk — by ortho do
- douple nappies abduction splint

Answer : Abduction Splint

16- ‫ﻣﻜﺮر‬Defenition of obesity , screaning ?


- BMI >98th centile
- Weist circumfrance > 95th centile
- BMI > 95th centile

Answer : BMI >95th centile ( As per Nice Guideline >95th centile if


screaning and survailance , >98th if for clinical assesment )

All FOP Recall till June 2021 with note 235 of 378
17 - 14 year old girl she presented with smelly vaginal discharge
she told you that she has 2 time unprotected sex with a boy , and
she knew that he has been sleeping with other classmate after
her , she told you not to tell her parents what to do ?
- contact social worker
- transfer to sexual clinic
- encourage her to tell her parents and you will keep her
confidentiality
- encourage her to tell her parents and you can’t keep her
confedentiality for guide a proper act (in their culture ) as she
unprotected sex
Answer : Transfer to sexual clinic Vs Social worker , As there is a
problem with the sexual behaviour

18 - 12 year old boy with crohn’s disease came for regular follow
up and told that he have some lesions around the perianal area ,
and told not to tell his parents what to do ?
- encourage him to tell his parents and keep confidentiality
- encourage him to tell his parents but tell him that you can’t
keep confidentiality

Answer : Encourage him to tell his parent and keep


confidentiality

19 - 45 year old mother having huntigton , knowing that it is


inherited she asked for her 9 old boy to be tested and not tell
him why ?
Hyperlipidemia lipid profile
Familial hyperpolyposis endoscopy , should do screen
- wait until he is competent
- do the test if he developed symptoms
- No Need as there is gene therapy now
- perform the test

Answer : wait until he is competent

20 - A 4 yr old boy newly diagnosed with diabetes after DKA , wt


loss . family hx of diabetes , Most important thing to tell to the
family? MODY no wt loss no DKA
- the diabetic could be due to his genetic background

- he will require lifelong insulin treatment
- there is associated autoimmune

Answer : will require life long insulin sure answer one took 100% in
diabetes

All FOP Recall till June 2021 with note 236 of 378
Occupational therapy
- eye hand coordination
- house skill adaptation

21- 15 year old girl irregular menses, brown pigmentation, over


weight, excessive facial hair. Diagnosis?

- Cushing 
 Htn
- PCOS
elements of hyperinsulinsm —>
pigmentation dt acanthodians
Answer : PCOS the majority in Facebook nigricans

22- 5 yr old with difficulty in writing and using scissors. Can’t


kick a ball , Normal neurological examination apart from
generalized mild hypotonia. Whom to refer?
- physiotherapy
- occupational therapy
- neurology should seen by neuro first for proper diagnosis
and exclude organic then refer to occupational
- education

Answer : Neurology ( as per guidelines for hypotonia ) Occupational

23- scenario with difficulty in writing , clumsy , difficult with


walking gait problems
- dyslexia
- developmental coordination disorder

Answer : Developmental coordination disorder ( other name is


dyspraxia )

24- A child with eyelid swelling started on iv antibiotics. After 2


days No improvement.painful and can’t gaze upward .where to
refer?

explanation of scenario
Ophthalmologist 

child start by periorbital cellulitis —> then
Neurosurgeon
complicate to orbital cellulite
Radiology
ENT
Opthal within 4 h —> ENT —> neurosurgeon if
Answer : Opthalmologist suspect brain abscess then need radiology
somewhere in guidelines

All FOP Recall till June 2021 with note 237 of 378
25- Baby delivered at 28 week now 10 days old and difficult to
wean off from ventilator . Has a loud systolic murmur. What’s
your Next step?

Dexamethasone

Furosemide
Prostaglandin E 
 Ibuprofen drug of choice
Spironolactone

Ibuprofen

Answer : Ibuprofen ( PDA closure)

26- Child with undetectable pulse,HR 300/min and narrow


complex tachycardia and stable what is the drug of choice ?

- Adenosine

- Amiodarone

- Flecanid

- Propranolol


Answer : Adenosine ( SVT treatment )

27- 2 years old girl with 6 perianal warts and mother was treated
for warts what is best advice to give ?

- not play with other children till warts gone 

- mostly warts are attributed to sexual abuse

- will spontaneously disappear ??
- chlamydia screen
- HPV vaccination not needed

Answer : Chlamydia screen —> like in pastest .. if it is chlamidia exclude sexual


abuse
chlamydia in urine

28 - 6 hour newborn with sero-perulant eye discharge what is the


cause ?
- Neisseria Meningitidis * < 5 days ; gonorrhea, ttt by cefotaxim
- Chlamydia trochmatis * > 5 days ; chlamydia, ttt by erythromycin oral + eye drops
- Neisseria Gonnorhea
- streptococcal

Answer : Neisseria Gonnorhea

29 - Mental retarded girl with lens dislocation what is the cause ?

All FOP Recall till June 2021 with note 238 of 378
- Marfan
- Homocystinuria 


Answer : Homocystinuria due to mental retardation

30 - 4 years , full time in nursery youngest of 6 brothers , teacher


concern from uncharacteristic behavioral disruption , no concern
at home
- clinical psychology assesment
- audiology assesment
- visual assesment
- developmental
- psychometric test

Answer : audiology assesment

31- child with fever and coryza 1 day


Dipstick
Prot + , Blood +++ , Leukocyte + , Nitrate - glomerular hematuria
What to do ?

- repeat dipstick after 1 week


- give antibiotic for UTI
- refer to pediatric clinic IgA need follow for hypertension
- measure urea , electrolyte , creatinine Measure urea creatinine dt. IgA

- measure albumin Review

Answer : Measure urea , electrolyte , creatining ( could be IgA


nephropathy , need to check hematuria management )

32 - mother receive dimorphin deliver a baby flobby , no resp


effort , covered with thick meconium HR 60 what inintial step ?
- give naloxone
- 5 rescue breath
- suction under direct laryndescope
- chest compression and ventilation 3:1
- Dry and stimulate 


Answer : Dry and Stimulate ( since he asked for initial )

33- ( 12 or 16 year ) boy presented with sudden left severe


testicular pain with swelling , he just came from football what is
your diagnosis ?

All FOP Recall till June 2021 with note 239 of 378
- Testicular torsion
 usually not swelling
- Haematocele
 Sudden painful swelling Review
No erythema
- Torsion of hydated cyst morgagni Scenario of truma
- epidydmitis

Answer : Testicular Torsion Vs Haematocele

34- 2 yr old unable to walk. Symmetrical crawling , Crusies along


furniture. Tone normal. Pull to Stands on tip toe. 
 Gwer’s sign
What is Diagnosis ?
 positive apgar score
- DDH 
 Exclude symmetry
- Spinfa bifida occulta

- Duchene muscular 

- Becker muscular dystrophy
- spastic diplegia
CAH ; more aggressive
Answer : Duchene Muscular Dystrophy virilisation

35- 6 yr girl hight 97th centile , with tanner score 2 of axillary and
pubic hair and 1 for breast cause:

- premature adrenarche
 ?! has positive criteria of centra
- premature thelarche
 puberty
- adrenal tumor
 More aggressive virilisation Review
- precocious central puberty breast stage should more

Answer : Premature Adrenarche

36- Baby 48 hour his billurinin 444 conjugated 17 mother is


scotich blood group O+ve Father is nigerian baby group A-ve
Family history of exageratted jundiced , coombs test -ve
What’s your diagnosis ?
Extensive physiological jundice Mother M N O
G6pd negative O type
ABO incomp. for Rh for ABO

Rhesus isoimunization
Galactosemia
ABO Incompatibility, coombs test can be
Answer : G6pd ABO Incompatibility
positive, weak positive or negative

37- 7 year old boy with 100mcg fluticasone


Which step in asthma ?
- Leukotrine antagonist
- increase 200mcq fluticasone

All FOP Recall till June 2021 with note 240 of 378
- LABA

Answer : LABA Leukotrine

38- 3 year old boy with inhaled fluticasone 200mcg and


salbutamol
But his asthma is not control , what you will do ?
- LABA
- Increase inhaled steroid
- Leukotriene antagonist

Answer : Leukotriene antagonist

39- scenario eye swelling with protrusion , fever , bp 140/85


Diagnosis ?
- hyperthryroid
- neuroblastoma High bp
- langerhans histocytosis
- orbital cellulitis

Answer : Neuroblastoma

40 - scenario with hx of febrile convulsion , what will increase


risk of recurrence ?
- prolong seizure
- family hx of epilepsy
- high fever Low fever around 38
Family history of febrile convulsions + prolong
seizure

Answer : Family hx of epilepsy

41- scenario with one boy develop bouts of cyanosis and cough ,
ttt ? Paroxysmal
- erythromycin Pertussis
- amoxicillin
-
Answer : erythromycin

42 - extensive eczema on the face , on hydrocortisone mother


woried about corticosteroid itchy then bleed
- tacrolimus ointment ( 1% )

All FOP Recall till June 2021 with note 241 of 378
- oral steroids
- oral criprofloxacin for 3-6 month Tacrolimas next after
- oral antihistamine hydrocortisone ?
Answer : oral antihistamine

43- 4 or 5 month not sure of age , breast fed , well baby , found
to have HGB of 58 what to do ? Well baby
- blood transfusion
- oral iron
- change milk
Iron deficiency anemia, below 3 give blood transfusion otherwise give
oral ,, see illustrated
Answer : oral iron
Pseudo ptosis
44- scenario about eye lid ptosis , when asked to raise eye brow
he did it given steroid for 2 days and improve in 1st week , wrinkles intact
presented to you ? Upper motor neuron —> ct
- discharge and if urgent symptom come back
- refer to neurologist MRI
- do CT
- refer to family doctor

Answer : do CT ( since it is upper facial palsy )


45- scenario about fever for 3 days given antibiotic then fever
subsided but there was widespread rash deflect MMR ?
- raseola infantum
- ebv
- measles
- allergy
- rubella

Answer : Raseola infantum

46 - 9 year girl have family issues ( step father come to home ) ,


argue with her mother and doesn’t want to get out of bed , then
when she got out turned pale fall down with twitching of hand ,
then wake up after 2 minutes and she don’t remember except
she was dizzy ?
- reflex anoxic early age toddlers age with trauma No loss of consciousness
- complex partial
- pseudoseizure can remember
- ! syncope history not supported

any child with abnormal movement —> ask for posticeal


* if still not regained consciousness so this is pure seizure
* if no posticteal ie; normal conscious — think for other

All FOP Recall till June 2021 with note 242 of 378
He has infrequent episodes in which he looks frightened, goes pale and
swallows repeatedly for 15-30 seconds, following which he seems tired —>
complex partial

Answer : Complex partial Repetitive move —> not remember —> confused

47 - scenario with a mother having difficult with breastfeeding ,


presented at Friday night , child has bruise on his left cheek ,
mother is a teacher , father is accountant , given oral vitamin K
What to do ?
- Clotting and review result on monday small hospital in UK no social
- phone the social worker on call worker in weekends but big
- admit for evaluation hospital do have social worker
- discharge and come back on Monday on call
Review
NAI
Answer : Not sure of answer Admit for evaluating
- admit
- call senior
48 - scenario of 7 years resp infection not responding to
antibiotic then develop red painful lesion on his chin ?!
- mycoplasma pneumonia

Answer : Mycoplasma pneumonia > 5 year

many source
mentions both ;
49- scenario with scald absence of
Sign suggest of abuse ? splash mark
- absence of splash marks and uniform
- blisters depth burn
- sparing flexures
- clear margin and uniform burn depth
- circumferential burn

Answer : Clear margin and uniform burn depth

50 - one question about syncope after swimming , family hx of his


uncle when he was young and following with cardiology ,
Diagnosis ?
- HOCM
- AS
- Long QT

Answer : Long QT syndrome

51- scenario with female 9 years with 2’ry enuresis and -ve
urinalysis , what is the 1st investigation ?

All FOP Recall till June 2021 with note 243 of 378
- plasma and urine osmalitity
- spacific gravity
- u/s post voiding night & day do U/S voiding for duplex kidney

Answer : Plasma and urine osmolality exclude DI

52- 20 month girl sort of arching of the back , prefuse sweating


Key words sea-saw respiration then she fall asleap 2-3 minutes , not repeat
in the same day , if interupted she will get angry , normal
development
- self gratification
- temper tantrum
- sandifier After feed not repeat in the same
feeling bored .. if
day —> exclude sandyfer awake —> cross of
Answer : Self gratification
leg
Infantile masturbation = benign idiopathic infantile dyskinesia
Pastest same scenario Ttt destruction

53 - scenario of baby 29 weeks ventilated started feed on day 10


then developed abdominal distenstion , unwell , stool with
mucous and blood , what is your 1st action ?
- Stop oral feeding
- call surgeon
- x-ray
NEC
Answer : Stop oral feeding Ngt insertion

54- scenorio with non blanching rash CRT 4 sec , fully immunized
Diagnosis ?
- meningeococcal sepsis
- nisseria meningitis

Answer : Meningeococcal sepsis

55- scenario of 5 month vomiting , blood in stool , breast feeding


what to do ?
- lactose free formula
- extensive hydrolyzed formula
- soya formula
- amino acid formula Cow milk allergy

Eliminate diary products from mother


All FOP Recall till June 2021 with note 244 of 378
Answer : Extensive hydrolysed formula ??

56- scenori of a teenager and when doing urine test next day
they found opiate , what’s your explanation 3 siblings same
- crack cocaine
- ectasy use
- due to lumber puncture
- anti-cough medication containing codiene
- magic mushroom

Answer : Magic Mushroom some solve it by exclusion


anti-cough medication containing
codiene
57- Caucasian , splenomegaly , mild jaundice .
- hereditary sperocytosis

Answer : Heriditary spherocytosis

58- 18 month breast feeding for 6 month then mixed diet ,


passing stool with blood on surface , stool is small and hard for 6
month , what to do ?
Hirschprung since birth
- dietry management has constipation and
- rectal myometry Not do in child case 37 also they don’t have
- colonoscopy hard stool. either no
Answer : Dietry management ( but some say to exclude stool or expulsion of
hurshsprung so rectal myometry ) loose stool during rectal
examination
59 - severe wheeze , urticarial rash , what you will do initial ?
- IM epinephrine
- IM chloramphinarmine
-
Answer : IM epinephrine

60 - weak radial pulse , absent femoral pulse


- coarctation of aorta
- Aortic stenosis

Answer : Coarctation of Aorta

61 - 2 month , rolled from sofa , diagnosis - can’t remember clearly


Answer : NAI

All FOP Recall till June 2021 with note 245 of 378
‫ﺗﻢ ﺑﺤﻤﺪ ﷲ‬
‫إن أﺻﺒﺖ ﻓﻤﻦ ﷲ وإن أﺧﻄﺄت ﻓﻤﻦ ﻧﻔﺴﻲ واﻟﺸﯿﻄﺎن‬
‫اﻟﺸﻜﺮ ﻣﻮﺻﻮل ﻟﻤﻦ ﺳﺎھﻢ ﻓﻲ إﺗﻤﺎم ھﺬا اﻟﻌﻤﻞ‬
‫ﻻ ﺗﻨﺴﻮﻧﺎ ﻣﻦ دﻋﺎﺋﻜﻢ‬

‫‪All FOP Recall till June 2021 with note‬‬ ‫‪246 of 378‬‬
Biliary atresia Growth velocity
* if low INR —> oral vitamin k * first 2 y ==> 25 cm : year
* high INT —> IV vitamin k to avoid hematoma * before onset of puberty ==> 5-7 cm / year
* positive bleeding —> ffp
FOP&TAS, Jun, 2017 * growth spurt ==> 7-12 cm / year

June tas 2017 See survivors p; 282

Acustic .... Pure tone .... ?? Nothing or audiogram Tympanogram any age ?? Constitutional stall stature ...
see equations .. midparental
1-hearing test in new born ...child 6 years old.....child with hearing aid
2-9 years old child grow in 9th centile for the last 3 years ..what is the growth velocity expected next
3-child is over weight ...weight ~99th centile or BMI ~91th centile or weight ~45kg
3-correct prescription of trimethoprim in mg or ml with different conc 8 mg/ kg / day divided q 12 hr 40 mg / 5 ml)
4-child with eczema his mother noticed lymph node in posterior cervical what to do ...reassure and discharge or
reactive lymphadenopathy —>reassurance &
reassure and follow with investigation if persist after 3 months
discharges dt present a cause
5-6 week infant with jaundice and clay stool INR 5 what to do ...IV or oral or IM vit K or fresh frozen plasma or
cryoprecipetate answer vit k IV •INRnormal baby vit k oral or IM • if with biliary atresia not to give oral dt. poor absorption dt. cholestasis • if biliary atresia with high
5 give vit k IV to avoid hematoma • if active severe bleeding with intracranial hge give fresh frozen plasma
| 6-csf with 30 neutrophil and 30 lymphocyte ...protein 1.6 glu 1.4 and 3 weeks history of irritability 6. survival; partially
EMQ treated meningitis
|
| 7- csf done 3 times and RBC 9000 or post head trauma lead to intracranial hge
neut=lymph
Thrombosis lead infarction area easy to oozing and bleeding
8-chid 7 days fever with couph and red eyes
9-vancomycin overdose after 24 houre in 10 days infant 24 hours
* red man $ .
10-vancomycin mech of action * nephritis Later
* hearings loss
11-omeprazole mech of action * Phlebitis
* Steven Jonson
12 -y descent in jagular venous pressure is for opening of what ?
13-second heart sound is represented with what in ECG T wave
14-child with burn with sever pain and screaming Morphine IV
EMQ 15- child with head injury and fractured femur .in sever pain nerve block avoid opioid in head trauma
16-meningococcal prophylaxis in 1 month old child ...and girl taking COP ,,,and pregnant female 16 • all age ciprofoxacine
• girl with contraceptive pills
17-chemotherapy SE abdominal pain...glucosuria...constipation Dexamerhasone .. --> ciprofloxacine
vinicrestine
18-infection ass with T cell dysfunction Di George not to give rifampicine
Nisseria in C5-C9 • pregnant --> ceftriaxone
19 -infection with complement dysfunction •one month --> ceftriaxone
20- gram -ve bacilli.....gram +diplococci....gram -ve cocci
21-emberyological defect in digoerge syndrome 3,4 pharyngeal pouch
22-element in cytochrome oxidase help in utilization of iron Vit . C & acidity Increase absorption
decrease absorption tea, anti acid, poi
23-help in ca absorption from kidney and gut Iliac resection
frequent seizures fat soluble vitamin ADEK
24-child wih macrocytic aneamia and rickets and bruises and dry skin what is the operation lead to this vit B 12
25-15 years old girl with paracetamol toxicity level at 4 hours is below threupetic treatment she denied any prop problem
ask if she pregnant, as giliak she is admit and referred for
lem at home ..ask her if she is pregnant or do pregnancy test or discharge competent for consent social & CMHC
or refer to psychiatry
family, health committee -> if
26 -brain dead child and his family refuse removal of live assisstence who can consent arguing ethic committee
27 child need appendecectomy come with his grand parents who will consent consent by senoior, consultant .. inform mother by phone
28-newborn his mother has bruise around her eyes and older sibling in social service records what to do ->Social services
If acute harm ->
29-DKA you give saline bolus what to give next Insulin infusion call police
30-hormone deficiency in DI
31-light reflex when light to LT eye RT contract and LT NOT CHANGED when light to RT eye RT contract an
d LT unchanged
32-where the defect in homonymas hemianopia ...bitemporal hemanopia ...lower quadrant homonymos hemiano
pia ...TAKE CARE IS IT RT OR LT LESION
33-statistic was 7 question 2 is direct and rest is strange
34-7 days old boy with glucose 1 and insulin 15 if insulin high —> do C-peptides if high insulinoma.. if low exogenous
35-First investigation in celiac dse IgA tTG insulin

36-DNA part which contains genes... And the part separte exons not contain genes.... Cell have half of chromos
ome Valproate inhibitor
Refampicine inducer

All FOP Recall till June 2021 with note 247 of 378
37- Child on penicillin prophylaxis from 4 months come with sever tooth decay the dentist said this because of p
enicillin what should u do?? stop drug or change to sugar free penicillin or ignore dentist opinion ethically don’t ignore
38-Pathophysiology of myasthenia gravis there is no option for anti ACh receptor antibody i shoose decrease in
motor nerve conduction
39-case about treatment of the EEG choatic appearance > infantile spasm ACTH or
vigabatrin ?? decrease dose ..
EMQ 40-tention headache and take paracetamol and morphine no improvement ..what to do next analgesia not more
3 time/ week
41-child has fever 39 and his limbs shake stop when you hold them ...mainly rigor
42-monoclonal antibody for res sensitial virus and TNF and IgE
43-child with itchy rash and swelling around eye completely disappear next day his mother has swelling around
her eyes when exposed to pollens Allergy .. atopic dermatitis
44-lamotrigene side effects skin rash up to SJS, renal stone
45-test supporting diagnosis of rheumatic fever ASOT
46- Effect of 6th Nerve lesion (horizontal diplopia)
47-drainage site of coronary sinus .....ductus venosus...
45-EMQ about lung function interpretation and diagnosis
46-EMQ about cardiac catheter diagnosis
47-child with hypothyroidism on thyroxin replacement and get high dose by mistake what is the clinical sign of
toxicity
48-short stature ..what is initial step...midparental height
49-child 4 month with abnormal head shape prominent RT parietal and LT occipetal Plagiochrphaly
50-mode of inheretance in case of prader willi....myotonia ....rickets in child and her mother have genu valgum
X linked dominant Hypophosphatemic rickets
51- what is the defenition of genetic imprenting
52-child need pyloric stenosis surgery his mother on methadone for opioid withdrawl program and both child an
d mother in foster care.....who can give the consent Mother
53-DRUGS ,,,Ca antagonist ....B1 antagonist ,,,,non specific adrenoceptor agonist
54-drugs ,,,,anti proteinuria ....potent vasoconstrictor
55-child vomiting And hypokalmic hypo chlormic alkalosis I think pyloric stenosi loss of acid from stomach
56-child need urgent blood transfusion his parent told u that his blood group is A +ve ...what type of blood grou
p u will use O -ve
57-case with heamolytic ureamic syndrome what u will find in blood film
58-what is the difference between renal and pre renal renal failure
59-drainage site of CSF Subarachnoid space
60-nitric oxide actions Cyclic Gmp
61-LT eye ptosis with normal eye movement and normal equal pupil size...
62-Ramsay hunt syndrome...type of lesion Lower motor neuron lesions of cranial n from hepes lesions
63-Exitatory neurotransmiter GABA, glycine are inhibitory .. the rest are excitatory

64-pathophysiology of ITP
65-neonate with hypoxia improved after O2 head pox cause? ?? respiratory cause Illustrated 329
66-mech of hypersensitivity after peanut ingestion IgE
67-Layer affected in staph scaled skin syndrome
68-Child with UTI prophylactic drug & developed jaundice G6PD
first step ---> bilirubin total&direct
69-Case of neonatal jaundic & pale stool .. Most likely biliary atresia .. What is first investigation most accurate ---> US or HIDA
70-cyanotic spells of fallot ... Increase systemic resistance terminate it morphine to relieve pain and to
release spasms of infundibulum
71- SSSS given ceftriaxone why need to add clindamycin works as antitoxin
72-Whole class having sever watery diarrhoea with presence of ova in stool. What is the organism Giardiasis
73-Mechanism of anemia in parvovirus

All FOP Recall till June 2021 with note 248 of 378
74-About the 12 years old child with hyperglycemia .. His father & grandfather had DM when they were 22-25y
ears... Mechanism of this MODY non obese
75-DKA...what increase in the blood....ketone Post surgery septic shock
76-child after appendectomy increase heart rate and prolonged capillary refill...what is next step give bolus for shock

77-case with hyperammonia & normal glucose UCD mainly if no acidosis.. may OTC
anabolic to muscle ( growth hormone, insulin )
78-what is inhibiting protein catabolism
for biliary atresia; medium chain triglycerides,
79-diet after kasai operation high protein, fat soluble vitamin.
80-Collapse after playing football.. Investigation long QT syndrome for ECG
81- Hirshsprung.. The deficient cells ganglion cell
82-Splenomegally & dilated veins on anterior abd wall.. Site of lesion .. Portal vein capital medusa in portal HTN
83-Protenuria & haematuria after throat infection .. Post strept glomerulonephritis.. What is the finding of immu
noflorescence microscope diffuse
84-there was a pedigree and want mode of inheretence
85-case with foul smelling stool undigested vegetable will child and open her bowel 3 times daily grow on 9th c
Normal percentile, significant
entile ...management low at 2 percentile
86- case with soiling and since he start school and start to refuse also to do at home.... afraid of school toilet
87-Hand preference at 9 months with normal development her father use the same hand
88-Case of absence seizures and ask about ch.ch EEG finding
89-Best measured by parametric tests. * hypsarrythmia = chaotic —> infantile spasm
* centretemporal —> rolandoc
90-Definition of obesity .. option obesity diagnosis * 3 hertz spike —> absence
Wt > 99th centile
BMI > 91st centile BMI > 30
cant remember the rest
91-EMQ Asthmatic child on high dose of steroid inhaler. .not controlled. .appear less than his colleque is same
class and also Asthmatic.
Ans :do short synacyin test
92-child have stony dullness in lt lower zone of chest .. chest us
93-baby with doubtful dislocation dislocation of hip what to do ?
94-Child present with limping and failure to abduction of hip following sport perthe disease
95-14yrs girl has obesity and tall for her peers, with breast development stage 3
what to do ?
MRI Brain
Do lft.lipd profile.
life style and wt management Correct answer For her obesity

refer for assessment by dietician


96-Child sizable. .before discharge nurse discover the is no movement in rt arm..what you do? clavicle X-ray Erb’s palsy
97-Child with severe eczema interfere with his daily life mother keep dairy milk what to-do next ?
1% hydrocoticosone or topical tacrolimus ????? Use of tacrolimmus should be after use of cortisone and age more than 2 year
98-EMQ hearing assessment
neonate ... brain evoked potiential
5 or 6 years PTA
HEARNING AID ... either speech discrimination or parents questionarre but i am not sure

99-girl with short stature and learning disability what investigation to-do
Karyotype for turner
100-EMQ Hematology

All FOP Recall till June 2021 with note 249 of 378
baby taking cow milk .. serum ferittin or iron and TIBC for iron deficiency anemia

baby with HUS blood film look for fragments cell


baby and his sister have pica lead level or abd X-ray
101-bby hve testicular enalargment nd inguinal lymph node …. ALL or testicular malignancy ???
102-Child presented with pallor and abdominal distention what investigation to do ?
Celiac tissue trans-glutaminase IgA
103-4month presented with fever and vomiting weight loss, what sign if there make urgent refer is needed .... i c
hosse drowiness ... but not sure Pallor , skin turger,
tachycardia, tachypnea
104- BOF case og polyuria and polydispa wt loss ... investigation ?? Blood glucose
105-BOF of child with whezee with infection ,, family hx of atopy and asthma ,,, clear chest ... i choose reassure
this is viral induce wheeze
106-CP child with gastrostomy NA 190 normal K ? I choose Fabricated illness ????

All FOP Recall till June 2021 with note 250 of 378
FEB 2017 FOP

Some notes :

# The exam was not that easy. Questions were full of tricks and
misleading points.
# The time was very short so we couldn't easily read all case scenarios
and answer options.
# When I say here a case of turner syndrome for example, it doesn't
mean he said turner syndrome in the exam. It means he provided a case
scenario of 3 to 4 lines so at the end you can conclude this is a case of
turner then you can choose the right answer related to it.
# Time is your worst enemy in the exam.
# Guidelines are more than important.
# RCPCH website sample papers (old and new) are more than important.

# These were important points for every future candidate.

- Bilateral firm breast masses in 18 month girl with normal


development .. what investigation to do.
- This is premature thelarche, usually transient ... chose reassurance

- Boy with history of measles before 10 days, convulsed today for


more than 10 mins, relieved after buccal midazolam but he is still
unconscious 2 hours post ictal with no focal neurological signs.
What to do ?
- Answers included IV Measles immunoglobulin / IV flumazenil / IV dexa /
CT or MRI
- Mostly correct = CT or MRI ADEM

- 10 years boy with night terrors. What to do ?

All FOP Recall till June 2021 with note 251 of 378
- I thought 10 years is quite old for night terrors as it typically lasts to pre
adolescence so I chose referring him to CAMHS. But after some
search it seems to be normal for that age. - Answer = reassurance

- Most serious painless complication of incarcerated inguinal


hernia
- I chose intestinal gangrene ... but painless makes it little hard to be the
correct answer ... other answers included testicular atrophy (?) //
intestinal stricture // femoral vein thrombosis.

- Baby 2 months with spiral humerus fracture. What to do ?


- No options for social services or calling the senior. I chose skeletal
survey.
If choice CT brain —> choose it

- A grandmother with her 16 year old daughter who presented with


her infant for vaccination.The grandma never vaccinated the 16yr
old mother. What to do ?
- Answer = Vaccinate both the mother and the baby ( as she can
consent for herself & she is responsible for him whatever the
grandma's opinion)

- mother developed chickenpox 5 days after delivery. what to do to


the infant ?
- VZIG + come back if rash develops

- Feverish, Bullous rash, widespread, hyperemic surfaces when


exfoliated. Mucosal surfaces not affected. Sterile culture.
- Answers = epidermolysis bullosa, TEN, SJS, SSSS
- Mostly = Epidermolysis Bullosa

- Varicella case diagnosis (can't remember history)


diagnostic test PCR

All FOP Recall till June 2021 with note 252 of 378
- Neonatal + varicella rash … ttt
- answer = acyclovir IV 7 to 10 days

- Case with rash, papular with umbilication/depression on top.


- Moescum Contagiousum

- A boy at ER, always goes to school alone, presented with GCS


12/15 and neck stiffness.
- Answers included subarachnoid hge / extradural hge / cerebral artery
occlusion I think - Mostly = subarachnoid
Meningism sign

- A male adolescent presented to ER with coma, Alcohol smell from


his mouth, facial ecchymosis and bruising, unequal pupils... he
was alone in the street. (Seems to be severely hit by thieves or
something).What is most probable cranial hemorrhage ?
- Answers Included subarachnoid, extradural, intra ventricular .. as I
think.
- Not sure of the right answer.

- Case of Eczema with 2ry bullous impetigo bacterial infection and


unwell… ttt ?
- chose IV Flucloxacillin

- Baby changed from breast to formula then developed severe


eczema and diarrhea and lost weight.
- Answer = Cow milk Protein Allergy

- Case with history of mild eczema but good feeding, then


developed diarrhea in last 24 hours.
- Mostly a case of rotavirus

All FOP Recall till June 2021 with note 253 of 378
- 2.5 years with loose stools with undigested particles. - Toddler
diarrhea

- Smiled at 8 weeks, Laughed at 3 months, normal head


circumference. But mother complains he does not follow objects
with his eyes. What is the most important test to do ?
- The scenario was trying to make sure the girl is developmentally
normal and not a case of GDD or microcephaly.
- Answer was testing red reflex

- Newborn with conjunctivitis developed about 60 hours after


delivery (I think), not responded to 3 days topical
chloramphenicol.
Organism ?
- Mostly Chlamydia V/S GONOCOCAL

- A Boy with episodes of loss of concentration frequently at school


(not witnessed at home) but parents say he usually forget what
they ask him to do.
- Day dreaming--

- Patient with abnormal uprolling and movement of eyes. Received


Metoclopramide since days.
- Oculogyric crisis of metoclopramide

- A girl with attacks of palpitation and tachycardia 3 times within 6


months after exercises. What to do ?
- Answers included 12 lead ECG / 24 Hour ECG / Exercise ECG ... -
chose Exercise ECG by cardiology

- They provided a table of blood sugar level during the day.


Readings showed that levels are normal (6-9 mmol) at the morning

All FOP Recall till June 2021 with note 254 of 378
and before launch meal, then they get much higher (~20 mmol)
before and after evening ---meal. Asked how to manage ? -
chose to increase the launch and evening doses of short acting insulin

- Case with Hyperglycemia + Met acidosis + tachypnea +


tachycardia + CRT 3 + Sodium 129 . First step ?
- Answer = Normal Saline 10 mL/kg
- Other answers included 20 mL/kg saline but according to NICE and
BSPED Guidelines only give 10 mL/kg in shock and if need more =
consult senior.

- Unilateral flank pain + radiating to groin + urine analysis shows


RBCs above 100 & wbcs 5 + history of fever I think and few small
cervical lymph nodes Pain in flank + hematuria ==> stone

- This is the typical pain of renal stone (Calculus)


- Other answers included HUS / PSGN / PYELONEPHRITIS / HSP

- A spirometry readings of a patient : FVC = 95% ,,, FEV1 = 65%,,,


FEV (25-75) = 28% (or similar range of values). What to do next ?
- This is mostly a case of small airways obstructive disease like
asthma.
- chose to repeat the test after beta agonist inhaler

- A patient receiving ttt for malignancy with significant neutropenia,


he is on paracetamol and ormorph as PRN. What to do for his
uncontrolled pain ?
- Answers included S.C morphine infusion // Oral slow release morphine
twice daily // transdermal patch

- A 2 months baby with UTI and culture shows E.coli. investigation


?
- Answer = U.S within 6 weeks
- (this is a case of simple UTI and this is the investigation of choice for
his age according to NICE guidelines)

All FOP Recall till June 2021 with note 255 of 378
- Joint pain with history of a school epidemic of gastroenteritis.
Diagnosis ?
- Reactive arthritis

- Case with bone aches in joints and shins specially at night. ESR
90 , normal WBCs and Hb. A brother with psoriasis.
- Options included psoriatic arthritis, SLE and ALL .. other options not
satisfactory. - Chose ALL May only present with joint pain, and more liable if associated
with ( first degree relative with psoriasis, dactilitis & nail pitting

- A 8 year old with limbing and knee pain after karate lessons. Knee
examination was free. Best diagnosis ?
- chose "Perth disease"

- Obese + 12 years + limbing


- Slipped femoral capital epiphysis

- Small child with hypoglycemia in ER , received IV dextrose and


now active and playing. Ketones are negative. Mother is a nurse
and father a lawyer and diabetic patient. The mother says he got 2
episodes of apnea Last year
- All options were not satisfying.
- chose Fabricated illness.
- same quesion in sample baber aswer was reactive hypoglycemia
which occur after meal What to do —> prolong glucose tolerance test

- A 5 year boy. Teacher complaint of his aggressive behavior. Boy


gets calm at drawing lessons but angry at group activities. He
maintains friendships in the playground. Not in ASD
- Scenario wanted to make sure this is not a case of ASD or ADHD
- chose Secretory otitis media (hearing loss)has problem with high
frequency sound

- ttt of a case of suspected Autism and learning disorders..


- Educational therapy (?)

Behavior education therapy

All FOP Recall till June 2021 with note 256 of 378
- ttt case with poor performance after family stress. He was a
always a good achiever with high marks.
- Family therapy (?)

- ttt of 13 year female with bullying activity ... she has been always
normal before
- psychotherapy (?) Cognitive behavior therapy
SOP

- Child with joint pain, one week later became hot, swollen tender
and painful on passive and active examination. X-ray was free.
Baby was looking well but I think feverish. Best next step ?
- Options included ultrasound and fluid aspiration .. not sure of the best
answer. In septic arthritis; may all CBC CRP ESR normal, X-ray &US normal —> only aspiration
will differentiate between reactive arthritis (cloudy) & septic arthritis (purulent)

- Case with manifestations of gynecomastia, small testicles, tall


stature.
- Kleinfilter

- A 11 year girl showing signs of 2ry sex characters started 2 years


ago. Cause of the early appearance ? - Normal puberty

- A girl with Acne + few pupic hair + normal clitoris ...


- CAH was the most accepted answer Premature
adreanalche

- Another case with signs of puberty ... Mostly was central


precocious puberty

- A tall boy + obese + advanced bone age.


- Cause = simple obesity (exogenous)

All FOP Recall till June 2021 with note 257 of 378
- Infant with poor feeding and fever. Culture urine showed UTI. Now
treated for UTI and feeding improved but developed jaundice. Best
explanation ?
- G6PD

- A pregnant says her uncle died at age 1 year because of an


immune disease. Then the son of her aunt's daughter died
because of immune disease. She wants investigation for her
unborn baby.
- Answer = tell her she can be investigated for carrier state.

- Neonatal Resuscitation. Now chest is rising but HR below 60. Best


next step ?
- Chest compression (see ERC Neonatal resuscitation guidelines)
Inflation for 30 seconds —> if not improved —> intubation + compression

- 7 month baby pale with splenomegaly. Diagnosis ?


- Best option was H.Spherocytosis.

- A case of ITP after viral illness =

- A case of Aplastic a. (Bleeding and anemia)

- Breast fed baby 3 weeks, lost weight after delivery, icteric, serum
sodium 150. Diagnosis ?
- Poor feeding

- Formula fed baby, increased weight after delivery. Now persistent


vomiting.

All FOP Recall till June 2021 with note 258 of 378
- He was 3.7 kg and receiving 125 ml per feed every 3 hours
- Answer = over feeding

- Baby with Intermittent bilious vomiting. Pass stool normally. Soft


non-distended abdomen. Diagnosis ?
- Options were between malrotation or Hirschsprung (?)
-
A case with acute asthma. Receiving nebulised salbutamol since 2
hours (I think) now cyanosed, drowsy, can't talk.
- Answer = call anesthesia
- revew asthma guideline

- A girl took 18 tablets of paracetamol un-witnessed last of which


was 5 hours ago as her parents sow her. She is not cooperative
and refuse to speak. > 16 tablets —> toxic dose

- Options included oral NAC or measure her level now or wait 4 hours
then measure (something like that)
- Answer = Intravenous N-acetylCysteine now

- Adolescent girl. Presented with her friends who were in a party.


They say she suddenly began to shout and make abnormal
movements or something then collapsed on the bed. She is now
non-oriented, closing her eyes and moaning.
- Best options were Postictal vs MDMA toxicity. (chose postictal .. Not
sure of the right answer) pusodosizure

- A 3 weeks neonate with fever 39c° , crying , irritable, feverish.


Best nest step ?
- Full septic screen + IV broad spectrum ABs

All FOP Recall till June 2021 with note 259 of 378
- A case of infant , feverish.
- Can't remember details but Best option was UTI.

- A case of anaphylaxis ... best next step ?


- IM adrenaline

- What is the cardiac sign that always indicate a pathological


cardiac lesion
- Thrill

- A case with Ambiguous genitalia. What to tell the parents about


his gender ?
- Can't remeber options exactly but was about postponing naming the
infant till some investigations.

- PDA cardiac sign ?


- Bounding pulse

- TGA cardiac sign ?


- Central cyanosis

- WPW cardiac sign ?


- Rate around 250 Delta wave , short PR , wide QRS

- A case with vomiting at 1 month. History of uncle went a GIT


operation at infancy. Mode of inheritance ?
- Case of pyloric stenosis
- Answer = multifactorial

All FOP Recall till June 2021 with note 260 of 378
- bottom shuffler ; hypotonia
- spina bigida ; hypotonia , reflex less
- myotonic dystrophy ; hypertonia , normal reflex

- A case with pale, loose stools


- Biliary atresia

- A case with features of turner.


- XO karyotyping

- A case with features of Duchene


- XLR

- Another case with signs of Duchene and it was the diagnosis :/


normal reflex, mild hypotonia

- A case with low phosphorus, low normal calcium, low vitamin D


and high PTH. Best ttt ? - Vitamin D
- Other options included 25 OH Cholecalciferol ,,, Oral phosphorus,,,,
oral calcium

- A case inpatient, convulsing since 2 mins, feverish. Best next step


?
- Options included : reassurance,,,, insert cannula ,,, buccal midazolam
,,,, antipyretic
- Mostly = reassurance (refer to CKS NICE guidelines for febrile
convulsions)

- A case showing signs of Kawasaki ... most important ttt ?


- Oral Aspirin was written in answers .. but chose IVIG

- A girl with antithyroid antibodies. Best combined ttt with


carbimazole ?

All FOP Recall till June 2021 with note 261 of 378
- Mostly propranolol. Not sure

- A case with low grade fever and cough


- Bronchiolitis

- A case with high grade fever, wheezes and crackles.


- bronchiolitis

- A case with positive screening test for PKU .. what next ?


- Measure phenylalanine level in blood.

- Case feverish + tachycardic + tachypneic + low pulse volume +


hepatomegaly.
- Answer = myocarditis

- A case with diffuse headache without vomiting


- Tension headache

- A case with headache, vomiting, abdominal pain - Migraine

All FOP Recall till June 2021 with note 262 of 378
- A case with headache mainly at the morning and vomiting -
Brain tumor

- A case 10 years with painful red scrotum.


- Testicular torsion

- A baby changed from breast to formula the lost weight


significantly. Next ?
- Best answer was to start hydrolysed formula

- A case I think with precocious puberty, normal random and


postprandial glucose level and also cholesterol level. Best
diagnosis ?
- Answer included brain tumors I think - But mostly = Polycystic ovary.
- Polycystic ovary syndrome (PCOS) is a heterogeneous familial
disorder characterized by chronic anovulation and
hyperandrogenism. This multi-system, polygenic, multi-factorial
disorder is associated with an increased risk for metabolic
abnormalities such as type 2 diabetes mellitus. Signs and
symptoms of PCOS often emerge during the peri-pubertal years
with premature pubarche (PP) being the earliest manifestation for
some girls. Insulin resistance and hyperinsulinemia are important
pathophysiological features that are common to both PP and
PCOS

- A case with soft palatal petechiae, cervical lymph nodes -


Answer = IMN

- A case girl stays at her room all day. Not meet friends. Not eat
with family. Lost weight. Didn't refuse to take the urine toxicology
test which was negative. Diagnosis ?
- Depression

All FOP Recall till June 2021 with note 263 of 378
- A case with meningitis. Best prophylaxis for contact brother ?
- No options for cipro
- chose Rifampicin

- A case with constricted pupil and bradycardia, bradypnea. -


Methadone toxicity

- Boy with difficulty in writing and some motor activities -


Dysprxia

- When to refer a child for developmental delay ? See FOP cases

- Many options but mostly was frequent falling by 18 months. For red flags

- A case with ear pain, fever, unilateral postauricular mass pushed


backwards and down. Best investigation ?
- Seemed to be mastoiditis.... options included US for the swelling / xray
mastoid / CT / Ear swap
- chose CT
- Diagnosing mastoiditis Your GP will examine the inside of the ear
with an otoscope (a device with a light and magnifying glass). If
your GP thinks you have mastoiditis as a complication of a middle
ear infection, they'll refer you to an ear, nose and throat (ENT)
specialist for further examination and tests. This usually includes
a blood test and an ear culture (where discharge from the ear is
tested for a bacterial infection). Some children may need to have a
CT scan, which uses X-rays and a computer to create detailed
images of the inside of the skull.
from NHS guidelines

- A case of face eczema + small postauricular mass. Best


investigation ?
- Reassure and follow up after 2 weeks

All FOP Recall till June 2021 with note 264 of 378
- A case with tonsillitis. A history of penicillin allergy. Best
alternative ?
- Options included different cephalosporins If child on AED —> avoid
- chose Clarithromycin erythromycin ( enzyme induced )
—> choose cephalexine

- A 14 yr Boy. Doing much excercises since age of 11. Now lost


weight and on 25 centile. - mostly anorexia nervosa.
BMI 17.5 kg / m2

- A case of a girl with seasonal allergic rhinitis. She wants a rapid


medication for her nose as she has exams.
- No options for intranasal antihistamine
- Options included oral cetirizine or intranasal beclomethasone ...
beclo seems to be the best answer.

- A case of UTI with dipstick showing +ve leukocytes and nitrites ..


a febrile and good general status … best treatment ?
Mostly = Trimethoprime. now co amoxclav for upper
lower uti cephalxin

- A case of 12 month baby with improper weaning and looks pale.


Most possible type of anemia.
Mostly = Iron Deficiency.

- A case of celiac disease (can't remember full details)

A case with inspiratory stridor started last night with a significant


tracheal tug … diagnosis ?
- options included viral croup & FB
- mostly = FB Abrup no corysa

A case of diarrhea since birth and failure to thrive. What is the most
important reason for referral of this child to a tertiary care for
investigation ?

All FOP Recall till June 2021 with note 265 of 378
- Options included : recurrent chest infections …or... height and weight
on 2nd centile…or...presence of undigested food
- Mostly = recurrent chest infections

A case of JIA .. parents are discomfort after being told that their
child will be transferred to another department. What is mostly that
department ? - Ophthalmology

A boy fall off his bike the previous day. Now presents with
asymmetrical scrotal swelling.
Answers included : hydrocele, testicular torsion, scrotal hematoma,
varicocele.
Mostly = hematoma

- A photograph of a case with extensive blue marks on the back.


They say it's only on the back not on the trunk or limbs.
- Diagnosis = Mongolian spots

): ‫دعواتكم‬

All FOP Recall till June 2021 with note 266 of 378
FOP OCTOBER 2016 Others
1/Child present with painful left knee, paracetamol doesn’t help but ibuprofen help. H/O
travelling to France 5 weeks ago. O/E he is afebrile. Diagnosis?
a. Rheumatoid arthritis.
b. Juvenile Idiopathic Arthritis.
c. Lyme disease.//??
d. Leukemia.
e. Septic arthritis.

1) 2 years old, previously healthy apart from febrile convulsion at age of 9 month, present for
hernia repair and has convulsion before the repair. Diagnosis?
a. Idiopathic epilepsy.
b. Hyperinsulinism.
c. Hypoparathyroidism.
d. Medium-Chain acyl-CoA dehydrogenase deficiency.
e. QT syndrome.

3) Child known to have frequent viral infection associated with cough and wheeze. He is playing with
pets at home. He is on PRN Ventolin, present with S.O.B, cough and fever. Your advice:

a. Inhaled corticosteroids.
b. Remove pets from house.
c. Skin allergy test.
d. Antibiotics for every viral infection.
e. ?

4) 10 years old girl brought by her mother, which is a sex worker, with green discharge appeared on her
pant. She refused examination and just ask for a medication, you suspect:

a. Inconsequential sex contact.


b. Consequential sex contact.
c. Sex contact with a girl. (The same ?)
d. ?

5) Preterm, 32 weeks, baby with RDS, post M.V. CSF: E.coli meningitis. Long-term complication:

a. ROP.
b. Chronic lung disease.
c. Hydrocephalus.
d. Deafness. ?
e. ?

All FOP Recall till June 2021 with note 267 of 378
6) 11 years old Down syndrome patient, asymptomatic, present to your clinic for regular check-up. O/E
looks well, weight 0.4 centile. As a screening, you should order:

a. Thyroid function test.


b. Coeliac screening.//????
c. Blood sugar.
d. X-ray spine
e. Blood film.

7) A known case of ALL on maintenance chemotherapy, following with you and receiving the
chemotherapy in a tertiary center. The specialist nurse call you from the tertiary center telling the
neutrophils count is abnormal and I think adjustment of the dose is required. Your action:

a. Call your consultant.


b. Recalculate the dose and make sure of it and agree with her.
c. Ask her to send a fax about her concern.
d.
e.

8) 3 years old child not sleeping well at night because of umbilical hernia pain. Your action:

a. Reduce and fix it.


b. Get surgical opinion.
c. Reassurance.

9) Your consultant prescribed methotrexate as a treatment for RA patient. The parents read on net that
this drug should prescribed only for malignant conditions. Your action:

a. Tell them as it prescribed by a consultant you should take it even if it is not licensed.
b. Refer them to rheumatologist. ?????
c. Convince them to take it.
d. Ask them NOT to take it.
e. Refer them to social worker for education.

10) You should refer the child for further developmental assessment if:

a. Not sit unsupported at 8 month.


b. Not babbling at 6 month.
c. Walk at toe at 18 month.
d. Convergent squint at 6 month. ???

11) Best drug if given intrapartum reduce the risk of HIV transmission:

a. Zidovudine. ??
b. Nevirapine.

All FOP Recall till June 2021 with note 268 of 378
c. Lamivudine.
d. ?

12) Neonate with TSH 180 and no uptake in thyroid region:

a. Thyroid aplasia.
b. Dyshormonesis.
c. ?
d. ?

13) 15 Years old present after 20 paracetamol tablet intoxication, you told her to wait for 4 hours to do
Paracetmol level but she escape. You called her home and parents state they don’t know where is she.
Your action:

a. Ask hospital security to find her.


b. Call child protection registrar.
c. Call police to look for her.
d. Tell parents to ask her to come to hospital when she returned home.
e. ???????????

14) A suitable way to confirm NGT position:

a. Inflation and auscultation.


b. Gastric aspiration.(guideline neonatology)
c. CXR.
d. ?
e. ?

15) Neonate found to have bilateral conjunctivitis and hepatosplenomegaly:

a. HSV (for conjunctivitis - but not common to have HSM :/ )


b. CMV
c. Rubella
d. ?
e. ?

EMQ1: Nutritional deficiency

a. Vitamin B6
b. Vitamin B12
c. Vitamin K
d. Vitamin D

All FOP Recall till June 2021 with note 269 of 378
e. Vitamin C
f. Zink
G. ?
H. ?
I. ?

1. Neonate with seizure. B6


2. Neonate with gum bleeding. Vit K
3. Neonate with broadening of ulna. Vit D

EMQ 2: End stage cancer on methotrexate???????????

a. Decrease the dose of morphine.


b. Decrease the dose of methotrexate.
c. Increase the frequency of morphine
d. Increase the dose of methotrexate.
e. ?
f. ?
g. ?
1. Need morphine more frequently than his plan.
2. Difficulty of swallowing== Decrease the dose of methotrexate
3. ?

EMQ3: DM management

1- 14 Y/O female diagnosed 6 months back with DM and she is on long acting insulin at night and short
acting insulin before meal. RBS at 2 am= 2.9 mmol/l and at morning before breakfast 10 mmol/l.

(Decrease LA insulin or decrease tea-time short acting insulin --- or even later time dose of the short
acting one).decrease night short acting

down ==hyperglycemia in mornining =increase lantua (long acting)

somoggy==hypoglycemia midline and hyperglycemia in morning decrease long acting

2- Teenager well controlled diabetic has infection now RBS=21, urine ++ ketone

(? Admit for IV fluids and insulin infusion or give extra dose of insulin ... as long as she is not
vomiting/abdominal pain and no ABG available).

3- 4 years-old child with fever 39 and vomiting after supper as well as abdominal pain and nausea, RBS=
6, urine no ketones → increase fluid intake or decrease insulin dose.

All FOP Recall till June 2021 with note 270 of 378
June 2016 exam
mixed fop+tas

1. neuromuscular junction receptor >> nicotinic acetylcholine

2. hip dislocation:
1.internal reduction 2. external reduction 3.double pampers*

3. changes in fetal circulation


4. Hb f best of 5 =
work on 2,3DPG and decrease it so shifting the curve to Left and increasing the affinity to
oxygen

5. Hbc,, Hbh ,,spherocytosis ,,,deletion


note
2 or 1 deletion of alpha chain, = alpha thalessemia trait
3 deletion of alpha chain, = HbH disease
4 deletion ==Hb part (alpha thalathemia major)
2 deletion of beta chain, =
B. major
1 deletion of beta chain=
B .minor
electrophoresis, = HbC , HbS
fragility test = spherocytosis EMA TEST(now EMA test is diagnostic)

, abnormal Na channel =???


deravate disease has abnormal NA channel(Mutation)

6/ statistic: bias information bias,


research from the clinic, =sampling bias
research without publication, = reporting bias
research the forms where not completed =artesian bias -late
look

7. bof diarrhea polyhydramnious >> bicarbonate 28, ph 7.5, cl 89 low Na low,


*cl loosing enteropathy (if there is polyhydraminous with hypo chloremic hypokalemic metabolic
alkalosis this fit with barter syndrome)

All FOP Recall till June 2021 with note 271 of 378
8. child has be well until early no fever bruising around the mouth / bleeding from
mouth , low GCS , bruises in abdomen
?? NAI
9. abnormal eating habit ( anorexia - bulimia ,, etc )

10. what is the way that children metabolise paracetamol - *sulfation(in neonate)
gluconridation in children

11. . which layer in epidermolysis bullosa symplex >> 1/ junctiona layer with scar,
2/junctional layer without scar,
3/*intraepidermal without scar,4/ intraepidermal with scar

12. pathophysiology of VSD: *1/hyperemic lung lt ventricular over load ,


2/oligeromic lung with rt ventricular overload,,,3/ increase pulmonary blood pressure,

13. *experiment to know the cause of increase MRSA: case control, cohrt,
*randamized control trial
Notes : Observational - prevelance and diagnosis = cross sectional
- common disease , causes , risks , prognosis = cohort
- rare diseases , causes , risks = case control
- association in population = Ecology
14. statistic: experemint 4 CI most statistical seqnificant
15. defenitions :
16. which of the following should measured in anaphylasis ( insect bite):
1/histamine, *2/mast cell tryptase,3/ IgE,

inculation and contarst and unknown ellergene= tryptase is specific


17. antibodies : - responsible of villous atrophy in celiac disease, = anti-tissue transglutaminas
IgA

18. - neonate delivered by CS due to bradycardia which persist after delivery, = anti Ro

- teenager malar rash arthritis red urine = anti douple stranded DNA

18. evoked auto acoustic emission response: 1/*choclear, 2/echo,


3/high amplitude voice ^

19. the test for obstructive sleep apnea: 1/paradoxical movement of


the thoracoabdominal,
*2/desaturation,
3/wake up more than 10
times

All FOP Recall till June 2021 with note 272 of 378
20. aortic arches Note :-:
6 branchials , 1 , 2 , 5 dissappear
3 left and right carotid arteries
4 letf - aortic arch , right -,,,,, rt brachiocephalic and rt subclavian
6 left and right pulmonary a. and ductus artrusus,

21. bilirubin characteristic of bilirubin.. phototherapy mechanism


phototherpy will convert the unconjugated to water soluble ^

22. SMA pathophysiology affect which cell>> ant. horn cell

23. pulmonary hypertention: pathophysiology:


pulmonary vascular resistance increase , delay in normal
circulatory changes potentiating right to left shunt across ductus
arteriosus and foramen ovale .

^25.allergic to penicillin + taking carbamezapine: ??! rifampicin, *ciprofloxacin, cefelexine


pastest - effectiveness of oral contraceptive is reduced with enzyme inducer like
( carbamezipine , phenytoin , phenobarb , tobiramite , rifampicin )

26. corrected QT= QT/ square root of RR

27. mode of action of monoclonal antibody


IL6= tocilizumab,
CD20 = rutixumab ,
^inositol monophosphate inhibitor == lithum

28. adrenaline==action cardiac - act on beta 1 receptor increase cardiac contractility and heart
rate,
- act on alpha 1 receptor and decrease mucous production,
- has long effect of beta 2 receptor and cause bronchodilator
Notes : a1( vessels ) = vasoconstriction a2=
sympatholytic
b1 ( heart ) = increase HR and contractility b2 ( lungs ) = bronchodilator , "
vasodilator " , increase insulin so affecting pottasium other similar q :
- drugs with following mecahnism :
1/alpha agonist cause inhibition of mucous sectestion = phenylphrine
2/ long acting b agonist = salmetrol
, 3/ short acting b agonist = salbutamol

29. pharma side effect


note:
metabolic acidosis + hyperkalemia + hyponatremia, *? spironolactone

All FOP Recall till June 2021 with note 273 of 378
respiratory center depression + hyperthermia, * ? opoid(cocaine -morphine)* ?
Notes : metabolic acidosis = carbonic anhydrase inhibitor (acitazolamide) , k+sparing (
spironolactone ) metabolic alkalosis = loop diuretics ( furesimide ) , thiazide hypercalciuria =
loop diuretics - hypocalcemia - nephrocalcinosis hypocalciuria = thiazide - hypercalcemia
weakest diuretics = carbonic anhydrase inhibitor = glucoma
all duretic cause alklosis except acetazolamide and k sparing cause acidosis
furosmide=hypocalcemia and hypercalceurea
thiazide=hypercalcemia TAS case 40

30. drug used in neonatal emergency:


1/ 34wk + low PLT with PDA indomethacin =*ibuprofin,
2/ severe aortic stenosis =
*PGE1 PGE2
3/ 35wks features of renal impairement (close PDA) ?? may be heart failure so
will use furesmide

31. acyclovir:
a. water soluble b. *excreated by kidney
c. metabolised and execreated by liver
d.( low or high ) bound to plasma protein Note
from 1st aid :
Mode of Action : Guanosine analogs
Side effect : Acute Renal failure if not adequatly hydrated
32. drug used for MRSA: vancomycin
33. Botox in botulism mechanism of action: destruction of Achestrase, *bind to
presenaptic receptor, bind to postsynaptic receptor
*NOTE:
* It also prevent the release of Ach at NMJ
34. diaphragmatic hernia occur
*8 - 12 wk, 12- 16 wks, 16- 24 wks, { 24 wks

35. double ureter insertion of the ureter on the bladder upper lobe ureter
inserted medial and below the lower lobe, *medial and above, lateral and
below, lateral and above.

36. optic nerve pathway bitemerol hemanopia, quadrantopia, homonomous


hemianopia
optic chiasma= bitemerol hemanopia
37. enzyme deficient in the urea cycle = ariginosuccinate
38. cardiac cycle: atrial depolarization = p wave ,, av node conduction =P-R
interval ? venticular depolarization QRS
ventricular repolarization = T wave

All FOP Recall till June 2021 with note 274 of 378
39. infection EMQ
*previously well child refuse feeding develop heart failure , = ?may be viral mycorditis (coxaki )
periventricular calcification = CMV,
registrar cant illist red reflex = rubella
40. Heb influenzea = gm -ve coccobacilli
listeria baby with green color muconium = gm+ve rods,
meningococcal = gm -ve diplococci

41. BOF polysaccharide vaccine used against post splenectomy:=


peumococci

42. vaccine table:I dont remmeber exactly, immunocompromized with his


brother 2month,
14 yrs, know the table by heart

43. BOf mother with hep B immunoglobulin to the baby:


HBs ag+ antiHbc ag, anti HBSag + Hbeag,
= Immunoglobulin + vaccine

44. vitamins EMQ Nutrient supplement for pt wit


perioral dermatits = zinc
vegitarian diet = ? b12
obese lazy diatery management not improved they give him drug
( i cant remmeber the name trade name ) = ? Note : drugs may
be used with obesity
orlistat - lipase inhibitor = all fat soluble vitamins ( ADEK )
Metformin - biguinade increase insulin sensitivity

45.vitamin absorbtion : small intestine, ( * iron , folate


= proximal small bowel , colon, ? *water
distal ilium = * vit b12 , bile salts
46. investigation
18month constipation failure to thrive, = ? cystic fibrosis
3month vomiting metabolic alkalosis, = ? Pyloric stenosis -- US

47. BOF direct effect parathyroid hormone increase ostoeclast activity,


* inhanced 1 alpha hydroxelase enzyme.

48. luekemia, dengue fever, malairia, typhoid


(fever+ bradycardia = typhoid )
(fever,from kenya, has fatique, malaise, vomiting, took antimalarial drug =
malaria )
( fever and bone pain = ?leukemia )

All FOP Recall till June 2021 with note 275 of 378
Notes : dengue fever : ( south east asia , caribbean , central and south
america ) headache , back pain , myalgia , arthralgia , nausea and
vomiting hemorrhagic fever ,,retroorbital pain,,bone breaking pain
49. vit D in the kidney: a. 1,25 dihydroxylase or 24 ,25 hydroxlase
50. osteopenia occur Chronic Kidney Disease : 1/ increase execretion of
phosphate, 2/*decrease excretion of phosphate , 3/decrease absorbtion in the
instine, decrease reabsorbtion of ca in the kidney
Note : - phosphate retention leads to hypocalcemia increased phosphate and
decrease calcium stimulate secondary hyperparathyroid :subperiosteal bone
resorption
- defecient renal 1 hydroxylase activity and defecient 1,25 (oh)2-d3 contribute
to hypocalcemia and rickets
51. cystic fibrosis pathophysiology of CFRT gene: C Note : fails to secrete chloride ions in
lung
fails to absorb chloride in sweat gland == so high sweat test ???
52. pulmonary function test
53. Di George, neonate delivered with cleft palate = 22q11
54. drug the block the Na channel = phenytoin
55. *von willibrand = PT normal APTT prolong boy with gum bleeding, mother the same
condition
56. epilepsy: day dreaming, benign vertigo ( pale, tearing blinking), complex partial ( aura)
57. .6 yrs old boy with headache and enuresis, NA 124, k 5.6, cortisol low, 1/*adrenal
insufficiency,2/ SIADH,3/ RTA
58. 10 yrs old boy with aneuresis camping =desmopression,
headache,osmolarity given
>> 1/cranial DI,
2/ increase dose of desmopresein
59. circle of Willis except = *ant cranial
60. down syndrome unbalance translocation ( young female )
61. anemia 6 month mother think he is pale, infrequent diarrhea, low MCV PLT 495 Retic low ,
formula milk started
= 1/ cow milk allergy, 2/*iron deficiency ,3/ folate B12, 4/physiological
62. neonate hypoglycemia replacement 8 mg per Kg not resolved: liver 2 cm
*hyperinsulinemia
, GSD
63. protusion of the eye and kidney problem = ?neuroblastoma
64. what is the indication of non accidental injury >> retinal hemorrhage
65. girl not eating, loss of wt. withdrawal from friends
*depression, bipolar
66. pitit mal eilepsy ECG = 3 spikes and waves
67. prophylasis of meningitis: *ciprofloxacin, ceftriaxone inj once
68. . Hb dissociation curve which is the characteristic of HBF to HBA that cause its high affinity
= HBF decrease binding to 2,3 DPG
69 CAH enzyme the commonest: = *21 hydroxylase

All FOP Recall till June 2021 with note 276 of 378
70.drug causes metabolic acidosis, nephrocalcinosis, hyponateremia : spironoloactone??
71. neonate become cyanosed on feeding and well on crying there is single 2nd heart sound>>
upper airway obstruction ( choanal atresia charge disease )
72. which nerve responsible for gastric emptying >> vagus nerve
73. the cause of total K depletion in DKA:
*hyperosmolR, POLYURea, acidosis, insulin, polydipsia

- senario about headache = tension


- management in kwash ( BOF )
- senario about Asperge==type of autism milder type

All FOP Recall till June 2021 with note 277 of 378
Feb 2016
1)neonate low birth weight his mother take pethidine one hour before labour
take oxygen but no chest movement what do next intubation or antidote
2)boy ride bicycle with car accident not lost consciousness most common
complications can happen
Posttraumatic stess
syndrome or extra dual hematoma
3)same as brevious question but ask about after 6 hours he becomes ill
what happens
Hip fracture or hematoma or haem thorax
3
Cases of constipation

Case don't eat not go with friends


Depression

Most common complications of boy has ADHD


Learning proplem or
SE of drugs

Ttt of allergy

Case of prolonged jaundice in 6 week boy mother HBS ag +ve


‫واضيع تكوني عارفه ايه‬0‫ين كل ا‬4‫متحان عموما عايز واحد مزاكر كويس عارف جايد‬4‫ا‬
Most common
What do next
Investigation

B‫ بياخد تقريبا ساعت‬bof‫تبداي ب‬


‫ ساعه‬extended ‫وبعدين‬
tumor you irradiate blood products before giving
it(hodgkins.wilms.neuroblastoma .medulloblastoma )

constipation‫ عن‬extended ‫سؤال‬


cystic fibrosis ‫ وحالة‬lhamophili ‫وحالة‬sickle cell ‫وواحد تحسب الريسك في حاله‬

pain managment ‫ عن‬extended ‫وسؤال برضه‬


‫ واخد‬chronic regional pain ‫ولد عمل الزايدة وعنده الم شديد تدي ايه وولد عند كسر وواحد عنده‬
‫لم موجود‬4‫ وليه ا‬B‫باراسيتامول وبروف‬

interosseous ‫دقيقه اخد ريكتال دايزيبام ولسه بيتشنج ومفيش وريد تدي ايه‬15 ‫ولد بيتشنح عدي‬
.… lorazepam 4‫ و‬phyntoin

All FOP Recall till June 2021 with note 278 of 378

‫الورقه التانيه‬
.mode of action of dompridone .ondansteron
c.s.f circulation
‫ختيارات وسؤال عن ايه التيست الي تستخدمه في كام‬4‫ وتستنتج ايه منها في ا‬forest spot ‫حصاء رسمه‬4‫ا‬
‫اختيار مديهولك‬
nnt ‫وسؤال عن‬
normal number to treat
‫شخاص ومحدش يعرف‬4‫ستخدمه في مثال جايبه هتجرب دواء في نفس ا‬0‫وسؤال عن ايه الدراسه ا‬

acid base balance ‫مساله عن‬

‫ختيارات‬4‫وا‬tissue‫ لل‬B‫كسج‬4‫ايه الي يزود ا‬


…… fever .alkalosis

‫فروض يعمل ايه الطبيب العام في الفحص‬0‫كان فيه كمان سؤال بيبي اليوم العاشر ا‬

def.of total lung capacity


Side effects of antiemetics


total anomails pul v ‫سؤال عن‬


Site of AV node

Mechanism of action of adenosine


adenosine ‫ ازاي بيشتغل في‬svt


Non REM sleep

All FOP Recall till June 2021 with note 279 of 378
Monoclonal antibodies

Psychogenic polydepsia

‫ كان فيه سؤال ازاي بتتكون‬diaphragmatic hernia morgagini type

All FOP Recall till June 2021 with note 280 of 378
FOP OCTOBER 2016

1/Child present with painful left knee, paracetamol doesn’t help but ibuprofen help. H/O
travelling to France 5 weeks ago. O/E he is afebrile. Diagnosis?
a. Rheumatoid arthritis.
b. Juvenile Idiopathic Arthritis.
c. Lyme disease.//??
d. Leukemia.
e. Septic arthritis.

1) 2 years old, previously healthy apart from febrile convulsion at age of 9 month, present for
hernia repair and has convulsion before the repair. Diagnosis?
a. Idiopathic epilepsy.
b. Hyperinsulinism.
c. Hypoparathyroidism.
d. Medium-Chain acyl-CoA dehydrogenase deficiency.
e. QT syndrome. Hypoglycemia

3) Child known to have frequent viral infection associated with cough and wheeze. He is playing with
pets at home. He is on PRN Ventolin, present with S.O.B, cough and fever. Your advice:

a. Inhaled corticosteroids.
b. Remove pets from house.
c. Skin allergy test.
d. Antibiotics for every viral infection.
e. ?

4) 10 years old girl brought by her mother, which is a sex worker, with green discharge appeared on her
pant. She refused examination and just ask for a medication, you suspect:

a. Inconsequential sex contact.


b. Consequential sex contact. Consensual
c. Sex contact with a girl. (The same ?)
d. ?

5) Preterm, 32 weeks, baby with RDS, post M.V. CSF: E.coli meningitis. Long-term complication:

a. ROP.
b. Chronic lung disease.
c. Hydrocephalus.
d. Deafness. ?
e. ?

All FOP Recall till June 2021 with note 281 of 378
6) 11 years old Down syndrome patient, asymptomatic, present to your clinic for regular check-up. O/E
looks well, weight 0.4 centile. As a screening, you should order:

a. Thyroid function test.


b. Coeliac screening.//????
c. Blood sugar.
d. X-ray spine
e. Blood film.

7) A known case of ALL on maintenance chemotherapy, following with you and receiving the
chemotherapy in a tertiary center. The specialist nurse call you from the tertiary center telling the
neutrophils count is abnormal and I think adjustment of the dose is required. Your action:

a. Call your consultant.


b. Recalculate the dose and make sure of it and agree with her.
c. Ask her to send a fax about her concern.
d. ?
e.

8) 3 years old child not sleeping well at night because of umbilical hernia pain. Your action:

a. Reduce and fix it.


b. Get surgical opinion.
c. Reassurance.

9) Your consultant prescribed methotrexate as a treatment for RA patient. The parents read on net that
this drug should prescribed only for malignant conditions. Your action:

a. Tell them as it prescribed by a consultant you should take it even if it is not licensed.
b. Refer them to rheumatologist. ?????
c. Convince them to take it.
d. Ask them NOT to take it.
e. Refer them to social worker for education.

10) You should refer the child for further developmental assessment if:

a. Not sit unsupported at 8 month.


b. Not babbling at 6 month.
c. Walk at toe at 18 month.
d. Convergent squint at 6 month. ???

11) Best drug if given intrapartum reduce the risk of HIV transmission:

a. Zidovudine. ??
b. Nevirapine.

All FOP Recall till June 2021 with note 282 of 378
c. Lamivudine.
d. ?

12) Neonate with TSH 180 and no uptake in thyroid region:

a. Thyroid aplasia.
b. Dyshormonesis. Goiter + hypothyroidism
c. ?
d. ?

13) 15 Years old present after 20 paracetamol tablet intoxication, you told her to wait for 4 hours to do
Paracetmol level but she escape. You called her home and parents state they don’t know where is she.
Your action:

a. Ask hospital security to find her.


b. Call child protection registrar.
c. Call police to look for her.
d. Tell parents to ask her to come to hospital when she returned home.
e. ???????????

14) A suitable way to confirm NGT position:

a. Inflation and auscultation.


b. Gastric aspiration.(guideline neonatology)
c. CXR.
d. ?
e. ?

15) Neonate found to have bilateral conjunctivitis and hepatosplenomegaly:

a. HSV (for conjunctivitis - but not common to have HSM :/ )


b. CMV
c. Rubella
d. ?
e. ?

EMQ1: Nutritional deficiency

a. Vitamin B6
b. Vitamin B12
c. Vitamin K
d. Vitamin D

All FOP Recall till June 2021 with note 283 of 378
e. Vitamin C
f. Zink
G. ?
H. ?
I. ?

1. Neonate with seizure. B6


2. Neonate with gum bleeding. Vit K
3. Neonate with broadening of ulna. Vit D

EMQ 2: End stage cancer on methotrexate???????????

a. Decrease the dose of morphine.


b. Decrease the dose of methotrexate.
c. Increase the frequency of morphine
d. Increase the dose of methotrexate.
e. ?
Chronic pain
f. ? * Slow release morphine bid + breakthrough
g. ? * if breakthrough reach maximum 8 —> SC pump
1. Need morphine more frequently than his plan.
2. Difficulty of swallowing== Decrease the dose of methotrexate
3. ? * Pain control but with there is side effects —> hyoscine patch
* uncontrolled pain with different swallowing —> fentanyl

EMQ3: DM management

1- 14 Y/O female diagnosed 6 months back with DM and she is on long acting insulin at night and short
acting insulin before meal. RBS at 2 am= 2.9 mmol/l and at morning before breakfast 10 mmol/l.

(Decrease LA insulin or decrease tea-time short acting insulin --- or even later time dose of the short
acting one).decrease night short acting

down ==hyperglycemia in mornining =increase lantua (long acting)

somoggy==hypoglycemia midline and hyperglycemia in morning decrease short acting(midnight)

2- Teenager well controlled diabetic has infection now RBS=21, urine ++ ketone

(? Admit for IV fluids and insulin infusion or give extra dose of insulin ... as long as she is not
vomiting/abdominal pain and no ABG available).

3- 4 years-old child with fever 39 and vomiting after supper as well as abdominal pain and nausea, RBS=
6, urine no ketones → increase fluid intake or decrease insulin dose.

Sugary fluid

All FOP Recall till June 2021 with note 284 of 378
June 2016 exam
mixed fop+tas

1. neuromuscular junction receptor >> nicotinic acetylcholine

2. hip dislocation:
1.internal reduction 2. external reduction 3.double pampers*

3. changes in fetal circulation


4. Hb f best of 5 =
work on 2,3DPG and decrease it so shifting the curve to Left and increasing the affinity to
oxygen

5. Hbc,, Hbh ,,spherocytosis ,,,deletion


note
2 or 1 deletion of alpha chain, = alpha thalessemia trait
3 deletion of alpha chain, = HbH disease
4 deletion ==Hb part (alpha thalathemia major)
2 deletion of beta chain, =
B. major
1 deletion of beta chain=
B .minor
electrophoresis, = HbC , HbS
fragility test = spherocytosis EMA TEST(now EMA test is diagnostic)

, abnormal Na channel =???


deravate disease has abnormal NA channel(Mutation)

6/ statistic: bias information bias,


research from the clinic, =sampling bias
research without publication, = reporting bias
research the forms where not completed =artesian bias -late
look

7. bof diarrhea polyhydramnious >> bicarbonate 28, ph 7.5, cl 89 low Na low,


*cl loosing enteropathy (if there is polyhydraminous with hypo chloremic hypokalemic metabolic
alkalosis this fit with barter syndrome)

Metabolic alkalosis + polyuria


* no diarrhea —> barter
* with diarrhea —> protein loosen

All FOP Recall till June 2021 with note 285 of 378
8. child has be well until early no fever bruising around the mouth / bleeding from
mouth , low GCS , bruises in abdomen
?? NAI
9. abnormal eating habit ( anorexia - bulimia ,, etc )

10. what is the way that children metabolise paracetamol - *sulfation(in neonate)
gluconridation in children

11. . which layer in epidermolysis bullosa symplex >> 1/ junctiona layer with scar,
2/junctional layer without scar,
3/*intraepidermal without scar,4/ intraepidermal with scar

12. pathophysiology of VSD: *1/hyperemic lung lt ventricular over load ,


2/oligeromic lung with rt ventricular overload,,,3/ increase pulmonary blood pressure,

13. *experiment to know the cause of increase MRSA: case control, cohrt,
*randamized control trial
Notes : Observational - prevelance and diagnosis = cross sectional
- common disease , causes , risks , prognosis = cohort
- rare diseases , causes , risks = case control
- association in population = Ecology
14. statistic: experemint 4 CI most statistical seqnificant
15. defenitions :
16. which of the following should measured in anaphylasis ( insect bite):
1/histamine, *2/mast cell tryptase,3/ IgE,

inculation and contarst and unknown ellergene= tryptase is specific


17. antibodies : - responsible of villous atrophy in celiac disease, = anti-tissue transglutaminas
IgA

18. - neonate delivered by CS due to bradycardia which persist after delivery, = anti Ro
Rash —> antira
- teenager malar rash arthritis red urine = anti douple stranded DNA

18. evoked auto acoustic emission response: 1/*choclear, 2/echo,


3/high amplitude voice ^

19. the test for obstructive sleep apnea: 1/paradoxical movement of


the thoracoabdominal,
*2/desaturation,
3/wake up more than 10
times
Hearing screen
* at birth , otoscustic , check cholchlea
middle ear
* if not pass or 6 month ABR check
audiotary upper pathway

All FOP Recall till June 2021 with note 286 of 378
20. aortic arches Note :-:
6 branchials , 1 , 2 , 5 dissappear
3 left and right carotid arteries
4 letf - aortic arch , right -,,,,, rt brachiocephalic and rt subclavian
6 left and right pulmonary a. and ductus artrusus,

21. bilirubin characteristic of bilirubin.. phototherapy mechanism


phototherpy will convert the unconjugated to water soluble ^

22. SMA pathophysiology affect which cell>> ant. horn cell

23. pulmonary hypertention: pathophysiology:


pulmonary vascular resistance increase , delay in normal
circulatory changes potentiating right to left shunt across ductus
arteriosus and foramen ovale .

^25.allergic to penicillin + taking carbamezapine: ??! rifampicin, *ciprofloxacin, cefelexine


pastest - effectiveness of oral contraceptive is reduced with enzyme inducer like
( carbamezipine , phenytoin , phenobarb , tobiramite , rifampicin )

26. corrected QT= QT/ square root of RR

27. mode of action of monoclonal antibody


IL6= tocilizumab,
CD20 = rutixumab ,
^inositol monophosphate inhibitor == lithum

28. adrenaline==action cardiac - act on beta 1 receptor increase cardiac contractility and heart
rate,
- act on alpha 1 receptor and decrease mucous production,
- has long effect of beta 2 receptor and cause bronchodilator
Notes : a1( vessels ) = vasoconstriction a2=
sympatholytic
b1 ( heart ) = increase HR and contractility b2 ( lungs ) = bronchodilator , "
vasodilator " , increase insulin so affecting pottasium other similar q :
- drugs with following mecahnism :
1/alpha agonist cause inhibition of mucous sectestion = phenylphrine
2/ long acting b agonist = salmetrol
, 3/ short acting b agonist = salbutamol

29. pharma side effect


note:
metabolic acidosis + hyperkalemia + hyponatremia, *? spironolactone

All FOP Recall till June 2021 with note 287 of 378
respiratory center depression + hyperthermia, * ? opoid(cocaine -morphine)* ?
Notes : metabolic acidosis = carbonic anhydrase inhibitor (acitazolamide) , k+sparing (
spironolactone ) metabolic alkalosis = loop diuretics ( furesimide ) , thiazide hypercalciuria =
loop diuretics - hypocalcemia - nephrocalcinosis hypocalciuria = thiazide - hypercalcemia
weakest diuretics = carbonic anhydrase inhibitor = glucoma
all duretic cause alklosis except acetazolamide and k sparing cause acidosis
furosmide=hypocalcemia and hypercalceurea
thiazide=hypercalcemia TAS case 40

30. drug used in neonatal emergency:


1/ 34wk + low PLT with PDA indomethacin =*ibuprofin,
2/ severe aortic stenosis =
*PGE1 PGE2
3/ 35wks features of renal impairement (close PDA) ?? may be heart failure so
will use furesmide

31. acyclovir:
a. water soluble b. *excreated by kidney
c. metabolised and execreated by liver
d.( low or high ) bound to plasma protein Note
from 1st aid :
Mode of Action : Guanosine analogs
Side effect : Acute Renal failure if not adequatly hydrated
32. drug used for MRSA: vancomycin
33. Botox in botulism mechanism of action: destruction of Achestrase, *bind to
presenaptic receptor, bind to postsynaptic receptor
*NOTE:
* It also prevent the release of Ach at NMJ
34. diaphragmatic hernia occur
*8 - 12 wk, 12- 16 wks, 16- 24 wks, { 24 wks

35. double ureter insertion of the ureter on the bladder upper lobe ureter
inserted medial and below the lower lobe, *medial and above, lateral and
below, lateral and above.

36. optic nerve pathway bitemerol hemanopia, quadrantopia, homonomous


hemianopia
optic chiasma= bitemerol hemanopia
37. enzyme deficient in the urea cycle = ariginosuccinate
38. cardiac cycle: atrial depolarization = p wave ,, av node conduction =P-R
interval ? venticular depolarization QRS
ventricular repolarization = T wave

All FOP Recall till June 2021 with note 288 of 378
39. infection EMQ
*previously well child refuse feeding develop heart failure , = ?may be viral mycorditis (coxaki )
periventricular calcification = CMV,
registrar cant illist red reflex = rubella
40. Heb influenzea = gm -ve coccobacilli
listeria baby with green color muconium = gm+ve rods,
meningococcal = gm -ve diplococci

41. BOF polysaccharide vaccine used against post splenectomy:=


peumococci

42. vaccine table:I dont remmeber exactly, immunocompromized with his


brother 2month,
14 yrs, know the table by heart Feco- oral
Rota vaccine
43. BOf mother with hep B immunoglobulin to the baby:
HBs ag+ antiHbc ag, anti HBSag + Hbeag, All we will give IV Ig + vaccinations expect
= Immunoglobulin + vaccine Hbe Ab
6 doses ( routine + extra at 0,1,12 )
44. vitamins EMQ Nutrient supplement for pt wit
perioral dermatits = zinc
vegitarian diet = ? b12
obese lazy diatery management not improved they give him drug
( i cant remmeber the name trade name ) = ? Note : drugs may
be used with obesity
orlistat - lipase inhibitor = all fat soluble vitamins ( ADEK )
Metformin - biguinade increase insulin sensitivity

45.vitamin absorbtion : small intestine, ( * iron , folate


= proximal small bowel , colon, ? *water
distal ilium = * vit b12 , bile salts
46. investigation
18month constipation failure to thrive, = ? cystic fibrosis
3month vomiting metabolic alkalosis, = ? Pyloric stenosis -- US

47. BOF direct effect parathyroid hormone increase ostoeclast activity,


* inhanced 1 alpha hydroxelase enzyme.

48. luekemia, dengue fever, malairia, typhoid


(fever+ bradycardia = typhoid )
(fever,from kenya, has fatique, malaise, vomiting, took antimalarial drug =
malaria )
( fever and bone pain = ?leukemia )

All FOP Recall till June 2021 with note 289 of 378
Notes : dengue fever : ( south east asia , caribbean , central and south
america ) headache , back pain , myalgia , arthralgia , nausea and
vomiting hemorrhagic fever ,,retroorbital pain,,bone breaking pain
49. vit D in the kidney: a. 1,25 dihydroxylase or 24 ,25 hydroxlase
50. osteopenia occur Chronic Kidney Disease : 1/ increase execretion of
phosphate, 2/*decrease excretion of phosphate , 3/decrease absorbtion in the
instine, decrease reabsorbtion of ca in the kidney
Note : - phosphate retention leads to hypocalcemia increased phosphate and
decrease calcium stimulate secondary hyperparathyroid :subperiosteal bone
resorption
- defecient renal 1 hydroxylase activity and defecient 1,25 (oh)2-d3 contribute
to hypocalcemia and rickets
51. cystic fibrosis pathophysiology of CFRT gene: C Note : fails to secrete chloride ions in
lung
fails to absorb chloride in sweat gland == so high sweat test ???
52. pulmonary function test
53. Di George, neonate delivered with cleft palate = 22q11
54. drug the block the Na channel = phenytoin
55. *von willibrand = PT normal APTT prolong boy with gum bleeding, mother the same
condition
56. epilepsy: day dreaming, benign vertigo ( pale, tearing blinking), complex partial ( aura)
57. .6 yrs old boy with headache and enuresis, NA 124, k 5.6, cortisol low, 1/*adrenal
insufficiency,2/ SIADH,3/ RTA
58. 10 yrs old boy with aneuresis camping =desmopression,
headache,osmolarity given
>> 1/cranial DI,
2/ increase dose of desmopresein
59. circle of Willis except = *ant cranial
60. down syndrome unbalance translocation ( young female )
61. anemia 6 month mother think he is pale, infrequent diarrhea, low MCV PLT 495 Retic low ,
formula milk started
= 1/ cow milk allergy, 2/*iron deficiency ,3/ folate B12, 4/physiological
62. neonate hypoglycemia replacement 8 mg per Kg not resolved: liver 2 cm
*hyperinsulinemia
, GSD
63. protusion of the eye and kidney problem = ?neuroblastoma
64. what is the indication of non accidental injury >> retinal hemorrhage
65. girl not eating, loss of wt. withdrawal from friends Can be up to 52 days will cause by
difficulty birth
*depression, bipolar
66. pitit mal eilepsy ECG = 3 spikes and waves
67. prophylasis of meningitis: *ciprofloxacin, ceftriaxone inj once
68. . Hb dissociation curve which is the characteristic of HBF to HBA that cause its high affinity
= HBF decrease binding to 2,3 DPG
69 CAH enzyme the commonest: = *21 hydroxylase

All FOP Recall till June 2021 with note 290 of 378
70.drug causes metabolic acidosis, nephrocalcinosis, hyponateremia : spironoloactone??
71. neonate become cyanosed on feeding and well on crying there is single 2nd heart sound>>
upper airway obstruction ( choanal atresia charge disease )
72. which nerve responsible for gastric emptying >> vagus nerve
73. the cause of total K depletion in DKA:
*hyperosmolR, POLYURea, acidosis, insulin, polydipsia

- senario about headache = tension


- management in kwash ( BOF )
- senario about Asperge==type of autism milder type

All FOP Recall till June 2021 with note 291 of 378
Fop june 2015
BOF:
1- RH. fever recently diagnosis : investigation ??
a. ASOT ✓
b. ESR
c. ECHO
2- sever sudden abdominal pain with vomiting, ass with pain left testicle diagnosis?
a. LF tortion of tests, ✓
b. tortion of morgagni
3- nocturnal enuresis, had abnormality in renal scan : what he had :
1. duplex kidney , ✓
2. cyst kidney,
3. horseshoe kidney child
4-had chest infection: on exam : stony dull chest :what investigation:
1. CT Scan,
2. U/S chest, ✓
3. and other option
5- 12 yrs old took 6 tabs of paracetamol, bulling in school, level under treatment
,plan to discharge , how you reassure the parent he is ready to go home ??
1. child not on child protection list
2. he didn't try to suicidal before.
3. other option *refer to psychiatry before discharge ✓

All FOP Recall till June 2021 with note 292 of 378
6- grandmother has given antipyretic child developed bleeding ,increase APTT
(53 sec ),fibrin 0.5 , what the diagnosis :
a. DIC, ✓
b. aspirin toxicity
7- baby has GE developed heart failure organism :
1. strept ,
2. coxsacki ✓
*coxsacki A hand foot and mouth disease
*coxsacki B viral myocarditis
8-baby born after 6 days lethargic ,after 9 days developed vesicles :
1. HSV ✓
2. varicella,
3. staph,
4. listeria
9- child had obesity ,dyslipidemia ,hyperglycemia ,and .................,what the
risk factor for her when she was born :

a. preterm 2.5 kg @ 1 yr old 7.5 kg


b. preterm 2.5 kg @ 1 yr old 10 kg
c. term 2.5 kg @ 1yr old 10 kg ✓
d. term 2.5 kg @ 1 yr old 7.5 kg
e. term 4.5 kg @ 1 yr old 10 kg
10- boy playing football, sudden syncope, become well and neurologically
normal, what is the best investigation for diagnosis?
1. EEG,
2. MRI,
3. ECG, ✓ prolonged qt 12 lead ECG
4. Blood sugar.

All FOP Recall till June 2021 with note 293 of 378
11-Build 3cubes, palmar grasp ,know his name ,What is developmental age
for the child?
18 mo to 2 yrs

12- Stammering child the cause may be


1. Autisic spectrum disorders
2. Bilingual parents ✓
3. Tie tongue

13- Sudden infant death syndrome?


1. Risk decrease if mother smoke outdoor
2. You need metabolic screening for the next baby ✓ Undiagnosed MCAD
3. It decrease die to nursing in prone position to avoid respiratory
obstruction
4. Use of apnea alarm decrease its incidence
5. Mother need to train about cpr
14- Child came with spiral fracture after fall from the bed Action
1. Ask senior ✓
2. Tell parents possibility of NAI
3. Make skeletal survey Senior then investigation
4. child physical abuse

15- girl came with vulva bleeding after falling from bicycle Whats ur action
1. Suture
2. Send to obs and gynea
3. Call child protection
4. Call ur senior pediatrician ✓

16- 8yrs came with ataxia and convulsion Best inv is


1. CT
2. Mri ✓
3. EEG

All FOP Recall till June 2021 with note 294 of 378
17- Girl her father abused her sexually came with (funny spills) Whts best
investigation
1. Mri
2. Ct
3. Eeg
4. Refer to physiatrist ✓ PTSD Post traumatic stress disorder

18 - Child start walking at 15months then develop motor regression


proceeded by upper respiratory infection
1. Duchene muscular dystrophy ✓
2. Becker muscular dystrophy
3. Chronic fatigue syndrome
Consider Guillain Barre syndrome in child with recent URTI or
campylobacter GE and developed ascending motor weakness ass with
autonomic and sensory manifestations
19- Female with short stature examination is normal Mid parental height at
25% Her ht 0.4 centile
1. Xo ✓
2. X linked recessive
3. X linked dominant
4. Autosomal recessive
5. Multifactorial

20- Mother has cataract her uncle has problem Her baby boy has myopathy
What is inheritance
1. Mitochondrial
2. X-linked recessive
3. X-linked dominant
4. Autosomal recessive
5. Autosomal dominant ✓ myotonic dystrophy

All FOP Recall till June 2021 with note 295 of 378
21- A newborn on examination found to have ambiguous genitalia with
hypospadias. Parents want to name their baby what is your action plan:
1. Tell them the sex of baby is uncertain and advise them to wait ✓
2. Tell them to choose a name match for both girl and boy
3. Tell them to ascertain sex after karyotyping
EMQ:
1 /neonate had vomiting ( 3 scenario ) :
1, day 1 vomiting bilious, Duodenal Atresia, duodenojejunal Atresia
2. day 6 bilious vomiting, Malrotation, Hirschsprung
3. maybe day 2 non-bilious vomiting overfeeding or reflux

2/ options : ceftriaxone, cefotaxime, amoxicillin, penicillin, vancomycin


/gentamycin
1-treatment of salmonella , ceftriaxone
2-Rx of listeria, amoxicillin plus genta in n guigelines
3-Rx of necrotizing fasciitis swab growth of MRSA vancomycin

All FOP Recall till June 2021 with note 296 of 378
All FOP Recall till June 2021 with note 297 of 378
1. Best antibiotic for:
• Newborn with listeria monocytogenes →
ampicillin
• Salmonella infection cephalosporin
• A cystic fibrosis patient with acute
pseudomaonas infection ceftazidine and
tobramycine

All FOP Recall till June 2021 with note 298 of 378
⚫ 2. An asthmatic patient received
salbutamol and Ipratropium bromide
nebulizer. The patient improved but
developed nausea and headache with
unequal pupils. The left one is 5 mm
and the right one is 3 mm and
unresponsive to light but there is
spontaneous eye movements. What is
the best test to diagnose the
underlying cause:
⚫ ▪Application of pilocarpine eye
drops Ipratropium side effect mydrosis
Application of pilocarpine —> if myosis—> then this side effect of drug

⚫ ▪Application of fluorescence eye


drops
All FOP Recall till June 2021 with note 299 of 378
 
⚫ 3. An infant with congenital
neutropenia with his family in a
camp, the infant developed fever
38.5 and the mother called you.
What is the best advice to the
mother:
⚫ ▪Go to the hospital as soon as
possible
⚫ ▪Give paracetamol and observe the
All FOP Recall till June 2021 with note 300 of 378
⚫ 4. What about that boy 3 years old
presented to ER with burn 4 cm on his chest
his mother said that he was running to his
grandmother and the tea is thrown up on
his chest now he received opiate and is good
the boil now is 4 cm what to do: ▪put ice
⚫ ▪give antibiotic
⚫ ▪IV fluids
⚫ ▪refer to dermatologist
⚫ ▪refer to social services
NAI

All FOP Recall till June 2021 with note 301 of 378
⚫ 5. Child presented with hemorrhagic
manifestations. There is family history of
2 relatives died from intracranial
hemorrhage. PT and APTT are normal.
What is the most likely diagnosis
⚫ ▪Check factor XIII
⚫ ▪Hemophilia Prolong only ptt
⚫  vw

All FOP Recall till June 2021 with note 302 of 378
6. A baby boy is born to mother
blue, heart rate 40/min, irregular
breathing, mother is given
morphine 1 hr before birth, what is
the most immediate step?
▪Adrenaline Initial steps
Inflation 5 breath
▪Intubation IfIntubation
chest rise —> inflation for 30 second
+ compression
▪Chest compressions
▪IM naloxone

All FOP Recall till June 2021 with note 303 of 378
1st day; karyotype + US
2nd day; Electrolytes
3rd day; 17 hydroxyprogestrone
7. Baby with ambiguous genitalia, you told parents
that you have to consult ... what is the next step:
▪Karyotyping
▪Send blood for U &Es
▪ What to lead to diagnosis ==> 17 hydroxyprogestrone
 
8. Newborn baby blood glucose was low (1.9
capillary and 1.6 venous), next step is:
 
▪Give bolus (dextrose 10%)
▪Give oral feeds 1- 1.9 —> symptoms ; give IV Dextrose 10 %
—> no symptoms ; give buccal glucose gel
▪Do a septic screen

All FOP Recall till June 2021 with note 304 of 378
9/A 5 month old African boy brought in by
mother that he is not moving his left leg, he
has viral Flu like illness last week, 2 days back
he also fell from sofa, she suspected her elder
son a 3yr old pulled him On exam he has
runny nose, and not moving his left leg and
afebrile, what's the cause? Osteomyelitis
Non Accidental injury
Transient synovitis
Sickle cell disease
All FOP Recall till June 2021 with note 305 of 378
9. 3 week baby with poor wt gain (his wt was
3.6kg). He is formula fed and his grandmother is
giving him 100 ml every 4 hours strictly. He takes
his feed well but he is miserable. What to do?
Give 140 ml every 4 hours
  
10-/T/F regarding MMR vaccination:
Grandmother can consent
Dad who is separated but divorced
A stranger......!
A sister (can't remember the age)
 

All FOP Recall till June 2021 with note 306 of 378
11-A mother of a 5 yr old who has
eczema noticed a swelling beneath
her ear while combing his hair?
Refer to dermatologist Reactive
Reassure n send home reassure and review
lymphadenopathy —>

after 2 weeks
Refer to otolaryngologist
Review after 6 wks

All FOP Recall till June 2021 with note 307 of 378
12-A 5 yr old girl is brought by mother, she has
lost 5 kg weight for past few months, agitated and
can't sleep in night? Prescribe some sleep remedy
Malignancy No sign of malignant
Hyperthyroid
Abuse
13-A 3 year old girl, whose elder sis is subjected
to sexual abuse, complains of perineal soreness
which sign on exam confirm sexual abusVaginal
discharge
Torn hymen
Perianal warts

All FOP Recall till June 2021 with note 308 of 378
14-Patient with atopic eczema and itchy skin lesion on top
of eczema then manifestation of cerebellar ataxia started
to appear:
1 post infectious encephalomyelitis
2 chicken box
3 herpes Super infect on eczema —> eczema
herpeticum —> ADEM
 
15-Which of the following is most likely to confirm
diagnosis of Pyloric stenosis :
1 Hypochloremia
2. Hypokalemia
3. Non bilious vomiting
4. Metabolic acidosis
5. Increased urea
All FOP Recall till June 2021 with note 309 of 378
16- A child with
constipation and impacted
feces up to the umbilicus.
What you will do?
a) Start laxative and
review
b) Enema to relieve
impaction
c) Admit to the ward for
disimpaction
All FOP Recall till June 2021 with note 310 of 378
17- A 15 days old baby with an
umbilical stump granuloma, 2
mm in size, and oozing a
serous fluid. What is your
intervention? If small size review later

a) Reassurance and review


b) Refer to surgery
c) Silver nitrate cauterization
d) Antibiotic powder
 
All FOP Recall till June 2021 with note 311 of 378
18- A child with swelling of both knee joints and
one elbow joint. To which doctor you should refer
the patient? a) Orthopedic
b) ENT
c) Physiotherapy
d) Ophthalmologist (i.e. a case of oligoarticular
arthritis, for fear of anterior uveitis)
e) Surgery
19-. A 3 years old child with a small membranous
tongue tie. He has a difficulty in speaking the
letters B, R, T. his grandfather refused to do
surgery. To which doctor you should refer the
patient? a)
Occupational therapist
b) Speech and language therapist
c) Surgery
All FOP Recall till June 2021 with note 312 of 378
.20-A 10 years old child
with oral painful mouth
ulcers and anal fissures.
What is your diagnosis?
a) Crohn’sdisease
b) Ulcerative colitis
c) Celiac disease

All FOP Recall till June 2021 with note 313 of 378
21- A 2 years old girl with an injury to the vulvar
area and mild bleeding, while cycling in the park
with her brother. What is your next step?
a) Call your senior pediatrician
b) Refer to a gynecologist Scenario not much —>! Tricycle
c) Suturing the vulvar wound ride at 3 year —> NAI
 

22-A 10 years old tall boy with gynecomastia and


small testes. What is your possible diagnosis?
a) 47 XXY
b) 46 XO
c) Marfan’s syndrome
 

All FOP Recall till June 2021 with note 314 of 378
23- A 6 months old infant with facial eczema,
found to have a palpable LN over the neck, which
is about 3 cm in size, smooth, and non-tender.
What you will do?
a) Refer to a surgeon
b) Reassurance Reactive adenitis dt eczema even up to 5 cm
c) Start antibiotic therapy No scenario of ENT infection

24- A full-term neonate, delivered by LSCS, and


developed respiratory distress 2 hours after birth.
What is the possible cause of respiratory distress?
a) Transient tachypnea of newborn
b) Respiratory distress syndrome
c) Meconium aspiration syndrome
 

All FOP Recall till June 2021 with note 315 of 378
Not affect Gtt
25. What is the most important investigation
before starting treatment with sodium
valproate?
a) Liver function tests Acute hepatic encephalopathy
b) Serum ammonia level
 
 
26- A small child with painless, soft, cystic
scrotal swelling. Both testes are palpable. What
is your possible diagnosis?
a) Hydrocele
b) Torsion testes
c) Incarcerated inguinal hernia

All FOP Recall till June 2021 with note 316 of 378
27. A small child with left
testicular mass, prominent
pallor and bruises. What is your
possible diagnosis? a) ALL
b) Testicular tumor
 

All FOP Recall till June 2021 with note 317 of 378
28- A 2 years old child with croup. Oxygen and
budesonide were given but no improvement.
What is your next step?
a) Nebulized adrenaline, 400 mcg/kg, of 1:1000
solution
b)
 
29. A 3 years old child with fever, vomiting, and
dysuria. Abdominal US revealed renal stones.
Urine routine showed leukocytes 2+, protein +,
and trace blood. What is your possible diagnosis?
a) Nephrocalcinosis
b) Renal tubular acidosis
c) Proteus UTI UTI + stones

All FOP Recall till June 2021 with note 318 of 378
30- A 2 years old boy with severe pain during night
and was crying excessively. The parents can calm
the baby with milk. He was pale with bruises over
the chin, but an active child. What is your possible
diagnosis?
a) Trauma
b) ALL
c) Chronic hemolytic anemia
d) Iron deficiency anemia
 
 
31- A 3 years old child with a small VSD. He has to
do adenoidectomy. What you will do for
prophylaxis against infective endocarditis?
a) Give amoxicillin before the procedure
b) No prophylaxis is indicated
All FOP Recall till June 2021 with note 319 of 378
32- A child with ptosis, meiosis, and
anhydrosis. What is your possible
diagnosis?
a) Horner’s syndrome
b) ………….
 
 
33- A 6 weeks old infant with jaundice and
pale stool. ALT, AST, and conjugated
bilirubin levels are elevated. What is your
immediate action?
a) Intramuscular vitamin K
b) Intravenous vitamin K
c) Oral vitamin K If high INR —> give IV vitamin K

d) Vitamin A
e) Surgery
All FOP Recall till June 2021 with note
If bleeding —> ffp 320 of 378
34- A 5 years old child suddenly collapsed
after entering the swimming pool, but he
regains conscious few minutes after
transfer to the hospital. His uncle had a
similar attack 10 years ago and had a long
term follow up with a cardiologist. What is
your possible diagnosis?
a) Prolonged QT syndrome
b) Hypertrophic obstructive
cardiomyopathy
c)
 

All FOP Recall till June 2021 with note 321 of 378
35- A 5 mo old infant presented with
developmental delay. He was born vaginally,
with a birthweight of 4.5 kg and APGAR score of
one at 1 minute and 8 at 5 minutes. What is the
possible cause of developmental delay? a)
Inflicted brain injury ???
b) Congenital brain malformation
c) Hypoglycemia
d) Hypoxic ischemic encephalopathy
 
 
36-A child with bloody diarrhea, anemia, and
poor urine output. What is the next investigation
to do? a) Serum complement levels
b) Serum creatinine level
c) Serum urea level
May s creatinfor triad of HUS ???
All FOP Recall till June 2021 with note 322 of 378
37- An IUGR baby with periventricular
calcifications on the CT scan. What is the
immediate complication? Immediately
a) Hearing loss seizu, rash and
b) Visual impairment pneumonia
c) Neurodevelopmental sequelae
??????????CMV as hearing loss may bedelayed
after infancy!!!!!!!!!!!!!???????????
 
 
38- A 2 years old child with generalized oedema
and rash over the chest and abdomen. What is the
next investigation to do?
Urine dipstick
 
 

All FOP Recall till June 2021 with note 323 of 378
39- year old presented with annular scaly
lesion on back parents were applying
ketocanazole for two week.
nowpresentedwith fever headche.history
of travelling 3 weeks before. What is
caustive organism:
1. Borreliaburgdorferi Lyme disease

2.leishmania
3. Ricktesia
4…….
5……..
ANSWER: Borrelia burgdorferi

All FOP Recall till June 2021 with note 324 of 378
FOP FEB 2011
BOF:

1) A new born baby on neonatal examination has no abnormal


finding ,mother is insisting to take the baby home, on discharge
his mom is noted to have a bruise around her eye ,he is the
second sibling of 2yrs old who is found to b on the child
protection register, What will be ur next step?

A. Call the police


B. Contact the named midwife for baby protection
C. Call the health visitor and discuss her case
D. Let them go without intervention
E. (dun remember) but no option regarding social worker

2) 2year old boy seen by his gp and founfd to be malnourished


and odematous.his weight is >3SD weight for height.cocious but
miserable.he sends him to emergency where he got
admitted.urine dipstick was negative (BOF) What will u do…
A. Feed orally with cows milk formula

All FOP Recall till June 2021 with note 325 of 378
B. Feed with soya milk formula
C. Give 1ml/kg furosemide
D. Give human albumin 4.5% 20ml/kg E. 5ml/kg 10% dextrose

3) A 7 yr old girl has umbliacted papules on face and hand ,her


friends tease her and don’t let her play with her her Management

A. Reassure
B. Refer to cryotherapy
C. Topical silver nitrate

4)A 15-year-old girl presents complaining of an odd patch of skin


that she noticed on her left thigh and which has developed over
the past couple of weeks. On examination there is a very firm
and slightly indurated pale area of skin on her upper thigh, which
is a few centimetres across, and the lesion has an erythematous
border. The pale area of skin appears to have a rather atrophic,
glazed appearance. What diagnosis fits best with this clinical
picture?

All FOP Recall till June 2021 with note 326 of 378
Lichen sclerosus et atrophicus
Pityriasis vesicular
Dermatomyositis
Morphoea

All FOP Recall till June 2021 with note 327 of 378
EMQ:

Theme: Development regression


A- Hypothyroidism
B- Batten disease
C- Aminoaciduria disorder
D- Human immunodeficiency virus (HIV)
encephalopathy
E- Peroxisomal disorders
F- Lead encephalopathy
G- Subacute sclerosing panencephalopathy
H- Spieler–Mayer–Schrögen syndrome

All FOP Recall till June 2021 with note 328 of 378
I- Leigh’s encephalopathy
J -Hydrocephalus secondary to a medulloblastoma

Three children present to the child development centre for


assessment of developmental regression. The diagnoses listed
above are possible causes for developmental regression. Match
the three clinical scenarios to one diagnosis from the diagnostic
suggestions above.

Scenario 1 A 3-year-old boy and his parents review a hospice. He


presented previously with myoclonic jerks, optic atrophy and
progressive dementia. His parents have found it increasingly
difficult to control his myoclonic jerks and he is becoming
increasingly agitated, requiring sedation. A bone marrow sample
found lipofuscin in his marrow cells. What is his

diagnosis?

All FOP Recall till June 2021 with note 329 of 378
Scenario 2 A 14-year-old Iranian girl presents with a 6-month
history of developmental regression. She has regressed in all
categories of her development and now wears nappies both day
and night. She has ten words in her own language and appears
unable to communicate. There is no previous medical history to
note. She is unvaccinated. She has had all the usual childhood
rashes. On clinical examination, she is well. Her Griffiths mental
scale scoring portrays her subquotient results as follows: gross
motor 8.5 months, social skills 12 months, language and hearing
14 months, and hand and eye coordination 18 months; she was
unable to carry out the performance tasks. What is the most
likely cause of this girl’s developmental regression?

Scenario 3 An 8-year-old girl presents with a 12-month history


of early morning headaches with associated nausea and vomiting
before breakfast. She is otherwise fit and well. Her friends have

All FOP Recall till June 2021 with note 330 of 378
noticed that she does not participate in lunchtime meals as she
used to. On clinical examination, she is found to have increased
reflexes and clonus on the right side. On ophthalmic examination,
there is papilloedema of the left eye. What is her diagnosis?

Which of the following diagnoses is the most likely


in a 3-year-old boy who has epilepsy and, on
examination, has numerous depigmented macules
and two café-au-lait spots?
- Tuberous sclerosis
-Neurofibromatosis 1
-Ataxia telangiectasia
-Incontinentia pigmenti
-Sturge–Weber syndrome

All FOP Recall till June 2021 with note 331 of 378
1.7 year old with faltering growth bloated
abdomen , Anorexic .Small bowel biopsy confimrs
crohns disease What is first treatment of choice

a polymeric diet Elements diet


b.prednisolone
c.infliximab .
d.none of the above

All FOP Recall till June 2021 with note 332 of 378
All FOP Recall till June 2021 with note 333 of 378
October Exam 2013 ASM
Modified by me Dr Ahmed Tawfeek @ 6-2016 & adding some noes from other college about

June2015 exam

------------------------- red collore by me !!!!!!!!! or agree for selecting answer!!!!!!

------------------------------------------------------

1. Best antibiotic for:

• Newborn with listeria monocytogenes → ampicillin

• Salmonella infection

• A cystic fibrosis patient with acute pseudomaonas infection Tobramycin + ceftazidime

2. Most important side effects for If non pseudomonas —> cefuroxime


• Phenytoin →

• Methotrexate → hepatotoxicity

• Ibuprofen → nephrotoxicity

• Oral prednisolone → delayed growth

• Lorazepam → respiratory depression

• Thiopentane → CNS depression

3. Tidal volume for a baby weighing 3 kg

4. Sensations carried by the posterior column fine touch, vibration &


proprioception. Pressure is sometimes included, though less
frequently,  
5. CSF circulation → Opening CSF pressure is 25 mmHg (T/F), …………..

6. Calculation of the serum osmolarity Serum Osmolality = (2 x (Na + K)) + (BUN


/ 2.8) + (glucose / 18).
7. Site of action of furosemide

8. ECG physiology → P wave represents, T wave represents, PROGNOSIS interval represent, Q wave represents

9. Diseases transmitted by mosquitoes (T/F)

▪ Dengue fever

▪ Yellow fever

▪ Relapsing fever

▪ Rocky mountain spotted fever

▪ Typhus

10. Causes of bilious vomiting (T/F)


All FOP Recall till June 2021 with note 334 of 378
1
October Exam 2013 ASM
▪ Biliary atresia

▪ Malrotation x

▪ Malrotation x

▪ Volvulus
Executive secretary 015 toxin
11. Mechanism of diarrhea for shigella, rotavirus, E. coli

12. An asthmatic patient received salbutamol and Ipratropium bromide nebulizer. The patient improved but
developed nausea and headache with unequal pupils. The left one is 5 mm and the right one is 3 mm and
unresponsive to light but there is spontaneous eye movements. What is the best test to diagnose the underlying
cause:

▪ Application of pilocarpine eye drops

▪ Application of fluorescence eye drops

▪ CT brain

13. A newborn with jaundice secondary to ABO incompatibility, required exchange transfusion 2 times, later the
baby developed signs of cerebellar dysfunction. What is the affected tracts?

▪ Pyramidal tracts

▪ Cerebellar tracts

▪ Negrostriatal tracts Grey matter

▪ Spinothalamic tracts

14. An infant with congenital neutropenia with his family in a camp, the infant developed fever 38.5 and the
mother called you. What is the best advice to the mother:

▪ Go to the hospital as soon as possible

▪ Give paracetamol and observe the baby # to give paracetamol

15. Cardiac catheter question

16. Forest plot A forest plot is a graphical representation of a meta-analysis.

17. What is treatment of lyme disease at 8 years old male patient:

In addition, doxycycline is relatively contraindicated in children <8 years of age and in women
who are pregnant or breast-feeding.

or age <8 years, may be treated with rifampin for 7–10 days using a dosage regimen of 300 mg twice per day
by mouth for adults and 10 mg/kg twice per day for children (maximum, 300 mg per dose) (B-III). Rifampin-
treated patients should be closely observed to ensure resolution of clinical and laboratory abnormalities.
Because rifampin is not effective therapy for Lyme disease,

All FOP Recall till June 2021 with note 335 of 378
2
October Exam 2013 ASM
For prevention of Lyme disease after a recognized tick bite, routine use of antimicrobial prophylaxis or
serologic testing is not recommended (E-III). A single dose of doxycycline may be offered to adult patients
(200 mg dose) and to children ⩾8 years of age

▪ ampicillin

▪ cefuroxime Review Essential

▪ benzylpenicillin

▪ doxycycline

▪ ceftriaxone

18. For focal segmental glomerulosclerosis, ESRD will develop after how many years:

▪ 2 months

▪ 2-6 months

▪ 1-2 years

▪ 5-10 years 6-8 years

▪ > 10 years

19. Drugs that can be given via the UAC

20. What about that boy 3 years old presented to ER with burn 4 cm on his chest his mother said that he was
running to his grandmother and the tea is thrown up on his chest now he received opiate and is good the boil
now is 4 cm what to do:

▪ put ice

▪ give antibiotic

▪ IV fluids

▪ refer to dermatologist

▪ refer to social services

21. position of the eye in oculomotor nerve palsy With unilateral third cranial nerve palsy (ie, oculomotor nerve
palsy), the involved eye usually is deviated "down and out"

22. Circle of Willis

23. gastrin hormone

24. limping and groin pain scenarios

25. physiology of renal electrolyte transport

26. Mechanism of cooling in newborns with HIE Prevent apoptosis 33-34 temp.. cooling
within 3 days
27. Child presented with hemorrhagic manifestations. There is family history of 2 relatives died from intracranial
hemorrhage. PT and APTT are normal. What is the most likely diagnosis

▪ Check factor XIII

▪ Hemophilia

All FOP Recall till June 2021 with note 336 of 378
3
October Exam 2013 ASM
28. A baby boy is born to mother blue, heart rate 40/min, irregular breathing, mother is given morphine 1 hr
before birth, what is the most immediate step?

▪ Adrenaline

▪ Intubation

▪ Chest compressions

▪ IM naloxone

29. Baby with ambiguous genitalia, you told parents that you have to consult ... what is the next step:

▪ Karyotyping

▪ Send blood for U & Es

▪ first BMS then send laboratory for confirmation


30. Newborn baby blood glucose was low (1.9 capillary and 1.6 venous), next step is:

▪ Give bolus (dextrose 10%)

▪ Give oral feeds

▪ Do a septic screen

31. You are designing a clinical trial to test a new chemotherapy regimen. What is the most important message
that parents need to understand?

▪ If relapse happens, there will be numerous alternatives for treatment

▪ Your child will be looked after more than kids who r not in the trial

▪ We would like to know if the new regimen has good outcomes

▪ The expected results from this drugs is unknown

32. In which cases ? (T/F)

▪ Focal segmental glomerulosclerosis

▪ Nephrotic syndrome

▪ Post streptococcal glomerulonephritis

▪ IgA nephropathy

33. You are giving vancomycin 10 mg/kg every 6 hours. You check the levels (trough is 4.9 (normal < 5) and peak
was high (slightly), What do u do?

▪ make it every 8 hours (10 mg/kg)

▪ Decrease the dose to 8 mg/kg, every 6 hours

Answer ???? → Increase interval if trough level is high but in the Q peak level is high so the answer is to decrease the
dose

34. Mantoux test is which type of hypersensitivity? → I, II, III, IV or V

All FOP Recall till June 2021 with note 337 of 378
4
October Exam 2013 ASM
EMQ

Steven Jonson syndrome-->Carbamazepine
 Phenytoin
Acute dystonia-->metochopramide

Hirsutism----> ? Hirsutism; Increase male hair distribution phenytoin, steroid
Hypertrichosis; Increase all hair body cyclo

MCQ>> If there is renal disease, avoid which medication?



The answer was Ibuprofen

You are designing a screening programmed for obesity

- Waist circumference>90%

- wt and ht above 90 BMI > 99 severe obesity


BMI > 98% obesity
BMI > 91% overweight

FISH can be used to diagnose which of the following?

- Williams syndrome All mirodeleation


- Zellweger syndrome

Shifting of Oxygen dissociation curve to the left:

Diagnosis of Measles?
 All MMR saliva PCR


IgM

IgG

PCR

T or F quest. Teratogenic drugs:



A. Aspirin 
 PDA
B. Na valproate

C. Nicotine replacement therapy

D. thyroxine 

E. insulin

A 5 month old African boy brought in by mother that he is not moving his left leg, he has viral Flu like illness last
week, 2 days back he also fell from sofa, she suspected her elder son a 3yr old pulled him

On exam he has runny nose, and not moving his left leg and afebrile, what's the cause?

Osteomyelitis

Non Accidental injury

Transient synovitis ?????
 onset at 3y
Sickle cell disease

a 3 week baby with poor wt gain (his wt was 3.6kg). He is formula fed and his grandmother is giving him 100 ml
every 4 hours strictly. He takes his feed well but he is miserable. What to do?

Give 140 ml every 4 hours


All FOP Recall till June 2021 with note 338 of 378
5
October Exam 2013 ASM
Demand feeding

Follow on formula milk
 fortify on formula milk
?

?

4 hrs is too long

On demond feeding is principal for breastfeeding & formula on condtion do not decrease total caloric intake

T/F Sudden unexpected death in infancy:



Apnea monitor decreased the incidence. (F)

Smoking,,,

MCQ: Keep PDA open is most important in:



Critical aortic stenosis

Critical pulmonary stenosis

Tricuspid atresia

?
 all can be = duct dependent
?

T/F regarding MMR vaccination:



Grandmother can consent

Dad who is separated but divorced
 Father
A stranger......!

A sister (can't remember the age)

? 2003 even father divorced can
consent

* A mother of a 5 yr old who has eczema noticed a swelling beneath her ear while combing his hair?

Refer to dermatologist

Reassure n send home

Refer to otolaryngologist

Review after 6 wks

A 5 yr old girl is brought by mother, she has lost 5 kg weight for past few months, agitated and can't sleep in night?

Prescribe some sleep remedy
 ————
Malignancy

Hyperthyroid
 usual onset of hyperparathyrodism
Abuse 20_50 year

Answer → ???? malignancy (this is not the age of hyperthyroidism and leukemia can present with bone pain at night
and pt cant not sleep)

Exam question 1B



Abducent


All FOP Recall till June 2021 with note 339 of 378
6
October Exam 2013 ASM
Mandibular

Oculomotor

Mandibular

Lacrimal

Zygomatic

Vagus

Troclear

Scenarios

1)which branch of trigeminal has both sensory n motor supply he mandibular nerve has sensory and motor
functions.

2)branch of which nerve if damaged cause postoperative stridor
 Vagus
3)which nerve is the first to damage in case of increase Cranial pressure Abducent

Which is not a part of circle of Willis?



Vertebral arteries ‫
ﺳﮭﻞ‬
Internal carotid

Anterior communicating artery

Posterior communicating artery

(The Circle of Willis is a part of the cerebral circulation and is composed of the following arteries:

Internal carotid artery (left and right)

Anterior cerebral artery (left and right)



Anterior communicating artery

Posterior cerebral artery (left and right)

Posterior communicating artery (left and right)

The basilar artery and middle cerebral arteries, supplying the brain AND vertebral arteries, are not considered part of
the circle

?????????????????????

A 3 year old girl, whose elder sis is subjected to sexual abuse, complains of perineal soreness which sign on exam
confirm sexual abuse

Vaginal discharge

Torn hymen

Perianal warts

Strept infection on swab

Which of the following is contraindication for circumcision: T..F



1. Paraphimosis

2. Hypospadius

3. Napkin rash

4. Cystitis

5.

All FOP Recall till June 2021 with note 340 of 378
7
October Exam 2013 ASM
Patient with atopic eczema and itchy skin lesion on top of eczema then manifestation of cerebellar ataxia started to
appear:
 Post infection cerebellitis lead to ataxia
1 post infectious encephalomyelitis

2 chicken box

3 herpes
 affect focal in temporal lobe
4 ...

5....

Which of the following is most likely to confirm diagnosis of Pyloric stenosis :
 All true except
1 Hypochloremia 

2. Hypokalemia 

3. Non bilious vomiting 

4. Metabolic acidosis 

5. Increased urea

????M acidosis

Prescription writing answer (T/F)



1- Dose should be written as 0.5ml (True) 

2- Dose in microgram can be abbreviated as 'mcg' (False) 

3- Latin abbreviations can be used. (True) 

4- Dose less than 1 gram should be written in mg (True) 

5- Doses in millilitres are recommended in place of cubic mm (True)

Cause of congenital diaphragmatic hernia

1. Failure of closure of septum transversus

2. failure of the GIT contents to return to the abdomen

Extended Matching Questions (EMQ)

❖ 1/ EMQ (paper B- PHARMACOLOGY)

options

▪ Muscrininc antagonist,

▪ dopamine antagonist,

All FOP Recall till June 2021 with note 341 of 378
8
October Exam 2013 ASM
▪ H1recptor antagonist,

▪ canabinoid recteptor antagonist,

▪ HT5 recptor antagonist.

mechanism of action of the following drugs:

• Domperidome.

• hyoscine hudrbromide.

• Ondesartan rececptor .

❖ 2/ EMQ (PAPER B – MICRO_PHARMA)

OPTIONS:

▪ pseudomonas

▪ aspregillus

▪ mycoplasama

▪ …..

▪ ……..

▪ ……….

DRUGS:

1-Tazocin =pipracillin/……..

2-azithromycine

3-amphotricin

❖ 3/EMQ (PAPER B- STATISTICS)

OPTIONS:

TABLE 2-2

▪ Incidence.

▪ senstivity .

▪ specificity.

▪ positiv predictive value.

CHOOSE ONE :

1. a/a+c
All FOP Recall till June 2021 with note 342 of 378
9
October Exam 2013 ASM
2. c

3. a/a+b

❖ 4/ EMQ (PAPER B- ANATOMY)

OPTIONS:

• Right atrium

• Tricuspid valve

• Inferior vena cava

• Superior vena cava

Anatomical structures:

1. Coronary venous sinus.

2. Ductus venosus.

3. Strcture open on y descent in JVP.

❖ 5/ EMQ(PAPER B)

❖ one emq about genetics

❖ 1-4 years old child present with picture of rickets and his mother has genu varum

❖ this is xlinked dominant

❖ 2-one questation about prader willi

❖ parental disomy

❖ 3-one questation about

❖ hypotonic child his uncle has cataract and his mother doesn't smell when she meets you

❖ this gene anticepation(trinuclotide repeate disorder)

❖ 6/ EMQ(PAPER A)

about immunization

1- 3 months old boy with his young brother treated from leukemia
oral polio will excreted in stool = live attenuated will affect the
dapt+hib+ipv immune compromise brother

2-baby boy with hiv what is contrindicated vaccine

bcg Yellow, typhoid


3-immunization given b4 splenctomy

ppv not pcv PPV uncongogated

All FOP Recall till June 2021 with note 343 of 378
10
October Exam 2013 ASM
❖ 7/ EMQ

❖ one emq about investigation:

❖ 1-picture of leckemia i dont remeber the ques

❖ blood film is the investigation

❖ 2-one questarion about coeliac dis

❖ antitissue transglutaminase

❖ 3-one queatation about child with bloody diarrhea renal function impaired

❖ blood film also HUS

❖ 8/EMQ

❖ one emq about bile stained vomiting:

❖ 1-1day old baby hypotonic looks dysmorphic with bile stained vomiting

❖ down syndrome --------biliary atresia Duodenal Atresia

❖ 2-6 days with progressive abdominal distension since birth pass stool once time only now present with bile
stained

❖ vomiting

❖ hirshsprung disease

❖ 3-4days child vey well breast feed today in the morning his cousin has cyctic fibrosis present with vomiting
capillary refill time 4 sec

❖ malrotation and volvolus Meconium Ilius

❖ 9/EMQ

❖ one emq about sick day rules in diabetes

❖ 1- 14 well controlled diabetic has infection now rbs=21, urine ++ ketone → i answered it wrong it should be
give extradose of insuline

❖ 2-one questation about morning hyperglycemia, smogy phenomenan should decrease basal insulin

❖ 3-4 year child fever 39 with vomiting and abdominal pain and nauseated, RBS= 6, urine no ketones → just
increase fluid intake

❖ 10/EMQ

❖ 14 years old girls take overdose of unknown drug

All FOP Recall till June 2021 with note 344 of 378
11
October Exam 2013 ASM
❖ 1-hot flushed hypertensive tacchycardia convlsion occure in ambulance

❖ i answer it methyphenadit cz this picture of amphetamine toxicity

❖ 2-girl with vomiting buy tablets from nearby super market all vital signs are normal

❖ i was confused between paracetamol and aspirin but i found respiratory rate normal so i choose paracetamol

❖ 3-girl with vomiting stained with blood and hypotensive

❖ this ferrous sulfate

❖ 11/EMQ

1-newborn with rash liver and spleen palpable 2cm below costal margine

healthy mother maternal platelet count is

194000

2-newborn with rash liver and spleen are just palpble health mother

Maternal platalt count is 23000

❖ 3-new born with rash and no organomegaly maternal blood count is 95000

❖ 12/EMQ

❖ one emq about pain management:

❖ 1-4 year old boy with burn

❖ i answer it iv morphine

❖ 2-child with accident and u request ct scan but orthopedic want to take him first to do thomson split

❖ I answer it femoral nerve block

❖ 3-girl with headache now receive 1gm paracetamol every six hour and brufen 400 mg every 8 hour wt will u
give her

❖ i answer no treatment bcz this is rebound headache

❖ 13/EMQ

one emq about hair falling and treatment

1-one was alopecia areata and i choose treatment with topical steroid

2- one was tinea capitis and choose oral grasiofluven

3-one was telgon effluvium this girl which has pneumonia 3 monthes ago

All FOP Recall till June 2021 with note 345 of 378
12
October Exam 2013 ASM
❖ i choose no treatment reassurance

❖ 14/EMQ

❖ One cardiology emq

❖ About wt is the sign u will found

❖ 1-baby with continous murmur was pda and the sign is bounding pulse

❖ 2-baby also with systolic murmur and rt bundle branch this is asd and sign is fixed splitting s2

❖ 3-15 year old girl with systolic murmur on lt second intercostal space and continous murmur on back this is
pulmonary atresia with asd and collatralls im not sure about it but I choose single s2 bcz pulmonary valve is
atreric

❖ 15/EMQ Coarctation of aorta

Parents with balanced translocation carry and cause


to case child with Down syndrome

Best of Five (BOF) The cause


1. 25 years old woman married to 51 years old man, what is most probably the cause of their baby having down
syndrome?

a) Balance translocation 46

b) Unbalance translocation 47 associate in older age parents

c) Paternal disomy ?????????/ In microdeletion

d) Maternal disomy

All FOP Recall till June 2021 with note 346 of 378
13
October Exam 2013 ASM
e) Mosaic

2. A 9 years old girl with Hashimoto’s thyroiditis. Her laboratory tests showed high TSH level and normal T3
level. What is the another test that is helpful in the diagnosis?

a) Anti-thyroid peroxidase antibodies Graves’ disease

b) Anti-thyroglobulin antibodies

c) Free T4 level

d) …………………

e) ………………...

Thyroid autoantibodies: Presence of typically anti-TPO (anti-thyroid peroxidase) and anti-Tg (anti-thyroglobulin) antibodies
delineates ‫ ﯾﺣدد وﯾﺻف‬the cause of hypothyroidism as Hashimoto thyroiditis or its variant; however, 10-15% of patients with
Hashimoto thyroiditis may be antibody negative

3. A child with constipation and impacted feces up to the umbilicus. What you will do?
————————
a) Start laxative and review

b) Enema to relieve impaction

c) Admit to the ward for disimpaction

???????????????????/

NICE 2014 :;;Do not use rectal medications for disimpaction unless all oral medications have failed and only if the
child or young person and their family consent.

4. A 15 days old baby with an umbilical stump granuloma, 2 mm in size, and oozing a serous fluid. What is your
intervention?
small size , clear fluid
a) Reassurance and review

b) Refer to surgery

c) Silver nitrate cauterization

d) Antibiotic powder

5. A child with swelling of both knee joints and one elbow joint. To which doctor you should refer the patient?

a) Orthopedic

b) ENT

c) Physiotherapy

d) Ophthalmologist (i.e. a case of oligoarticular arthritis, for fear of anterior uveitis)

e) Surgery

6. A 3 years old child with a small membranous tongue tie. He has a difficulty in speaking the letters B, R, T. his
———————————
grandfather refused to do surgery. To which doctor you should refer the patient?

a) Occupational therapist

b) Speech and language therapist

c) Surgery
Ask child to pronounce I,
th, S, D, T
All FOP Recall till June 2021 with note 347 of 378
14
October Exam 2013 ASM
d) …………..

e) …………………

7. A 10 years old child with oral painful mouth ulcers and anal fissures. What is your diagnosis?

a) Crohn’s disease

b) Ulcerative colitis

c) Celiac disease

d) ……………….

8. A 2 years old girl with an injury to the vulvar area and mild bleeding, while cycling in the park with her
brother. What is your next step?

a) Call your senior pediatrician

b) Refer to a gynecologist

c) Suturing the vulvar wound

9. A 10 years old tall boy with gynecomastia and small testes. What is your possible diagnosis?

a) 47 XXY

b) 46 XO

c) Marfan’s syndrome

10. A 6 months old infant with facial eczema, found to have a palpable LN over the neck, which is about 3 cm in
size, smooth, and non-tender. What you will do?

a) Refer to a surgeon < 2 size

b) Reassurance

c) Start antibiotic therapy

11. A full-term neonate, delivered by LSCS, and developed respiratory distress 2 hours after birth. What is the
possible cause of respiratory distress?

a) Transient tachypnea of newborn

b) Respiratory distress syndrome

c) Meconium aspiration syndrome

12. What is the most important investigation before starting treatment with sodium valproate?

a) Liver function tests Hepatotoxic


b) Serum ammonia level

13. A small child with painless, soft, cystic scrotal swelling. Both testes are palpable. What is your possible
diagnosis?

a) Hydrocele

b) Torsion testes Sample paper


c) Incarcerated inguinal hernia

All FOP Recall till June 2021 with note 348 of 378
15
October Exam 2013 ASM
d) Mumps orchitis

e) Epididymo-orchitis

14. A small child with left testicular mass, prominent pallor and bruises. What is your possible diagnosis?

a) ALL

b) Testicular tumor

15. A 2 years old child with croup. Oxygen and budesonide were given but no improvement. What is your next
step? oral dexamethasone
a) Nebulized adrenaline, 400 mcg/kg, of 1:1000 solution

b)

16. A 3 years old child with fever, vomiting, and dysuria. Abdominal US revealed renal stones. Urine routine
—————————
showed leukocytes 2+, protein +, and trace blood. What is your possible diagnosis?

a) Nephrocalcinosis

b) Renal tubular acidosis

c) Proteus UTI features of UTI


??????????????????????????????????????????

17. A 2 years old boy with severe pain during night and was crying excessively. The parents can calm the baby
with milk. He was pale with bruises over the chin, but an active child. What is your possible diagnosis?
—————-
a) Trauma

b) ALL

c) Chronic hemolytic anemia

d) Iron deficiency anemia

18. A 3 years old child with a small VSD. He has to do adenoidectomy. What you will do for prophylaxis against
infective endocarditis?

a) Give amoxicillin before the procedure

b) No prophylaxis is indicated

19. A child with ptosis, meiosis, and anhydrosis. What is your possible diagnosis?
—————————-
a) Horner’s syndrome

b) ………….

20. A 6 weeks old infant with jaundice and pale stool. ALT, AST, and conjugated bilirubin levels are elevated.
What is your immediate action? not sure it is biliary atresia, may be cholestais

a) Intramuscular vitamin K hematoma

b) Intravenous vitamin K

c) Oral vitamin K Can’t absorb dt defect in enerohepatic circul

d) Vitamin A
U have to do US and HIDA then if confirmed diagnosis —> transfer to
e) Surgery special liver services for surgery (fop case 36)
All FOP Recall till June 2021 with note 349 of 378
16
October Exam 2013 ASM
??????????????????????????????

21. A 5 years old child suddenly collapsed after entering the swimming pool, but he regains conscious few
minutes after transfer to the hospital. His uncle had a similar attack 10 years ago and had a long term follow
up with a cardiologist. What is your possible diagnosis?

a) Prolonged QT syndrome

b) Hypertrophic obstructive cardiomyopathy

c)

22. A 5 mo old infant presented with developmental delay. He was born vaginally, with a birthweight of 4.5 kg
and APGAR score of one at 1 minute and 8 at 5 minutes. What is the possible cause of developmental delay?

a) Inflicted brain injury IDM


b) Congenital brain malformation

c) Hypoglycemia Cause developmental delay


d) Hypoxic ischemic encephalopathy APGAR score ok

23. A child with bloody diarrhea, anemia, and poor urine output. What is the next investigation to do?

a) Serum complement levels

b) Serum creatinine level

c) Serum urea level

May s creatin for triad of HUS ??????????????/

24. An IUGR baby with periventricular calcifications on the CT scan. What is the immediate complication?

a) Hearing loss

b) Visual impairment

c) Neurodevelopmental sequelae

??????????CMV as hearing loss may bedelayed after infancy!!!!!!!!!!!!!???????????

25. A 2 years old child with generalized oedema and rash over the chest and abdomen. What is the next
investigation to do?
Henuch scheine purpura
a) Urine dipstick

b)

26.

59/ BOF

All FOP Recall till June 2021 with note 350 of 378
17
October Exam 2013 ASM
And other about child with left testicular mass and brusies

SUGGESTED ANSWER: leukemia

1/ BOF (PAPER B-Statistics)

▪ Q @ Relative risk.

2/ BOF

▪ Q@Odds ratio.

3/ BOF

▪ Q@Numebr needed to treat.

4/ BOF

▪ Q@Confidence interval.

5/ BOF

▪ Type of study

6/ BOF

PT FAIL TO ABDUCT EYE

SUGGESTED ANSWER : SIXTH NERVE PALSY.

8/ BOF(PAPER B- )

Consequnces of cerebral calcification in CMV……… Repeat

1.seizures immediate effect


2.neurodevelpmental disablity, affect movement
3.neurodvelpmntal delay,

4.globaldevelpmental delay affect all include hearing and vision


All FOP Recall till June 2021 with note 351 of 378
18
October Exam 2013 ASM
5………………

10/ BOF (PAPER A )

Girle 6 y with unilat breast mass .. pubertal development show breast 2 ...all rest 1

1. precoius puberty

2. unilat mastitis

3.premature thelarche can be unilateral or bilateral


4. premature adrenarche

SUGGESTED ANSWER: premature thelarche.

11/ BOF(PAPER B) Dermatophyton


8 year old presented with annular scaly lesion on back parents were applying ketocanazole for two week.now
————-
presentedwith fever headche.history of travelling 3 weeks before. What is caustive organism:

1. Borrelia burgdorferi traveling to beach

2.leishmania

3. Ricktesia

4…….

5……..

ANSWER: Borrelia burgdorferi( lyme dis.)

12/ BOF(PAPER B)

What arteries contrbute in Circle of willis?

1.basilar

2. vertebral.

3. internal carotid.

13/ BOF(PAPER B)

Side effects of frequent usage of Furosemide?

1. Hypomageseamia.

2. Hypocalcaemia.

All FOP Recall till June 2021 with note 352 of 378
19
October Exam 2013 ASM
3. Nephrocalcinosis Chronic use
4. …….

5. ……….

Metabolic side effects including hypokalemia, hypomagnesemia, and an increase in serum uric acid, have
been relatively common especially with higher doses. 

14/ BOF(PAPER B)

Action of Spironolctone?

1. Compete aldosteron

2. ………..

3. ………..

4. ………….

5. ………….

Competitively attach to aldosterone receptors & prevent its action

15/ BOF(PAPER B)

A girl with half facial weakness, was lower motor facial palsy with convergent squint?

1. cerebraltumour .

2. benign idiopatic hypertion.

3. oculomotor nerve plsy

4.bells palsy,

5.pontine tumor,

ANSWER: pontine tumor

16/ BOF(PAPER B)

narrowest part of upper airway?

ANSWER: SUBGLOTIS

17/ BOF(PAPER B)

All FOP Recall till June 2021 with note 353 of 378
20
October Exam 2013 ASM
Cardiac cthetrization showing BP AND SATURATION in all chambers of heart and main artries what is result
interpretation?

1. Aortic regure.

2. Pulmonary diltation.

3. Aortic stenosis.

4. ……………

5. …………….

ANSWER: AORTIC STENOSIS.

18/ BOF(PAPER B)

FEV 1 ,PEFR ,FVC....results interpretation to differentiate b/w restrictive and obstructive lung disease

ANSWER: OBSTRCTION LUNG DISEASE.

19/ BOF(PAPER B)

ECG changes in AVSD?

Superior axis
SUGGESTED ANSWER: CROHN S DISEASE.

21/ BOF(PAPER B)

Gentamicin....drug levels adjustment...pre dose n one hour post dose values given ......we had to increase or decrease
the dose according to drug levels

23/ BOF

craniopharyngioma cause which field defect?

i choose bitemporal hemianopia

24/ BOF

about formula ... premature infant now 3 m old... weaning completly from breast feed since 1 month ..have diarroea

start from 10 days give :

-premature formula

All FOP Recall till June 2021 with note 354 of 378
21
October Exam 2013 ASM
- soye formula

- cow f

25/ BOF

anaomaly in gastrochiasis(complete abscence af anterior abdominal wall layers)

26/ BOF

most common type of oesophageal atresia

atresia with distal fistula

27/ BOF

pathophysiology in vsd

28/ BOF

one questation was about congenital chloride darrhea



chloride was low and he has metabloic alkalosis 
 High chlor in stool
he is 4 weeks with persistent diarrhea and history of ployhydrammnois
All FOP Recall till June 2021 with note 355 of 378
22
October Exam 2013 ASM

31/ BOF(PAPER A)

that for pregnant women with hepatitis a and will have labour after one week

all year u study hepatitis b and in exam u find it hepatitis a

SUGGESTED ANSWER: LOW RISK OF TANSMISION.

32/ BOF

also another questation about absence epilepsy & Juvenile myoclonic


which drug if used will increase the frequency of attack?

i choose carbamazepin

33/ BOF

apnea breath test for brain stem death

??????????????// a posi2ve apnea test (lack of spontaneous respiratory efforts in the presence of an elevated PaCO2)

35/ BOF

PT WITH EGG ALLERGY WHAT TYPE OF HYPERSENSTIVITY?

1. hypersenstivity reaction type 1.

2. TYPE 1 AND 4.

3. TYPE 2

4. TYPE 3 all food allergy IgE mediated except


5. TYPE 4 cow milk has 2 type allergy

36/ BOF

PT WITH NEURODEVELOPMENTAL REGRESSION AND USING OF HANDS FREQUENTLY WHAT IS THE


DIAGNOSIS?

SUGGESTED ANSWER: Rett syndrome

37/ BOF

All FOP Recall till June 2021 with note 356 of 378
23
October Exam 2013 ASM
wischot aldrich syndrome

38/ BOF

klinfilter syndrome

39/ BOF

children complain from abdomnal pain and fever headache abdominal examination mild right upper quadrent

this questaion was on pastest

i choose lobar pneumonia

40/ BOF(PAPER A)

1-neonate born cs mother taking methadone baby not responding no breathing HR=80

i choose inflation awith bag

!!!!!!!!!!!!!! methadone is an opioid

41/ BOF(PAPER A)

2- 3 years old boy found on floor of shop no other one with you wt u will do

i choose shout for help safety


stimulate
ventilate 5 time
42/ BOF(PAPER B) shout for help
How to differrentiate between dislocatable hip and dislocated hip?

positive barlow test

Barlow manoeuvre: tests for dislocatable hip

The hip is held fl exed and adducted. The femoral head is pushed

downwards. If dislocatable, the femoral head will be pushed posteriorly

out of the acetabulum.Ortolani manoeuvre: tests for dislocated hip

Abduct hip with upward leverage of femur. A dislocated hip will

return with a palpable clunk into the acetabulum.

If examination is questionable hip ultrasound is undertaken at

4–6 weeks of age. Urgent orthopaedic referral is required for a

dislocated or dislocatable hip.

All FOP Recall till June 2021 with note 357 of 378
24
October Exam 2013 ASM
43/ BOF(paper A)

children looks mildly dysmorhic and mild learning disability and was taken into foster care for social causes wt is ur
they r check mother alcohol level if high .. they will
diagnosis?
take the child
i choose fetal alcohol syndrome

44/ BOF(PAPER B)

mechanism of action of adrenaline in anaphy;axis

rise peripheral vasculare resistannce

45/ BOF(PAPER A)

Gp see newborn will examine it he found click in hip but when registrerar examine her again was normal wt will u do?

Us

Review after 6 monthes

Refer to orthopedic

Do not remember the other

?????????????if at doubt do US after 4-6 wks

screen test if positive —> US —> orthopedics


if negative —> review at 6 month with US
46/ BOF

one bof about obese girl with elevated liver enzyme

i choose non alcoholic steatohepatitis

48/ BOF(PAPER A)

Q about assesment of hearing .. what most important event to search for


All FOP Recall till June 2021 with note 358 of 378
25
October Exam 2013 ASM
I choose bilateral otitis media affected him 2 weeks ago

49/ BOF
133
ONE QUES ABOUT TUBEROUS SCLEROSIS

4 MONTHE CHILD WITH ATTACK OF FLEXTION POOR INTERACTION AND HIS MOTHER TREATED

FROM HTN

50/ BOF
134
in ca absorption which is under direct effect of parathyroid hormone?

-1 alpha hydroxylation

-kidney...absorptuon...of.....phosphate

-intestinal reabsorption

- bone resoption is correct

51/ BOF(PAPER A)
135
Q about anemia ... child ..his mother and father farmers and

Hb ? ..mcv ?

My answer is lead poisoning

53/ BOF 136


this is the 4th sample and contain

9000 rbcs

glucose 6mmol

protien 0.7

two vague answers

trumatic tape

subarachnoid hge

i choose subarachnoid he

bcz this is the 4th sample taken

if u exaime csf sample and found rbcs u should take 3 concecutive sample and rbcs count should be decrease in every

sample
All FOP Recall till June 2021 with note 359 of 378
26
October Exam 2013 ASM
but in the 4th sample it still 9000

137
54/ BOF

one bof about wy cow milk should be modified?

SUGGESTED ANSWER: high phosphate content

55/ BOF(PAPER A) 138


38w newborn after c.s with tachyapnea HIS APAGAR WAS 5 AFTER 1 MINUTE AND BCM TACCYPENIC
AFTER 4 HOURS

SUGGESTED ANSWER: TTN

56/ BOF139
one bof about complication of infant of diabetic mother

- microcolon

- sacral agenesis most common sacral agenesis


-other choices

57/ BOF 140


ABOUT GIRL ATHELTE COMPLAIN FROM AMENORRHEA FOR LAST 6 MONTHES

SHE IS THIN SLIM AND EXCRCISE 2 TIMES PER DAY

JOINED TO OLYMEPAD TEAM


Anorexia nervosa
BODY MASS INDEX 18
BMI < 17.5
- ATHELETE AMENORRHEA

- Anorexia nervosa

141
61/ BOF

ONE CHILDREN TREATED FOR RESPIRATORY INFECTION FOR 5 DAYS WITH AMOXICILLIN AND NOT
IMPROVING HE HAS PAINFULE NODULE ON CHIN OF TIBIAWT IS THE MOST COMMON ORGANISM?

-MYCOPLASMA

-TB

All FOP Recall till June 2021 with note 360 of 378
27
October Exam 2013 ASM
142 62/ BOF

ONE PHARAMA QUES

ABOUT BOY TREATED FOR TB AND NOW CANT DIFFERRENTATITE BETWEEN COLOURES

WT IS MOST DRUG TO CAUSE?

I CHOOSE ETHAMBUTOL(IT CAUSE OPTIC NEURITIS)

63/ BOF 143


ONE BOF ABOUT CHILD WHO SET NEAR TV AND OTOMERTIST FOUND VISUAL ACUITY IN LT EYE

6/18 AND RT EYE 6/6 AND HE HAS RECURRENT OTITIS MEDIA AND FOLLOW UP WITH ENT AND HAS

MULTIPLE BROWN PIGMENTED ON BACK

WT IS THE CAUSE OF IMPAIRED VISION? Optic glioma


I CHOOSE OPTIC NERVE GLIOMA(I THINK THIS MAY BE NEUROFIBROMATOSIS

64/ BOF 144


children has appendictomy receive fluid for 48 hour 100ml kg now bcm confusion and headache serum sodium was

low wt type of fluid u will used:

i answer 20%mannitol

65/ BOF
145
BRAIN STEM APNEA TEST

dxt should be more than xxmol/L (i think was 7)



patient must be put on room air for 10 minutes

PCO2 must be within xx mmHg range (sorry i really cant remember the values)

All FOP Recall till June 2021 with note 361 of 378
28
October Exam 2013 ASM
A. Procedure of the apnea test

Disconnection of the patient from the respirator

If continuous or intermittent oxygen supply is preceded by denitrogenation of blood gases, high PaO2 levels
can be sustained for very long periods of time.[26] Preoxygenation removes alveolar nitrogen stores and
facilitates oxygen transport.[27] There are several techniques for ascertaining that there is sufficient
oxygenation during AT.[15] The first method is to disconnect the patient from the respirator and to insert a
catheter or cannula into the endotracheal tube down to the level of the carina and provide pure oxygen at a
rate of 4–10 l/min. This would ensure sufficient alveolar ventilation and transport of oxygen to the blood
even without any respiratory movements.[17]

In the second procedure the patient is not disconnected from the respirator but the minute volume is
decreased to a very low level (0.5–2 l/min), with the respirator in the synchronized intermittent mandatory
volume ventilation mode and with pure oxygen provided for inspiration. In this procedure, the patient is not
disconnected until the required PaCO2 is achieved. Lang and coworkers prefer this method as it prevents
tracheopulmonary complications and allows the examiner to detect any spontaneous respiratory effort.
[14,15] Al Jumah et al, have proposed a third procedure of biphasic intermittent positive airway pressure
(BIPAP), a method known as ‘bulk diffusion’.[28]

66/ BOF146
FACIAL NERVE PALSY

1. DROOLING OF SALIVA AT SAME SIDE

2. ……………

3. ………………..

4. …………….

147
5. ………..

67/ BOF

Q ABOUT PT. WITH KAWASAKI DISEASE ON ASPIRIN CAME WITH PROLONG PT AND APTT WHAT
MOST PROPLY THE CAUSE?

Ryes Syndrome
SUGGESTED ANSWER IS: LIVER DYSFUNCTION. due to Aspirin

68/ BOF(PAPER A)
148
this boy 8 days come with poor feeding and vomiting and deepening jaundice

at 4 day present with jaudice and put under phototherapy and at 6 day go out of incubator noramlly

temp35

ABG show metabloic acidosis

glucose 2.1mmol

All FOP Recall till June 2021 with note 362 of 378
29
October Exam 2013 ASM
bilirubin 201mmol conjugated 8

liver palpable 2cm below costal margine

what is ur diagosis?

1-hypoplastic left heart

2-glycogen storage

3-sepsis

4-kernictrus

my explanation that this is case of galactosemia and present with e coli sepsis
————
and i choose sepsis

149
69/ BOF(PAPER B)

ORDER OF CORTICOSTERIOD ACCORDING TO POTENCY

150
HEBD
Hydrocortisone
70/ BOF Ermovete
Betnovet
PREDICTION OF METABOLIC SYNDROM Dermovete
Low birth weight coupled with adult obesity is a strong determinant of the metabolic syndrome in postmenopausal
women.

MCQS

1/ MCQ 151
Q/RECOGNISED CAUSES OF NOSE BLEEDING IN CHILDREN?

1. VON WELLBIRAND DISEASE.

2. HEAMOPHILIA

3. UPER RESPIRATORY TRACT INFECTION.

4. ……………..

5. ………………

All FOP Recall till June 2021 with note 363 of 378
30
152
October Exam 2013 ASM

2/ MCQ

Q/RECOGNISED CAUSES OF ALKALOSIS? All

1. ANXIETY. Respiratory alkalosis

2. PYLORIC STENOSIS.

3. DIURETICS.

4. …………..

5. ……………..

3/ MCQ (PAPER A) 153


unicef .......... for breast milk

4/ MCQ(PAPER A) 154
POLICY TO WRITE PRESCRIBITION OF DRUGS?

1. U sing latin abbrevation/term in prescription is acceptable



mcg stands for micrograms

2. all prescription less than 1gram should be written in microgram



0.5mg is an acceptable form of prescription

3. H ow to write 0.5 mg in numbers or letters

4. …….

5. …….

5/ MCQ(PAPER B) 155
Q about CA and NA and phosphate and CL and k which is more intracellular? King

6/ MCQ (PAPER A) 156


MILE STONE IN 3 YEAR OLD BABY

7/ MCQ

Q ABOUT RSV
157
1. INCUBATION PERIOD.

All FOP Recall till June 2021 with note 364 of 378
31
October Exam 2013 ASM
2. ………….

3. ………..

4. ………….

5. ………….

The incubation period-the time from exposure to RSV until you have symptoms-ranges from 2 to 8 days but
usually is 4 to 6 days.1

8/ MCQ 158
Q ABOUT FASCIAL NERVE

1. HAVE SIX TERMINAL BRANCHES x only 5

2. NERVE TO STAPIDUS RUNING IN EXTERNAL CANAL arises at facial canal

3. SUPPLY POSTERIOR 2/3 OF THE TONGUE. x

4. ……………

5. ………………

• intermediate nerve
near origin
• geniculate

• greater petrosal
• pterygopalatine ganglion

inside

facial canal
• nerve to the stapedius
• chorda tympani
• lingual nerve
• submandibular ganglion

• posterior auricular
• suprahyoid
• digastric
• stylohyoid

at stylomast
oid
 • parotid plexus
foramen • temporal
• zygomatic
• buccal
• mandibular
• cervical

All FOP Recall till June 2021 with note 365 of 378
32
October Exam 2013 ASM

• Facial motor nucleus

Nuclei • Solitary nucleus


• Superior salivary nucleus

9/ MCQ 159
RESTRECTIVE LUNG DISEASE VERSUS OBSTRUCTIVE LUNG DISEASE

10/ MCQ 160


SIDE EFFECTS OF ADENOSINE.

11/ MCQ
161
SIDE EFFECTS OF PROSTAGLANDIN

12/ MCQ

MET HB
162
13/ MCQ 163
IMMUNITY IN NEONATE VS IMMUNITY IN CHLIDREN

14/ MCQ 164


HB-O2 DISOCIATION CURVE

15/ MCQ 165


21-HYDROXLASE DEFICIENY

All FOP Recall till June 2021 with note 366 of 378
33
October Exam 2013 ASM

16/ MCQ 166


LIVE ATTENUTED VACCINES

17/ MCQ 167


SENSATION CARRIED ON POSTERIOR COLUMN

18/ MCQ 168


LIFE THREATING ASTHMA

All FOP Recall till June 2021 with note 367 of 378
34
October Exam 2013 ASM

the lovely big topic of acid base and electrolyte imbalance

1-addisone disease
169
2-diabetes insipdius

3-SIADH

4-congenital adrenal hyperplasia

5-pyloric stenosis

6-habitual drinks

alot of questation in hematology

2 queation about g6pd


170
one about hereditry spherocytosis

3 about leukemia

2 about hemolytic uremic syndrome

171 1. causes of normal anion gap

2. mechanism of action of adenosine , adalimumab , Etanercept , dopamine 172


173 3. child 3 years with elevated creatinine kinase , to confirm which investigation

175
4. Q forest plot
5. Ophthalmology pathway defects 174
6. Differences between pre renal and renal failure
7. Hb dissociation curve
8. About billirubin :
1. most are carry by albumin
179 180
2. turn to billiverdin before inter bile system ( didn’t remember the other options)
9. physiology of neonates : sweat gland formation , evaporation from skin
10. causes of white shadows in TAPVD x ray
11. pain management ( EMQ )
--------------------------------------------
183

All FOP Recall till June 2021 with note 368 of 378
35
October Exam 2013 ASM

MRCPCH Exam june 2015


RH. fever recently diagnosis : inventigastion ?? a.ASOT,b.ESR,c.ECHO
ASOT

184
185 sever sudden abdominal pain with vomiting ,ass with pain left testicle ?diag: a. LF tortion of tests, b. tortion of
morgagni Testicular Torsion

neonate had vomting ( 3 senario ) :1,day 1 vomiting bilios ,2. day 6 bilious vomiting ,3. may be day 2 non-bilious
vomting Duodenal atresia Volvulus Reflux

186 Valvulus
Miconium ilius in CF
Hitshprung Overfeed
Sepsis

3 senario about eye condition : night blindness,options : retinitis pegmentosa,retinal detachment,retinal

187
hge,clomboma,bilatral cataral,nystagmus

nocternal neuresis ,had abnormality in renal scan : what he had : duplex kidney ,cyst kidney,horsehole kidney 188
child had chest infection:on exam : stony dull chest :what investigation: CT Scan, U/S chest,and other option
U/S chest
189
12 yrs old took 6 tabs of paracetamol,bulling in school,level under treatment,plan to dischage, how you reasure the
parent he is ready to go home ?? child not on child protection list,,,,,,, he didn't try to suicidal before.,other
option ,,,,,,,,,,,,,,, support mother Psychiatrist assessment
————————————- 190
aneamia and inestigative 191
Other options in paracetam Qs

Supporting mother
192

senario about celiac type of HLA ? 193


t is well known that CD is strongly associated with specific HLA class II genes known as HLA-DQ2 and HLA-DQ8 located on
chromosome 6p21.

Guidelines for CD diagnosis from the European Society for Paediatric Gastroenterology, Hepatology, and Nutrition
(ESPGHAN) propose the option to omit the duodenal biopsy in the diagnosis of CD if all 4 of the following criteria are met in
children and adolescents[4, 5, 6, 7] :

1. Signs and symptoms suggestive of CD


2. Anti-transglutaminase type 2 antibody (anti-TG2) levels more than 10 times the upper limit of normal
3. Positive confirmation tests of anti-endomysium-IgA antibodies (EMA)
4. At-risk HLA-DQ2 or HLA-DQ8

All FOP Recall till June 2021 with note 369 of 378
36
October Exam 2013
194
ASM
aboy hitby abicycle develop progressive drop foot : nerve supply : common peroneal nerve

195
Colle's fracture .......loss of sensation of in palm : nerve supply : median .

196
suxamethonium contraindcation in : a. mutiple sclorosis,b. ducheane muscular destrophy,c. spinal bifida occulta,d.
spinal muscular atrophy type 3
Succinylcholine # in any
muscular disease

succinylcholine (Rx)Anectine, suxamethonium


• Classes: Neuromuscular Blockers, Depolarizin
Contraindications
Malignant hyperthermia, lack of ventilatory support, ocular surgery, penetrating eye injuries, closed-angle glaucoma,
genetically determined disorders of plasma pseudocholinesterase, history of malignant hyperthermia, myopathies associated
with elevated serum creatine kinase

tuberculin test type of hypersensivity reaction : type 1 to 5 answrType 4 hypersensitivity 197


motor and sesory ( both ) branch of trigeminal nerve : a. maxillary,b.mandibular,c. optic, d. occulomoter 198
answer mand

199
1st cranial nerve to be affacted by incease intracranial pressuer : optic and other options

All FOP Recall till June 2021 with note 370 of 378
37
October Exam 2013 ASM
vassopresin effect ,there data about NA osmolality of urin ( baby has 2 hrly feeds thristy,wet diappers,normal

developmental,his height and weight of 0.2% ,0.4 % of centile ),


200
The single most important effect of antidiuretic hormone is to conserve body water by reducing the loss of water
in urine. A diuretic is an agent that increases the rate of urine formation. Injection of small amounts of antidiuretic
hormone into a person or animal results in antidiuresis or decreased formation of urine, and the hormone was
named for this effect.

Antidiuretic hormone stimulates water reabsorbtion by stimulating insertion of "water channels"


or aquaporins into the membranes of kidney tubules. These channels transport solute-free water through tubular
cells and back into blood, leading to a decrease in plasma osmolarity and an increase osmolarity of urine.

data num.before water depravation test and after vasoperssin ( plasma osmolality )

he asked about diagnosis: a,neprogenic DI , b. central DI ,c. SIADH,d. water toxication

201 1000 people of reseach ,new test of CF ,out of 1000 .............10 detected positive of which 2 with disease ,what is
nagative predicted value ?? a. 10%,b. 80%,c.99.2%

May answer is 2diseased of 10 positive test so 20%

202 neonatal screen or test to detect CF : immunoreactive trypsin,b. alpa 1 anti-trpsin , c. fecal elastase
All FOP Recall till June 2021 with note 371 of 378
38
October Exam 2013 ASM

A sample of blood is either taken from the bottom of the baby's foot or a vein in the arm. A tiny drop of
blood is collected onto a piece of filter paper and allowed to dry. The dried blood sample is sent to a lab for
analysis.

The blood sample is examined for increased levels of immunoreactive trypsinogen (IRT). This is a protein
produced by the pancreas that is linked to CF.
* neonatal screen do IRT if positive —> repeat IRT —> if still positive — genetic test for confirmation
by sweat test not done in neonate
* child do sweet test as confirmation

203 grandmother has give antipyretic ........child developed bleeding ,incease APTT (53 sec ),febrin 0.5 , what the diagnosis
: a. DIC ,b. asprin toxicity
normal level of fibrinogen 1.5-4

204 ‫ﺳﺆال ﻛﻮﯾﺲ‬


Aspirin chronic doses cause increase apttt not one dose

Severe, rapid-onset DIC causes severe thrombocytopenia, very prolonged PT and PTT, a rapidly declining
plasma fibrinogen level, and a high plasma D-dimer level.

So answer may be DIC

205 4 months old with ECG axis ( 280-350):diagnosis : a. AVSD,b, ASD,c. VSD ,d. PDA,e. tetraloy of fallot
superior axis deviation AVSD

Answer may be AVSD as it always has Rt vent hyper trophy !!!!!

Extreme Axis Deviation


• Ventricular rhythms – e.g.VT, AIVR, ventricular ectopy

• Hyperkalaemia

• Severe right ventricular hypertrophy

206 baby has GE developed heart faluire : organisum :strept ,coxasacki


viral myocarditis

207 baby had high pitch cty,CSF analysis: growth of gram positive rod : organasim : listeria,HIB ,E.coli, sterpt,staph
Listeria

208 extended match : treatment of samonlla ,Rx of listeria ,Rx of necrotis fascitis swab growth of MRSA : options :
cefriaxon,cefotaxime,amoxicillin,pencilin,vancomycin gentamin Salmonella —> ceftriaxone
Listeria —> Amicicillin
MRSA —> Vancomycin

209 newborn on gantamcin twice daily :

All FOP Recall till June 2021 with note 372 of 378
39
October Exam 2013 ASM
A the dose andA interval ,A the doas and lenght the interval ,A the dase and interval ,no change

210 baby born after 6 days lethrgic ,after 9 days developed vesciles : HSV,varicella,staph,listeria
Review

211child had obesity ,dislipideamia,hyperglyceamia,and .................,what the risk factor for her

when she was born :

a. perterm 2.5 kg @ 1 yr old 7.5 kg

b. perterm 2.5 kg @ 1 yr old 10 kg

c. term 2.5 kg @ 1yr old 10 kg Metabolic syndrome


Term low birth wt
d. term 2.5 kg @ 1 yr old 7.5 kg
at one year double the
e. term 4.5 kg @ 1 yr old 10 kg wt 3time

212 boy playing football,suden syncope,become well and neurological normal ,what is the best investigation to giagnosis:
EEG,MRI,ECG,Blood sugar.

213 Build 3cubes,palmar grasp,know his name,....

What is develpmental age for the child 18 month

214 Stummering child the cause may be

Autisic spectrum disorders

Bilingual parents

Tie tongue

‫ﻓﻰ ﺳﻮال‬suxamethonium contraindications for DMD will lead more hypotonia


anaesthesia malingnant hyperthermia
antidote dantrolene
215 ‫ﺗﺎﻧﻲ اﻟﺒﻨﺖ اﻟﻌﻨﺪھﺎ‬limp pain

216 6tabs paracetamol and he is stable level not reach toxic dose what is your action plan
suicidal attempts if adult —> psychology
if child —> abuse

217 Sudden infant death syndrome

Risk decrease if mother smoke outdoor

All FOP Recall till June 2021 with note 373 of 378
40
October Exam 2013 ASM
You need metabolic screening for the next baby

It decrease die to nursing in prone position to avoid respiratory obstruction

Use of apnea alarm decrease its incidence

Mother need to train about cpr

‫ ﻓﻲ ﺳﺆال‬statistic

‫ﺑﺘﺎع ال‬best types of study design

according to following order

218 ‫ اﻟﺸﺎﻓﻊ اﻻﻣﻮ‬seldomly smile when she see him Myotonic dystrophy

219 ‫ﻓﻲ واﺣﺪ ﺑﺘﺎع اﻟﻄﻔﻞ اﻟﻠﻤﻦ ﯾﻨﻮم ﺑﺠﯿﮫ‬

Scissoring in leg; arching of back, crying,,,,,,, Sandyfer Syndrome

220 ‫ﺑﺘﺎع ال‬vegetarian mother and child come with bilous vomiting

Extended maching

221vitamin ‫ وواﺣﺪ ﺑﺘﺎع اﻷم اﻟﺒﺘﺪي اﻟﻄﻔﻞ ﻟﺒﻨﮭﺎ ﺑﺎل‬NG tube‫وﺟﺎه ﺷﻨﻮ ﻛﺪة ﻣﺎﻋﺎرﻓﮫ‬

The most likely organism * NGT —> staph aureus coagulase +ve flora
* IV cannula and shunt —> staph epidermis coagulase -ve
* Laban if not sterile —> klebsiella, E. coli

222 ‫ ﺑﺘﺎع اﻟﻄﻔﻞ اﻟﺠﺎﺑﻮه أھﻠﮫ وﻗﺎﻟﻮ وﻗﻊ ﻣﻦ اﻟﺴﺮﯾﺮ ﻟﻘﻮا ﻋﻨﺪه‬spiral fracture

Action

Ask senior

Tell parents possibility of NAI waiting for radiology report

Make skeletal survey

All FOP Recall till June 2021 with note 374 of 378
41
October Exam 2013 ASM
‫ ﻓﻲ ﻛﻢ ﺳﺆال ﻛﺎن‬child physical abuse

223 ‫ﻓﻲ ﺳﺆال ﺑﺘﺎع ال‬notric ...test

To detect smoking
Nitric oxide test - increase in 1ry ciliary dyskinesia, BA
Severe asthma - decrease in kartagner, CF, allergic rhinitis

224 ‫وﺳﺆال اﻷزﻣﺔ اﻷدوه ﻛﻞ اﻟﻌﻼﺟﺎت ﻣﺎﻧﻔﻌﺖ ﻣﻌﺎه وﺑﺘﺰﯾﺪ ﻣﻊ اﻟﺮﯾﺎﺿﺔ‬

‫ﺗﺸﺨﯿﺼﮭﺎ ﺷﻨﻮ‬ Exercise induce asthma


Variant asthma

225 Murmer split with respairation fixed split ASD

226 Mechanism if action of PPI

Hydrogen potassium ATP

227Negative predictive value

228 ‫ ﺳﻨﮫ وﺟﺎت‬13 ‫اﻣﮭﺎ ﻗﺎﻟﺖ وﻗﻌﺖ ع اﻟﻌﺠﻠﮫ وﻛﺎن ﻣﻌﺎھﺎ اﺧﻮﻋﺎ ﻋﻤﺮو‬bleeding from vulva

Whats is ur action

Suture

Send to obs and gynea

Call child protection

Call ur senior pediatricion

All FOP Recall till June 2021 with note 375 of 378
42
October Exam 2013 ASM

229 8yrs came wih ataxia and convulsion

Best inv is

Ct

Mri exclude post cerebellum and brain


tumour
Eeg

Chromosomal breakage

230 Girl her father abused her sexually came with ‫طﺎﺷﮫ وﻣﺎﻋﺎرﻓﮫ ﺷﻲ واﻻ ﺳﺎﻗﻮھﺎ ﻟﻠﻜﺮﺳﻲ وﻓﺠﺎه‬

‫(ﺑﻘﺖ ﺗﺤﺮك ﯾﺪﯾﻨﺎ ﺣﺮﻛﺎت ورا ﺑﻌﺾ ووﻗﻔﺖ‬funny spills) post traumatic stress

Whts best investigation

Mri

Ct

Eeg

Refer to phychtrist

231‫ﻓﻲ اﻟﺴﻨﺎرﯾﻮ ﺑﺘﺎع اﻟﻄﻔﻞ اﻟﻤﺎﺑﯿﻘﺪر ﺑٮﯿﻄﻠﻊ اﻟﺴﻠﻢ‬

‫ﻛﺎن ﻛﻮﯾﺲ‬start walking at 15months then develop motor regression

proceded by upper respiratory infection

The diagnosis is ‫ﻓﻲ اﻟﺴﻨﺎرﯾﻮ ﺑﺘﺎع اﻟﻄﻔﻞ اﻟﻤﺎﺑﯿﻘﺪر ﺑٮﯿﻄﻠﻊ اﻟﺴﻠﻢ‬

‫ﻛﺎن ﻛﻮﯾﺲ‬start walking at 15months then develop motor regression

‫ اى دا ﺑﺘﺎع‬proceded by upper respiratory infection

duchen muscular dystrophy

dystrophyduchen muscular dystrophy

Becker muscular dystrophy

Chronic fatigue syndrome


GBS

232 Female with short stature examination is normal

Mid parental height at 25% Turner mid parents normal


All FOP Recall till June 2021 with note 376 of 378
43
October Exam 2013 ASM
Her ht 0.4 centile

Xo not turner dt normal examination

X linked recessive

X linked dominant

Autosomal recessive

Multifactorial = ( genetic + environment)

233 Mother has cataract her uncle has problem

Her baby boy has myopathy

What is inheritance

Mitochondrial

X linked recessive

Xlinked dominant

Autosomal recesive
Mayotonic dystrophy = AD

234 A newborn on examination found to have ambiguous genitalia with hypospadias

Parents want to name their baby what is your action plan:

Tell them the sex of baby is uncertain and advice them to wait

Tell them to choose a name match for both girl and boy

Tell them to ascertain sex after karyotyping

End of june2015 exam

All FOP Recall till June 2021 with note 377 of 378
44
October Exam 2013 ASM

All FOP Recall till June 2021 with note 378 of 378
45

You might also like